Sunteți pe pagina 1din 56

Study Notes – Surgery James Lamberg 28Jul2010

Textbooks: Essentials of Surgery, Abernathy Surgical Secrets, First Aid for Surgery Clerkship
Free Practice Suturing Board: ETHICON 1-800-255-2500 (Registered Nurse Services, Johnson & Johnson)
--------------------------------------------------------------------------------------------------------------------------------------------
Common Problems in Surgery
General: Shock, Trauma, Acute Abdomen and Appendicitis, Hernia, Gastrointestinal Hemorrhage, Peptic Ulcer
Disease, Biliary Tract Disease, Pancreatitis, Abdominal Masses, Colonic and Ano-rectal Disease, Arterial Occlusive
Disease, Breast Masses
Endocrine: Thyroid, Parathyroid, Adrenal, Pancreas, Testicle, Ovary
Cancer: Breast, Colon, Gastric, Pancreatic, Lung, Skin
--------------------------------------------------------------------------------------------------------------------------------------------
Procedures: NEJM Videos In Clinical Medicine: http://www.nejm.org/multimedia/videosinclinicalmedicine
Sterile Gloving by Jennifer Baler, RN, MSN, CCRN: http://www.youtube.com/watch?v=pAKZ3mdFIj4
Gowning & Surgical Scrub by Valencia Cardiovascular Technology: http://preview.tinyurl.com/348vkpn
--------------------------------------------------------------------------------------------------------------------------------------------
Some Tips For Your First Day In The Operating Room (From Clerkship Students)
Scrub Cap Rule: You cannot enter an O.R., regardless of what is happening, without a scrub cap (hair cover) on.
Mask Rule: If a patient is in the O.R., you have to wear a mask in the room.
Fogging: You may find no matter what type of mask you use in the O.R., whether it be a combined shield or with
separate safety goggles, that your vision still fogs up. This is difficult to control due to the heat of your breath and
the cool O.R. air. One trick is to put some tape over your nose and the nose-bridge of your mask, sealing your mask
to your face. The latex-free brown tape works very well and doesn’t leave residue on your face.
Handling Tools: Learn the correct way to hold a scissor and needle holder (not between your thumb and
pointer/middle finger). Watch how the doctors hold tools.
Measurements: It’s good to know what 1mm and 1cm look like. 1cm is about the width of your pinky nail.
Scrub Nurse: They will save you; they know all the ins and out of the O.R. and can help you correctly gown, know
where to stand, how to hold tools, and what the names of the tools are.
Safe Spot: “Surgery is a hands-on sport…if your hands are not on something, you’re not in the game.” When you are
standing at the table observing, your hands should be placed on the patient, not at your waist or crossed/folded. Note
that many patients will have warming devices on (Bair Hugger) or Sequential Compression Devices (SCDs), which
inflate and deflate. So, you may feel this move under you if you’re at the patient’s legs; it’s not the patient moving.
Knots: Don’t tie any knots unless you are absolutely certain you know how to tie them as square knots.
Other Details: You will learn the names of tools as you go, details of instrument handling, and techniques. Basic
Surgical Skills textbooks (e.g. Mayo Clinic) provide pictures and further explanations if needed.
Rules of Surgery: Eat when you can, sleep when you can, don’t mess with the pancreas.
--------------------------------------------------------------------------------------------------------------------------------------------
How To Succeed In Clerkship – First Aid For The Surgery Clerkship (Kaufman, Stead, & Stead)
In The OR: Eat before you begin the case. Pay attention and do a good job, even if you are just holding the retractor.
Some students get light-headed from standing in one position, so make sure to shift your weight and bend your
knees from time to time. This will keep you from fainting. Try to get a list of the cases for the coming day and study
the basics the night before. Find out who you are working with and do a quick bibliography search on the surgeon.
Assess the mood in the OR. Some surgeons will engage students while others will pretend you aren’t there. If your
questions and comments reflect that you have read about the procedure and disease, things will go well. Keep a log
of all surgeries you have attended, scrubbed on, or assisted with.
On The Wards: Most surgical ward teams begin rounding between 6 and 7am. Give yourself at least 10minutes per
patient you are seeing to learn about the events that occurred overnight. Do not make scrubs your uniform unless
specifically told to; dress professionally. The surgical rotation is often difficult, stressful, and tiring. Smooth out
your experience by being nice to be around. Smile a lot and learn everyone’s name. Learn the hierarchy; address
questions about ward functioning to interns or residents and address medical questions to attendings. Address
patients and staff in a respectful way. Take responsibility for your patients, knowing everything there is to know
about them. Be self-propelled and self-motivated. Be a team player and help with tasks, but don’t steal the spotlight,
steal a procedure, or make a fellow medical student look bad. Be honest and never document information that is
false. Keep patient information handy, such s on a small notebook.
Present patient information in an organized manner: “This is a [age] your old [gender] with a history of [major
history] who presented on [date] with [major symptoms], and was found to have [working diagnosis]. [Tests done]
showed [results]. Yesterday the patient [important changes, new plan, new tests, new medications]. This morning
the patient feels [in patient’s words], and physical exam is significant for [findings]. Plan is [state plan].”

DO NOT DISTRIBUTE -1-


Study Notes – Surgery James Lamberg 28Jul2010

Presenting the chest radiograph (CXR): “This is the CXR or Mr. Jones. The film is an AP view with good
inspiratory effort. There is an isolated fracture of the 8th rib on the right. There is no tracheal deviation or
mediastinal shift. There is no pneumo- or hemothorax. The cardiac silhouette appears to be of normal size. The
diaphragm and heart borders on both sides are clear; no infiltrates are noted. There is a central venous catheter
present, the tip of which is in the superior vena cava.”
Procedure note: “Under sterile conditions following anesthesia with 5cc of 2% lidocaine with epinephrine and
negative wound exploration for foreign body, the laceration was closed with 3-0 Ethilon sutures. Wound edges were
well approximated and no complications occurred. Wound was dressed with sterile gauze and triple antibiotic
ointment.”
Operative Notes: It is also useful to learn how to write preoperative, operative, and postoperative notes. However,
interns and residents will likely be writing these.
Your Rotation Grade: Inpatient evaluation includes ward time with residents and attendings, may be worth 50% of
your grade, and is largely subjective. Ambulatory evaluation includes clinic performance and clinical notes. Written
examination, such as NBME or “Shelf” exam. Objective structures clinical examination (OSCE) includes a student’s
bedside manner and physical exam skills, which could comprise up to 25% of the grade.
Study: If you see a patient with a thyroid mass, read about Graves disease, Hashimoto’s, thyroid cancer, and the
technique of needle aspiration.
Prepare a talk: You may be asked to give a small talk during your rotation. If not, you should volunteer. The ideal
topic is slightly uncommon but not rare, for example: bariatric surgery.
Lights: Study in a bright room to help prevent you from falling asleep.
Eating: Make sure your meals are balanced, with lean protein, fruits and vegetables, and fiber. A high-sugar, high-
carbohydrate meal will give you an initial burst of energy for 1 to 2 hours, but then you’ll drop.
--------------------------------------------------------------------------------------------------------------------------------------------
Survival Guides – Netter Anatomy & Student Consult
Most frequently encountered cases: groin lumps (hernias), varicose veins, neck masses, breast lumps, jaundice,
bowel obstruction. Other topics for attending quizzes include: anesthesia, analgesia, antibiotics, IV fluids.
When studying a topic, organize as: Definitions, Classifications, Epidemiology, Etiology, Clinical Presentation,
Differential Diagnosis, Investigations/Tests, and Management.
Be Safe: Assume a mature and professional role, know patient’s history/vital signs/investigation results, don’t do
procedures you are unsure of, ask questions when in doubt, most important person in the room is the patient.
Be Sensible: Admit you don’t know the answer, don’t rush, don’t argue.
Be Sharp: Dress smartly in respect of your patients and teachers, be accurate and brief when answering or asking a
question (“speak up then shut up”).
Be Seen: Come early and leave late (but do get some sleep), ask if you can help, don’t ask “when can I go home?”
Do’s & Don’ts: Attend ward rounds, check inpatients on the ward and present them on ward rounds, get to know the
inpatients and follow their progress, make presentations at meetings or discussions, help out with ward duties, see
outpatients at clinics (do history, examination, differential diagnosis, and decide on investigations), if possible see
patients in the emergency department, attend surgeries to learn (not just to be present), spend your time wisely for
your own gain, know the rules of the operating room (wear hat, mark, correct footwear, wear your ID badge, remove
jewelry, know how to scrub in or ask someone with experience)
--------------------------------------------------------------------------------------------------------------------------------------------
Top 100 Secrets – Abernathy Surgical Secrets (5th, Harken & Moore)
1) Clinical determinants of brain death are the loss of the papillary, corneal, oculovestibular, oculocephalic,
oropharyngeal, and respiratory reflexes for > 6 hours. The patient should also undergo an apnea test, in which the
pCO2 is allowed to rise to at least 60 mmHg without coexistent hypoxia. The patient should be observed for the
absence of spontaneous breathing.
2) The estimated risks of HBV, HCV, and HIV transmission by blood transfusion in the United States are 1 in
205,000 for HBV, 1 in 1,935,000 for HCV, and 1 in 2,135,000 for HIV.
3) The most common location of an undescended testicle is the inguinal canal.
4) Most common solid renal mass in infancy is congenital mesoblastic nephroma and in childhood Wilms' tumor.
5) Ogilvie's syndrome is acute massive dilatation of the cecum and the ascending and transverse colon without
organic obstruction.
6) The best screening for prostate cancer is digital rectal exam combined with serum prostate-specific antigen.
7) The most common histologic type of bladder cancer is transitional cell carcinoma.
8) Carcinoma in situ of the bladder is treated with immunotherapy with intravesical bacillus Calmette-Guérin.
9) Localized renal cell carcinoma is treated with surgery (radical nephrectomy).

DO NOT DISTRIBUTE -2-


Study Notes – Surgery James Lamberg 28Jul2010

10) The most common cause of male infertility is varicocele.


11) The most common nonbacterial cause of pneumonia in transplant patients is cytomegalovirus.
12) Chimerism is leukocyte sharing between the graft and the recipient so that the graft becomes a genetic
composite of both the donor and the recipient.
13) OKT3 is a mouse monoclonal antibody that binds to and blocks the T-cell CD3 receptor.
14) The most common disease requiring liver transplant is hepatitis C.
15) Cystic hygroma is a congenital malformation with a predilection for the neck. It is a benign lesion that usually
presents as a soft mass in the lateral neck.
16) In neuroblastomas, age at presentation is the major prognostic factor. Children younger than 1 year have an
overall survival rate > 70%, whereas the survival rate for children older than 1 year is < 35%.
17) The most feared complication of diaphragmatic hernia is persistent fetal circulation.
18) The three most common variants of tracheoesophageal fistula are (1) proximal esophageal atresia with distal
tracheoesophageal fistula, (2) isolated esophageal atresia, and (3) tracheo-esophageal fistula with esophageal atresia.
19) Atresia can occur anywhere in the GI tract: duodenal (50%), jejunoileal (45%), or colonic (5%). Duodenal
atresia arises from failure of recanalization during the 8th-10th week of gestation; jejunoileal and colonic atresia are
caused by an in utero mesenteric vascular accident.
20) The types of aortic dissection are ascending (type A) dissection, which involves only the ascending or both the
ascending and descending aorta, and descending dissection (type B), which involves only the descending aorta.
21) A solitary pulmonary nodule is < 3 cm and is discrete on chest radiograph. It is usually surrounded by lung
parenchyma.
22) Mediastinal staging is indicated in patients with apparent or documented lung cancer who have (1) known lung
cancer with mediastinal nodes > 1 cm accessible by cervical mediastinal exploration, as assessed by CT scan; (2)
adenocarcinoma of the lung and multiple mediastinal lymph nodes < 1 cm; (3) central or large (> 5 cm) lung cancers
with mediastinal lymph nodes < 1 cm; and (4) lung cancer with risk of thoracotomy and lung resection.
23) The most common causes of aortic stenosis are now congenital anomalies and calcific (degenerative) disease.
24) In mitral regurgitation, the left ventricle ejects blood via two routes: (1) antegrade, through the aortic valve, or
(2) retrograde, through the mitral valve. The amount of each stroke volume ejected retrograde into the left atrium is
the regurgitant fraction. To compensate for the regurgitant fraction, the left ventricle must increase its total stroke
volume. This ultimately produces volume overload of the left ventricle and leads to ventricular dysfunction.
25) The indications for CABG are (1) left main coronary artery stenosis; (2) three-vessel coronary artery disease
(70% stenosis) with depressed left ventricular (LV) function or two-vessel coronary artery disease (CAD) with
proximal left anterior descending (LAD) involvement; and (3) angina despite aggressive medical therapy.
26) Hibernating myocardium is improved by CABG. Myocardial hibernation is reversible myocardial contractile
function associated with a decrease in coronary flow in the setting of preserved myocardial viability. Some patients
with global systolic dysfunction exhibit dramatic improvement in myocardial contractility after CABG.
27) The surgical treatment of ulcerative colitis is total colectomy with ileoanal pouch anastomosis.
28) Dieulafoy's ulcer is a gastric vascular malformation with an exposed submucosal artery, usually within 2-5 cm
of the gastroesophageal junction. It presents with painless hematemesis, often massive.
29) The role of blind subtotal colectomy in the management of massive lower gastrointestinal bleeding is limited to
a small group of patients in whom a specific bleeding source cannot be identified. The procedure is associated with a
16% mortality rate.
30) Colorectal polyps < 2 cm have a 2% risk of containing cancer, 2 cm polyps have a 10% risk, and polyps > 2 cm
have a cancer risk of 40%. Sixty percent of villous polyps are > 2 cm, and 77% of tubular polyps are < 1 cm at the
time of discovery.
31) Patients with colorectal cancer with lymph node involvement (Dukes' C) should receive chemotherapy
postoperatively to treat micrometastases.
32) Goodsall's rule states the location of the internal opening of an anorectal fistula is based on the position of the
external opening. An external opening posterior to a line drawn transversely across the perineum originates from an
internal opening in the posterior midline. An external opening, anterior to this line, originates from the nearest anal
crypt in a radial direction.
33) Incarcerated inguinal hernia: structures in the hernia sac still have a good blood supply but are stuck in the sac
because of adhesions or a narrow neck of the hernia sac. Strangulated inguinal hernia: hernia structures have a
compromised blood supply because of anatomic constriction at the neck of the hernia.
34) Chvostek's sign is spasm of the facial muscles caused by tapping the facial nerve trunk. Trousseau's sign is
carpal spasm elicited by occlusion of the brachial artery for 3 minutes with a blood pressure cuff.
35) The two surgical options for Graves' disease are subtotal thyroidectomy or near-total thyroidectomy.

DO NOT DISTRIBUTE -3-


Study Notes – Surgery James Lamberg 28Jul2010

36) The only biochemical test that is routinely needed to identify patients with unsuspected hyperthyroidism is
serum thyroid-stimulating hormone concentration.
37) The surgically correctable causes of hypertension are renovascular hypertension, pheochromocytoma, Cushing's
syndrome, primary hyperaldosteronism, coarctation of the aorta, and unilateral renal parenchymal disease.
38) The "triple negative test" or "diagnostic triad" for diagnosing a palpable breast mass includes physical
examination, breast imaging, and biopsy.
39) Chest wall radiation is indicated after mastectomy in patients with greater than 5 cm primary cancers, positive
mastectomy margins, or more than four positive lymph nodes, all of which are associated with heightened
locoregional recurrence rates.
40) Sentinel lymph nodes are the first stop for tumor cells metastasizing through lymphatics from the primary tumor.
41) The most common site of origin of subungual melanomas is the great toe. Amputation at or proximal to the
metatarsal phalangeal joint and regional sentinel lymph node biopsy are advised by most authors.
42) Ramus marginalis mandibularis, the lowest branch of the nerve that innervates the depressor muscles of the
lower lip, is the most commonly injured facial nerve branch during parotidectomy.
43) Waldeyer's ring is the mucosa of the posterior oropharynx covering a bed of lymphatic tissue that aggregates to
form the palatine, lingual, pharyngeal, and tubal tonsils. These structures form a ring around the pharyngeal wall.
This may be the site of primary or metastatic tumor.
44) A patient in whom the head and neck examination is completely normal but FNA of a cervical node reveals
squamous cancer should have examination of the mouth, pharynx, larynx, esophagus, and tracheobronchial tree
under anesthesia (triple endoscopy). If nothing is seen, blind biopsy of the nasopharynx, tonsils, base of tongue, and
pyriform sinuses should be done at the same sitting.
45) The microorganisms implicated in atherosclerosis include Chlamydia pneumoniae, Helicobacter pylori,
streptococci, and Bacillus typhosus.
46) The cumulative 10-year amputation rate for claudication is 10%.
47) The absolute reduction in risk of stroke is 6% over a 5-year period in asymptomatic patients with > 60% stenosis
who undergo carotid endarterectomy plus aspirin versus patients treated with aspirin alone (5.1% versus 11%).
48) Abdominal aortic aneurysm's average expansion rate is 0.4 cm/year.
49) Heparin binds to antithrombin III, rendering it more active.
50) The patient with suspected intermittent claudication should initially be evaluated by obtaining ankle brachial
index or segmental limb pressures at rest.
51) Shock is suboptimal consumption of O2 and excretion of CO2 at the cellular level.
52) Nitric oxide is synthesized in vascular endothelial cells by constitutive nitric oxide synthase and inducible NOS,
using arginine as the substrate.
53) Saliva has the highest potassium concentration (20 mEq), followed by gastric secretions (10 mEq), then
pancreatic and duodenal secretions (5 mEq).
54) Basal caloric expenditure = 25 kcal/kg/day with a requirement of approximately 1 g protein/kg/day.
55) 6.25 g of protein contains 1 g of nitrogen.
56) Dextrose has 3.4 kcal/g, protein 4 kcal/g, fat 9 kcal/g (20% lipid solution delivers 2 kcal/mL).
57) Maximal glucose infusion rates in parenteral formulas should not exceed 5 mg/kg/min.
58) Refeeding syndrome occurs in moderately to severely malnourished patients (e.g., chronic alcoholism or
anorexia nervosa) who, upon presentation with a large nutrient load, develop clinically significant decreases in
serum phosphorus, potassium, calcium, and magnesium levels. Hyperglycemia is common secondary to blunted
insulin secretion. ATP production is mitigated, and the classic presentation is respiratory failure.
59) Glutamine is the most common amino acid found in muscle and plasma. Levels decrease after surgery and
stress. Glutamine serves as a substrate for rapidly replicating cells (interestingly, it is also the number one metabolic
substrate for neoplastic cells), maintains the integrity and function of the intestinal barrier, and protects against free
radical damage by maintaining GSH levels. Glutamine is unstable in IV form unless linked as a dipeptide.
60) Fever is caused by activated macrophages that release interleukin-1, tumor necrosis factor, and interferon in
response to bacteria and endotoxin. The result is a resetting of the hypothalamic thermoregulatory center.
61) Cardiac output = heart rate x stroke volume; normal CO is 5-6 L/min.
62) SVR = [(MAP - CVP)/CO] x 80; normal SVR is 800-1200 dyne.sec/cm-5.
63) Hypovolemic shock: low CVP and PCWP, low CO and SVO2, high SVR.
64) Cardiogenic shock: high CVP and PCWP, low CO and SVO2, variable SVR.
65) Septic shock: low or normal CVP and PCWP, high CO initially, high SVO2, low SVR.
66) Kehr's sign is concurrent LUQ and left shoulder pain, indicating diaphragmatic irritation from a ruptured spleen
or subdiaphragmatic abscess. The diaphragm and the back of the left shoulder enjoy parallel innervation.

DO NOT DISTRIBUTE -4-


Study Notes – Surgery James Lamberg 28Jul2010

67) Rebound tenderness implies peritoneal inflammation and irritation not simply abdominal tenderness.
68) The 5 Ws of post-operative fever are wound (infection), water (UTI), wind (atelectasis, pneumonia), walking
(thrombophlebitis), and wonder drugs (drug fevers).
69) Cricothyroidotomy should not be performed in patients < 12 years old or any patient with suspected direct
laryngeal trauma or tracheal disruption.
70) The radial (wrist) pulse estimates SBP > 80 mmHg; femoral (groin) pulse estimates SBP > 70 mmHg; and
carotid (neck) pulse estimates SBP > 60 mmHg.
71) A general rule for crystalloid infusion to replace blood loss is a 3:1 ratio of isotonic crystalloid to blood.
72) Raccoon eyes (periorbital ecchymosis) and Battle's sign (mastoid ecchymosis) are clinical indicators of basilar
skull fracture.
73) CPP = MAP - ICP. Some debate exists on the minimum allowable CPP, but consensus indicates that a cerebral
perfusion pressure of 50-70 mmHg is necessary.
74) Violation of the platysma defines a penetrating neck wound.
75) Tension pneumothorax is air accumulation in the pleural space eliciting increased intrathoracic pressure and
resulting in a kinking of the SVC and IVC that compromises venous return to heart.
76) Most common site of thoracic aortic injury in blunt trauma is distal to the take-off of the left subclavian artery.
77) The most common manifestation of blunt myocardial injury is arrhythmia.
78) Indications for thoracotomy in a stable patient with hemothorax include an immediate tube thoracostomy output
of > 1500mL and ongoing bleeding of 250mL/h for 4 consecutive hours.
79) Beck's triad is hypotension, distended neck veins, and muffled heart sounds.
80) The hepatic artery supplies approximately 30% of blood flow to the liver while the portal vein supplies the
remaining 70%. The oxygen delivery, however, is similar for both at 50%.
81) The Pringle maneuver is a manual occlusion of the hepatoduodenal ligament to interrupt blood flow to the liver.
82) Splenectomy significantly decreases IgM levels.
83) 90% of trauma fatalities due to pelvic fractures are due to venous bleeding and bone oozing; only 10% of fatal
pelvic bleeding from blunt trauma is arterial (most common site is superior gluteal artery).
84) Intraperitoneal bladder rupture from blunt trauma: operative management. Extraperitoneal rupture: observant
management.
85) Pseudoaneurysm is a disruption of the arterial wall leading to a pulsatile hematoma contained by fibrous
connective tissue (but not all three arterial wall layers, which defines a true aneurysm).
86) The earliest sign of lower extremity compartment syndrome is neurologic in the distribution of the peroneal
nerve with numbness in the first dorsal webspace and weak dorsiflexion.
87) Posterior knee dislocations are associated with popliteal artery injuries and are an indication for angiography.
88) Management of suspected navicular fracture despite negative radiography is short-arm cast and repeat x-ray in 2
weeks; at high risk for avascular necrosis.
89) Parkland formula: lactated Ringer's at 4mL/kg x %TBSA (second- and third-degree only) of burn. Infuse 50% of
volume in first 8 hours and the remaining 50% over the subsequent 16 hours.
90) The metabolic rate peaks at 2.5 times the basal metabolic rate in severe burns > 50% TBSA.
91) Gallstones and alcohol abuse are the two main causes of acute pancreatitis.
92) Alcohol abuse accounts for 75% of cases of chronic pancreatitis.
93) Isolated gastric varices and hypersplenism indicate splenic vein thrombosis and are an indication for
splenectomy.
94) The treatment for gallstone pancreatitis is cholecystectomy and intraoperative cholangiogram during the same
hospital stay once the pancreatitis has subsided.
95) Proton pump inhibitors irreversibly inhibit the parietal cell hydrogen ion pump.
96) Definitive treatment of alkaline reflux gastritis after a Billroth II includes a Roux-en-Y gastro-jejunostomy from
a 40-cm efferent jejunal limb.
97) Cushing's ulcer is a stress ulcer found in critically ill patients with central nervous system injury. It is typically
single and deep, with a tendency to perforate.
98) Curling's ulcer is a stress ulcer found in critically ill patients with burn injuries.
99) Marginal ulcer is an ulcer found near the margin of gastroenteric anastomosis, usually on the small bowel side.
100) Most common cause of small bowel obstructions is adhesive disease. The second common cause is hernias.
--------------------------------------------------------------------------------------------------------------------------------------------
Suturing Basics – eMedicine Suturing Techniques (MacKay-Wiggan)
Lidocaine: 1% or 2% solution, lasts 30-60 minutes, preferred for vascular disease or immunocompromised.
Lidocaine 2% with Epinephrine: Vasoconstriction, lasts 2-6 hours, not for fingers/toes/penis/nose/earlobes.

DO NOT DISTRIBUTE -5-


Study Notes – Surgery James Lamberg 28Jul2010

Bupivacaine (Marcaine): 0.25% or 0.5% solution, lasts up to 20 hours, cardiotoxic at high doses.
Local anesthetic max doses: Lidocaine is 4.5mg/kg (30cc for 70kg pt), Bupivacaine is 3mg/kg (50cc for 70kg pt).
Anesthetic injection: Use smallest needle possible (25 or 30 gauge), injected through open wound, warming syringe
by rolling in your hand may reduce injection pain, creams (EMLA, ethyl chloride) likely work via placebo effect.
Irrigation: Clean wound with saline or Ringer lactate only, not Betadine or soap, do not shave hair or eyebrows.
Scalpels: #10 for long straight cuts (abdomen), #11 for puncture cuts (central line), #15 for plastic surgery.
Suture size: 4-0 nylon for general skin closure, 5-0 nylon for face, 10-0 is tiny, #5 larger than #1.
Suture needles: Cutting needle for skin, tapered needle for soft tissues, blunt needle for infection risk (e.g. HIV).
Suture material: Absorbable for buried sutures, silk for homeostasis, nylon for general closure.
Needle Driver: Ring finger through loop, middle finger over loops, thumb is not through loop (just pressing it),
index finger extends along the needle holder shaft, sewing is via pronation and supination of hand.
Arming needle: Grasp two-thirds to three-fourths distance from needle tip, needle point faces non-dominant hand.
Simple interrupted stitch: Most common, 1cm gaps (width of pinky fingernail), enter at 90-degrees minimally to
help evert skin edges, “build pyramids, not ditches”, for edges “approximate, don’t strangulate”, scar tissue will
never expand to fix defects/gaps (myofibroblasts), all knots should be tied square, Surgeon knot may be used once.

Key structures to align: Center of laceration, vermillion border, lip philtrum, ear helix, eyebrow, eyelid, hairline.
Signs of infection: Redness (rubor), heat (calor), pain (dolor), odor, purulent drainage.
Suture removal: Grasp knot, cut other side of suture close to skin, pull toward incision to reduce tension.
Running stitch: Not common in the ED, provides even wound tension, one damaged stitch ruins entire suture.
Horizontal mattress stitch: Helps evert edges and reduce wound tension.
Vertical mattress stitch: Helps evert edges and reduce dead space, does not significantly reduce tension.
Half-buried mattress (corner) stitch: For stellate lacerations, preserves blood supply to skin tips.
Subcuticular stitch: Absorbable in healthy tissue, no external marks, minimizes scarring, cover with Steri-Strips.
Deep sutures: Absorbable in healthy tissue, for multilayer closure.
Locking continuous stitch: For airtight or watertight closure, such as stomach, bowel, lung.
--------------------------------------------------------------------------------------------------------------------------------------------
Mayo Clinic Basic Surgical Skills (1999, Sherris & Kern)
Tissue Injury: Wounding has three stages. Inflammation is 1-5 days and has increased erythema, lymphocytes,
proteolytic enzymes causing debridement via leukocytes, and pain. Proliferation is 5-14 days and involves
fibroblasts producing collagen, creating the scar (cicatrix). Scar maturation is 14 days onward and involves some
wound contraction and 95% strength by week six.
Healing Factors: Healing affected by increasing age, increased weight, poor nutrition,
electrolyte imbalances, blood supply, immune responses, chronic illnesses, drugs (antineoplastic,
radiation therapy), and smoking.
Intention: Healing by primary intention involves sutures. Healing by secondary intention
involves natural processes (no sutures), where granulation tissue starts from deeper tissues and
moves outward (4-8 weeks). Tertiary intention is recommended for dirty wounds, and involves
secondary intention then suturing (delayed suturing).
Tension Lines: Incisions should be made parallel to relaxed skin tension lines to minimize
scarring. Undermining with scissors below the skin can help to relax the skin so that it is under less tension.
Dead Space: Deep wounds should be fully closed to prevent dead-space, which causes dehiscence (breakdown).
Kern’s Rule: Percent of anesthetic * 10 = mg of drug per mL. E.g. 2% lidocaine is 20mg lidocaine/cc.
Drawing Anesthetics: Since medication bottles are pressurized, you will need to inject air into the bottle in the
amount of fluid you want to remove. So, to draw 3mL of lidocaine, you would attach the needle to the syringe, pull
in 3cc of air, insert the syringe into the lidocaine bottle held upside down, inject 3cc into the bottle, then draw out
3cc of lidocaine. Use a small needle (27g or 30g) and inject slowly.

DO NOT DISTRIBUTE -6-


Study Notes – Surgery James Lamberg 28Jul2010

Pre-Scrubbing: Since you can’t touch your face after you scrub, you must have your cap, mask, and eye protection
on before scrubbing. Tape over your nose and the mask edge can help prevent fogging.
Scrub Sinks: Sinks are usually activated by automatic sensor, a knee push, or a foot petal.
Drying: Post-scrub, enter the O.R. backwards with hands up. The scrub nurse should hand you a towel. Keep your
hands held outward so the towel bottom doesn’t touch your scrubs. Dry from fingertips down to elbow.
Gloves: Common sizes range from 6 1/2 (small) to 8 (large). Ensure your thumbs will line up with the thumbs when
you are putting them on. If your hands are not fully dry, this will be an annoying process.
Scalpels: A #10 blade is held like a violin bow with pointer finger on top for pressure, cutting so the belly of the
blade is at 30-degrees from the skin. A #15 blade is held like a pencil at 45-degrees from the skin.
Tissue Scissors: Tissue (dissection) scissors have blunted tips that curve upward. They are held with the ring finger
through a loop and the thumb through a loop. The middle finger rests for support and the pointer finger points.
Common scissors are the straight Mayo, curved Mayo, and Metzenbaum (Metz).
Forceps: Held like chopsticks. Common forceps are the Adson, Bayonette Bipolar, Bonney, Debakey, Russian, and
Sponge Stick.
Retractors: Pull tissue for visualization. Common retractors are the Army Navy, Baby Bennette, Deaver, Deep
Gelpi, Mayo Body Wall, Nerve Root, Richardson, and Weitlaner.
Hemostat (Clamp): For grasping tissue and vessels. Common clamps are the Allis, Carmalt, Debakey, Jake, Kelly,
Kocher, Mosquito, Right Angle, and Tonsil.
Suction: Some are continuous (no port) and some are intermittent (finger port).
Electrical: There are electrocautery devices and electrosurgical pencils (e.g. Bovie). Used for burning tissue to arrest
bleeding as well as for cutting. Since they are electrical, they do not cut through rubber gloves.
Cutting Sutures: Usually a 3-4mm tail is recommended. Slide down to the knot with the scissors, rotate 90-degress,
and cut. This will ensure you are not directly on the knot and are about 3mm above.
Absorbable Suture Materials:
Tensile Tissue Knot
Name Material Handling Absorption
Strength Reactivity Security
Collagen Beef Tendon Poor +2 Fair Poor 1 to 2 weeks
Plain gut Animal Collagen Poor +4 Fair Poor 1 to 2 weeks
Chromic gut Animal Collagen Poor +3 Fair Fair 1 to 2 weeks
Polyglactin 910 Coated
Coated
Polyglactin 370 and Good +1 Good Fair 3 months
Vicryl
Calcium Sterate
Dixon "S" Polyglycolic Good +1 Fair Good 3 months
PDS Polydioxanone Good +1 Poor Poor 6 months
Monocryl Polyglicaprone 25 Fair +1 Good Good 3 months
Non-Absorbable Suture Materials:
Tissue
Name Material Handling Knot Security
Reactivity
Silk Silk +4 Good Good
Ethilon Polyamide (Nylon) +2 Fair Fair
Dermalon Polyamide (Nylon) +2 Poor Poor
Surgamid Polyamide (Nylon) +2 Poor Poor
Nurolon Polyamide (Nylon) +2 Good Fair
Prolene Polyolefin (Polypropylene) +1 Poor Poor
Mersilene Polyester +2 Good Good
Dacron Polyester +2 Good Good
Ethibond Polyester (Coated Polybutilate) +2 Good Good
Surgical Steel Stainless Steel +1 Poor Good
Nurolon: May predispose to infection. Prolene: Low coefficient of friction. Stainless Steel: May kink.
Square Knots: Use a resource such as the ETHICON Knot Tying Manual or the Conidian Knot Tying Manual.
Practice until your muscle memory prevents you from tying anything but square knots. At one point during the
square knot tie, your hands must cross.
Throws: Most knots hold after 3 to 6 throws. Silk will hold after 3 to 4 throws. Proline requires 5 to 7 throws.

DO NOT DISTRIBUTE -7-


Study Notes – Surgery James Lamberg 28Jul2010

Surgeon’s Knot: This may be used for the first throw. However, it should not be a substitute for good tension. Also,
this knot is technically not square, so the number of throws needed increases by one.
Instrument Tie: Performed by placing the needle holder between the ends of the suture, over the injury line. The long
end is looped over the needle holder and the short end is grasped and pulled through. The next step is repeated again,
with the needle holder in midline and the long end (now on the other side of the wound) looping over the tool.
Free Vessel Ligation: Free vessel end is held by hemostat and first part of square knot is tied and slid down the
hemostat onto the vessel. At least 4 knots should be placed. The end is then cauterized before hemostat release.
Figure-Of-8 Stick Tie: Useful for larger vessels. A needle is passed back and forth in a figure-of-8 fashion and then a
knot is tied starting with a surgeon’s knot. The end is then cauterized before hemostat release.
Bone Wax: Bleeding areas of bone can be controlled with a soft, clay-like material called bone wax.
Halving: A cutaneous wound should be closed by halving. The first suture should divide the wound in half and each
additional sutures should divide into halves as well. This helps keep edges lined up.
Subcutaneous Closure: For deeper wound to prevent dead space, absorbable sutures must be used.
Subcuticular Closure: A buried technique that improves scar appearance.
Purse String: For closing holes. Do not enter the lumen with the suture.
Fusiform (Elliptical) Excision: When excising skin, it is good to have the length be 1.5 to 3 times the width of the
incision. Anything closer to a 1:1 ration will prevent closure without puckering ends (“dog ears”).
Z-Plasty: useful for lengthening contracted scars and reorienting the direction of scars.
Suture Removal: Clip near knot, close to skin, grasp knot, and pull through.
Staple Removal: Using a staple removal tool, put bottom part below staple on skin, then squeeze tool.
--------------------------------------------------------------------------------------------------------------------------------------------
Suturing Basics – First Aid Surgery Clerkship (Stead, Stead, & Kaufman)
Location Suture Size/Type Suture Technique Removal
3-0 or 4-0 nylon or Interrupted in galea, single tight layer in scalp,
Scalp 7-12 days
polypropylene horizontal mattress if bleeding not well controlled
5-0 Vicryl/Dexon in Close perichondrium with interrupted Vicryl and
Pinna 3-5 days
perichondrium close skin with interrupted nylon
4-0 or 5-0 Vicryl (SQ)
Eyebrow Layered closure 3-5 days
6-0 nylon for skin
Single-layer horizontal mattress or simple
Eyelid 6-0 nylon 3-5 days
interrupted
4-0 Vicryl (mucosa)
If wound through lip, close three layers (mucosa,
Lip 5-0 Vicryl (SQ or muscle) 3-5 days
muscle, skin); otherwise do two-layer closure
6-0 nylon (skin)
Simple interrupted or horizontal mattress if
Oral 4-0 Vicryl N/A
muscularis of tongue involved
6-0 nylon (skin) Simple interrupted for single layer, layered
Face 3-5 days
5-0 Vicryl (SQ) closure for full-thickness laceration
4-0 Vicryl (SQ, fat)
Trunk Single or layered closure 7-12 days
4-0 or 5-0 nylon (skin)
3-0 or 4-0 Vicryl (SQ, muscle) Single-layer interrupted or vertical mattress; apply
Extremity 10-14 days
4-0 or 5-0 nylon (skin) splint if over a joint
Single-layer closure with simple interrupted or
Hands/Feet 4-0 or 5-0 nylon 7-12 days
horizontal mattress; apply splint if over a joint
Meticulous placement to obtain even edges, allow
Nail bed 5-0 Vicryl N/A
to dissolve
--------------------------------------------------------------------------------------------------------------------------------------------
Article – Benign Prostatic Hyperplasia (Barry & Roehrborn, BMJ 2001;323:1042–6)
Common symptoms: urinary frequency, urgency, a week and intermittent stream, needing to strain, a sense of
incomplete emptying, and nocturia.
Beneficial interventions: Alpha-blockers, 5alpha reductase inhibitors, transurethral resection (TURP), transurethral
microwave thermotherapy (TUMT), and transurethral needle ablation (TUNA).
Likely to be beneficial: Saw palmetto plant extracts, beta sitosterol plant extracts, and rye grass pollen extracts.

DO NOT DISTRIBUTE -8-


Study Notes – Surgery James Lamberg 28Jul2010

Unknown effectiveness: TURP versus less invasive surgical techniques.


Alpha Blockers: May be more effective than 5alpha reductase inhibitors. May cause dizziness with withdrawal.
5alpha Reductase Inhibitors: May decrease libido (e.g. finasteride).
TURP: Better than watchful waiting and does not increase risk of erectile dysfunction or incontinence. Still, may
cause bleeding, erectile dysfunction, and retrograde ejaculation.
TUMT: Limited evidence that TUMT is more effective than alpha blockers over six months.
TUNA: Fewer side effects than TURP but less symptom improvement.
Saw Palmetto: One review found self-rated improvement was better in those taking saw palmetto.
Beta Sitosterol: One review found improvement in lower urinary tract symptoms in the short term.
Rye Grass: One review found self-rated improvement was better and reduced nocturia in the short term.
--------------------------------------------------------------------------------------------------------------------------------------------
Article – Bowel Obstruction & Pseudo-Obstruction (Kahi & Rex, Gastroenterol N Am 2003;32:1229-47)
Bowel obstruction is due to mechanical interruption of flow while pseudo-obstruction is characterized by dilation of
the bowel in the absence of a causative anatomic lesion.
Small bowel obstructions are usually due to benign lesions while colonic obstruction is usually due to neoplasms.
Peritoneal adhesions following laparotomy account for up to 75% of small bowel obstructions (SBOs).
SBO Symptoms: Abdominal distension, vomiting, constipation, crampy abdominal pain with paroxysms that occur
every 4 to 5 minutes. Proximal obstructions tend to have more nausea and vomiting. Bowel sounds may be
hyperactive and high-pitched (tinkling) initially, but may become hypoactive as motility decreases.
The presence of peritoneal signs suggests strangulation, mandating urgent laparotomy.
Radiographs: Supine abdominal films can help estimate the degree of distension. Upright abdominal films can show
distended loops of small bowel.
Managing SBO: IV fluids, NG tube, early surgical intervention due to low likelihood of spontaneous resolution.
Adenocarcinoma accounts for the majority (60%) of large bowel obstructions (LBOs).
Managing LBO: IV fluids, NG tube, surgical intervention, endoluminal stents for nonresectable tumors.
Volvulus, or torsion, most commonly involves the cecum, followed by the sigmoid colon. Radiographs classically
show a massively dilated, kidney-shaped cecum extending into the left upper quadrant.
Acute colonic pseudo-obstruction (ACPO) is synonymous with acute colonic ileus and Ogilvie’s syndrome. It is
thought to be caused by hypersympathetic activation combined with suppressed parasympathetic activation. There
are a wide variety of causes, such as narcotics, anticholinergics, alcoholism, trauma, and abdominal surgery.
Managing ACPO: IV fluids, NG tube, assess underlying conditions such as medications or electrolyte imbalances.
Ambulation helps; if non-ambulatory use prone or knee-chest position. Perform rectal exam every 6 hours. If
conservative therapy fails, prokinetic agents may be used (e.g. erythromycin, cisapride, metoclopramide, and
neostigmine). With neostigmine, symptomatic bradycardia may occur so atropine should be available. The
effectiveness of neostigmine may decrease the use of colonoscopic decompression. Surgical therapy is a last resort.
--------------------------------------------------------------------------------------------------------------------------------------------
Article – Management of Gallstones (Ahmed & Ramsey, Am Fam Physician 2000;61:1673-80)
Gallstones are a major cause of morbidity worldwide and cholecystectomy is the most commonly performed
abdominal surgery. Up to 90% of stones are cholesterol with the rest being pigmented stones.
Biliary sludge (microlithiasis) is made of mucin, precipitates of cholesterol, and calcium bilirubinate. It usually
precedes the formation of gallstones. Increasing age, female gender, and pregnancy increase risk.
Diabetics have a propensity for obesity, hypertriglyceridemia, and gallbladder hypomotility.
Common duct stones (choledocholithiasis) is most commonly caused by a stone from the gallbladder.
Flukes and parasites (e.g. Asian population) increase risk for bile duct stones.
Differentiating Features of Gallstone-Induced Complications
Biliary Acute Chronic
Feature colic cholecystitis cholecystitis Cholangitis Pancreatitis
Pain site Epigastrium RUQ RUQ RUQ Epigastric
Pain duration < 3 hours > 3 hours Variable Variable Variable
Mass No masses RUQ mass No masses +/- +/-
Fever x +/- +/- +/- +/-
Increased WBC x +/- +/- +/- +/-
Increased amylase Normal +/- x +/- +/-
Biliary colic pain develops suddenly, rapidly increases in intensity over 15 minutes, and can last up to 3 hours.

DO NOT DISTRIBUTE -9-


Study Notes – Surgery James Lamberg 28Jul2010

Complications of gallstones occur 70-80% of the time with biliary colic, 12% with perforation of the gallbladder,
10% with acute cholecystitis, and the rest with disorders such as emphysematous cholecystitis, Mirizzi’s syndrome,
and hydrops of the gallbladder.
Lab testing includes WBC count (leukocytosis with a “left shift”), serum aminotransferase, alkaline phosphatase,
bilirubin, and amylase levels.
Ultrasonography should be a routine examination for the confirmation or exclusion of gallstone disease. Sensitivity
and specificity is >95% for gallstones >2mm in diameter. Ultrasound is less sensitive for choledocholithiasis. A
sonographic Murphy sign may be present.
Endoscopic retrograde cholangiopancreatography (ERCP) is the best method to diagnose choledocholithiasis.
CT and MRI are now comparable to ERCP in terms of accuracy.
Hepatobiliary scintography (HBS) using Tc-99m-IDA, also known as a hepatobiliary iminodiacetic acid (HIDA)
scan, can confirm or exclude acute cholecystitis with a high degree of sensitivity and specificity. Failure to image
the gallbladder within 90 minutes despite adequate views of the liver, common bile duct, and small bowel, strongly
suggests acute obstruction of the cystic duct. The use of cholecystokinin (CCK) during a scan may be helpful for
imaging as well as useful in recreating the same pain symptoms.
Adult patients with silent or incidental gallstones should be observed and managed expectantly.

Non-operative therapies include:


Oral bile acid dissolution (ursodeoxycholic acid): 30-90% stone clearance with zero percent mortality. 50%
recurrence of stones, most useful for stones < 5mm.
Contact solvents (methyl tert-butyl ether/n-propyl acetate): 50-90% stone clearance, 70% recurrence, experimental,
may cause duodenitis, nephrotoxicity, and hemolysis.
Extracorporeal shock-wave lithotripsy (ESWL): 70-90% stone clearance, 70% recurrence, not approved by FDA,
needs a center with expertise, may be costly.
Laprascopic cholecystectomy contraindications include: high risk for general anesthesia, morbid obesity, signs of
gallbladder perforation, giant gallstones, end-stage liver disease with portal hypertension, suspected gallbladder
malignancy, and last trimester of pregnancy.
--------------------------------------------------------------------------------------------------------------------------------------------
Article – Pancreatitis (Mitchell, Byrne, & Baillie, Lancet 2003;361:1447-55)
Inappropriate activation of the proteolytic enzyme trypsin is thought to be the initial step in the development of
pancreatitis. Most patients recover uneventfully, but up to 10-15% may develop systemic inflammatory response
syndrome (SIRS), mediated by cytokines, immunocytes, and the complement system.
Gallstones are the leading cause of acute pancreatitis in developed countries. Alcohol abuse is a close second. Other
causes include drugs, tumors, hypertriglyceridemia, trauma, iatrogenic, and scorpion stings.
Diagnosis of acute pancreatitis is difficult and many cases show at autopsy. Cullen and Grey-Turner signs
(periumbilical and flank bruising, respectively) are rare and non-specific. Increased serum amylase is expected, but
not always seen. CT or ultrasound leads to a correct diagnosis in 81-95% of patients. Lipase estimation is more
sensitive than amylase in alcohol-induced acute pancreatitis. Serum lipase rises within 4-8 hours of an episode,
peaks at 24 hours, and returns to normal after 8-14 days.
An increase in lipase three times the upper limit normally excludes non-pancreatic causes.
Balthazar Scoring uses CT appearances to predict acute pancreatitis severity. The Atlanta Classification bases severe
acute pancreatitis on a Ranson criteria score of 3 or more, or an APACHE II score of 8 or more. Urinary trypsinogen
activation peptide can accurately predict severity of acute pancreatitis 24 hours after onset.

DO NOT DISTRIBUTE - 10 -
Study Notes – Surgery James Lamberg 28Jul2010

Treatment should include enteral nutrition via feeding tube and possibly starting antibiotics (e.g. imipenem),
especially if necrosis is present. ERCP may be used for patients with progressive jaundice or acute cholangitis. If
pseudocysts are presents and do not resolve, drainage is indicated and should be examined for amylase, CA19-9, and
CEA. In gallstone pancreatitis, the gallbladder is often removed during pancreatic surgery.
CFTR gene mutations are associated with chronic pancreatitis.
ERCP remains the gold standard for diagnosis and staging of chronic pancreatitis. Magnetic resonance
cholangiopancreatography (MRCP) with or without secretin stimulation is a useful alternative to ERCP.
Some feel the gold standard test for pancreatic exocrine insufficiency is stimulation by secretin-cholecystokinin or a
Lundh test meal.
Alcohol-induced pancreatitis may diminish over time. Patients with chronic pain may seek surgical resection,
although there is no guarantee of pain relief, especially in those who are narcotic-dependent.
Narcotic-dependent patients will likely benefit from management in a dedicated pain clinic.
Chronic pancreatitis is a risk factor for pancreatic neoplasms (pancreatic intraepithelial neoplasms, PanIN).
Pancreatic cancer is an almost 100% fatal cancer. CA-19-9 tumor marker may be elevated.
--------------------------------------------------------------------------------------------------------------------------------------------
Kaplan Videos (2001) – Surgery with Carlos Pestana, MD, PhD
--------------------------------------------------------------------------------------------------------------------------------------------
Questions are usually high yield (most common) or high impact (misdiagnosis could seriously harm patient).
--------------------------------------------------------------------------------------------------------------------------------------------
Trauma: ABCs (Airway, Breathing, Circulation)
* Patient involved in a car accident is fully conscious and voice is normal. This implies the patient has a clear
airway. If the vignette talks about shortness of breath, it is a breathing problem so airway is still intact.
* Patient with multiple stab wounds is conscious with normal voice, has several stab wounds has an expanding
hematoma in neck or subcutaneous emphysema. This patient needs an airway, such as intubation. This includes
anesthetic with pulse oximetry as well as oral- or nasal-tracheal intubation. Subcutaneous emphysema implies
tracheal-bronchial injury, so fiber optic bronchoscope intubation is needed.
* Severe car accident, multiple injuries, unconscious, spontaneous but gurgling/noisy breathing. Being unconscious
alone is an indication for intubation in trauma.
* Unconscious, spontaneous with noisy/labored breathing. Neck pain at site, couldn’t move lower extremities, lost
consciousness during ambulance ride. Attempted intubation during ride was failed. The neck cannot be moved due
to possible cervical spine injury. Answer is always airway first (ABCs). The best answer is likely nasal tracheal
intubation over a fiber optic scope.
* MVC, awake, facial fractures, bleeding into airway, voice masked by gurgling sounds. Face (nose or mouth) is
smashed, so not oral tracheal intubation. Answer is also not an emergency tracheostomy, as this is for the operating
room or ICU where patient already has airway. So correct answer is cricothyroidotomy.
* High speed MVC, smashed face, unconscious, head injury. Jet catheter in the trachea will likely not be enough.
So, pick cricothyroidotomy.
* Unconscious trauma patient is rapidly intubated in the ED. Spontaneous breathing, bilateral breath sounds, SpO2 >
95%. This patient doesn’t need breathing help. Most breathing problems relate to chest trauma.
* 22yo gang member arrives in ED with multiple gunshot wounds to abdomen, diaphoretic, pale, anxious, wants
blanket and drink of water. BP 60/40, pulse 150 barely present. So, patient is in shock (BP below 90, high pulse).
Three conditions are responsible for shock in a trauma patient. Most common is bleeding (hypovolemic,
hemorrhagic), also pericardial tamponade or tension pneumothorax. Chest has to be involved for the last two, chest
trauma is implied with blunt trauma like a car accident. Tension pneumothorax or pericardial tamponade would have
distended neck veins (JVD), while bleeding shock has flat/empty neck veins. Tension pneumothorax interferes with
both breathing and circulation, so gasping for breath and flaring nostrils. Other signs of tension pneumothorax are
decrease breath sounds on one side, tympany on chest percussion on one side, and shifted mediastinum. Treatment
of hypovolemic shock involves resuscitation phase of Lactate Ringer (LR) solution, however the current philosophy
first step is to stop the bleeding. So, stop bleeding first if possible then fill the vascular tree with fluid. So, in the
patient who was shot in the abdomen, they need an emergency laparotomy first before fluid resuscitation. The
correct answer is not give fluid to attain a certain blood pressure or give blood to reach a hemodynamic stability
with hemoglobin and hematocrit at a certain level.
* Bank robbery, innocent bystander shot in abdomen. Fully staffed trauma center is two miles from location. Patient
is in shock. What should the EMTs on scene do? “Scoop and run.” Don’t waste time starting an IV or managing the
patient on scene as the clock is ticking (golden hour). So the treatment is “diesel” (drive to the hospital).

DO NOT DISTRIBUTE - 11 -
Study Notes – Surgery James Lamberg 28Jul2010

* 19yo man shot in groin after drug deal gone bad, BP 90, pulse 105, bright red spurting blood from wound. First
step is put a finger on bleeding and stop it with local pressure. Do not blindly put a clamp on (could injure other
structures) and do not use a tourniquet.
* MVC, unconscious, BP 80/60, pulse 85, pale, no JVD. Anesthesia intubates patient, other team puts in a central
line. So, we have to assume the patient is bleeding from somewhere. Since it’s blunt trauma and we don’t know
where the bleeding is, we should fill the vascular tree first. Current philosophy is to use the peripheral veins for fluid
resuscitation. Anesthesia is at the head, one team is at the neck, another at the chest, etc. So, use two large bore
(16g) catheters at antecubital fossae in arms or cut-downs of the ankles. Use 1L or 2L of balanced electrolyte
solution, such as Ringer lactate or Ringer acetate or Normal Saline (NS). Don’t use sugar, like D5W as it could
create an osmotic diuresis and invalidate the fluid output measurement. Add blood as soon as it is available. Most
important management parameters are urinary output and venous pressure.
* 4yo child shot in arm during drive-by shooting. Site of bleeding is controlled with local pressure, child is
hypotensive and tachycardia, unable to start IVs. Next step is intraosseous (IO) cannulation at proximal tibia. Bolus
is 20ml/kg such as with NS.
* During a wilderness trek, man is attacked by bear, injuries to arms and leg, buddy stops bleeding but when EMTs
arrive an hour later he is in shock. Transportation to hospital will take two hours. EMTs, in this situation, should
spend time on site to start IVs and give fluid.
* 22yo gang member arrives in ED with multiple gunshot wounds to chest and abdomen, is diaphoretic, pale, cold,
anxious, wants blanket and water. BP 60/40, pulse rate is 120. Since it’s chest, might think about pericardial
tamponade or tension pneumothorax. If only given this information, have to assume bleeding (hypovolemia). Else
the question would have to say JVD or high central venous pressure. The question may ask what else you need to
make a decision, which would be neck vein status or ventral venous pressure.
* Multiple gunshot wounds to chest and abdomen, anxious, pale, BP 60/40, pulse is 150, diaphoretic. Large
distended veins in neck and forehead. Breathing OK and bilateral breath sounds. Answer is pericardial tamponade.
How do you manage this patient? Patient is dying, so don’t waste time with a chest x-ray. Don’t ask for blood gases.
You have the diagnosis. So, do something that empties the pericardial sac, such as pericardial window, pericardial
tube, pericardiocentesis, or median sternotomy. In the meantime, you can help the patient by giving them additional
fluids. Adding intravascular volume seems counterintuitive, but it helps while waiting to empty the pericardial sac.
* Domestic dispute, young woman stabbed in chest with 6” long butcher knife. Entry wound at 4th intercostal space,
left of the sternal border, BP 80/50, pulse 110, cold, diaphoretic, JVD present, bilateral sounds present. So,
pericardial tamponade. Here, there is only one injury and it requires repair so the answer would be median
sternotomy because it decompresses the sac and provides a means to repair the heart.
* 22yo gang member arrives in ED with multiple gunshot wounds to chest and abdomen. Has labored breathing,
cyanotic, diaphoretic, cold, BP 60/40, pulse 150 weak, large JVD, trachea deviated to left, right-sided of chest is
hyperresonant to percussion with no breath sounds. So, tension pneumothorax. Clinical diagnosis exists and patient
is dying, so don’t order chest x-ray, CT scan, or blood gases. Immediately decompress the pleural space, needle
decompression via 2nd intercostal space midclavicular then use a chest tube after. Eventually an exploratory
laparotomy as this guy was shot in the chest and abdomen.
* 22yo man involved in high-speed head-on automobile collision. Arrive in ED in coma with fixed dilated pupils,
fractures in both arms and right lower leg, BP 60/40, pulse 150. Implication of coma and pupils is closed head
injury. Where is the bleeding causing the shock coming from? Answer is not acute subdural, acute subarachnoid, or
any other head problem. Not enough room in head to bleed sufficient volume to go into hypovolemic shock. Need to
lose 1.5L of blood for shock.
* Types of shock are hypovolemic, cardiogenic, and vasomotor (loss of peripheral vascular tone).
* 72yo man lives alone, calls 911 for severe chest pain, cannot give coherent history. Arrives at ED, cold,
diaphoretic, BP 80/65, pulse 130, JVD present, short of breath. Non-trauma, so most likely a massive myocardial
infarction with cardiogenic shock (e.g. JVD present). This case is intrinsic cardiogenic shock so we treat the patient
as a myocardial infarction, not like hypovolemic shock with fluid.
* Three cases: 17yo girl stung by swarm of bees, warm and flush, BP 75/20, pulse 150. CVP is low. 22 minutes after
penicillin injection, man gets wheezes, BP 70/20, pulse 150, low CVP. Patient undergoing hernia repair has spinal
anesthesia placed, anesthesia is higher than expected, BP 70/20, patient is warm and flush. All are vasomotor shock
since patient is warm and flush rather than pale and cold. Treat patient with vasocontrictors to restore tone. Volume
replacement would not hurt these patients, but vasocontrictors are needed such as antihistamines (pseudophedrine)
or alpha-agonists like dopamine (beta1, alpha1), norepinephrine (alpha1, beta1), and phenylephrine (alpha1).
--------------------------------------------------------------------------------------------------------------------------------------------
Trauma: Head Injuries

DO NOT DISTRIBUTE - 12 -
Study Notes – Surgery James Lamberg 28Jul2010

* 22yo patient arrives to ED with axe implanted in head and based on size you can assume an intracranial wound has
been sustained. Answer is do not remove an impaled foreign body until the patient is in the OR, as the object may be
tamponading an injured vessel thus preventing bleeding.
* During a mugging, a man is hit over the head with a blunt instrument. Scalp laceration present and skull x-ray
shower underlying linear skull fracture. No loss of consciousness and can remember events (grade I concussion). In
this case, the wound can be cleaned and sutured in the ED. Loss of consciousness is grade III concussion.
* During a mugging, a man is hit over the head, scalp laceration, underlying comminuted depressed skull fracture.
This patient needs to go to the operating room, not managed in the ED.
* Man hit by car, some laceration, normal neurologic examination. Crew reports loss of consciousness (LOC) at site
but patient awakens in ambulance and is lucid. Every head trauma patient with LOC gets a CT scan. Negative CT
scan, the patient can go home and the family should wake the patient up frequently over 24 hours.
* Three cases: Pedestrian hit by car, LOC, bruising around both eyes (raccoon eyes). Pedestrian hit by car, LOC,
clear fluid from the nose or clear fluid from the ear. Pedestrian hit by car, LOC, ecchymoses behind ear (Battle
sign). These are signs of basilar skull fracture. A nasal airway should not be used. Should get CT scan to help
identify hematomas that need to be evacuated. No specific treatment is needed, usually, for basilar skull fracture.
Basilar skull fracture implies massive trauma, so associated neck injury is possible. Must be in a cervical C-collar.
Cervical plain radiographs (PR) should be three view, AP, lateral, and open-mouth odontoid. Also, these patients
will need CT scan of neck so answer may be CT scan of head that includes the neck.
* 14yo boy hit over right side of head with baseball bat, loses consciousness for a few minutes, then continues to
play. Is found unconscious one hour later, right eye fixed and dilated, contralateral palsy. This is acute epidural
hematoma, which involves LOC, lucid phase, and LOC with dilated pupil. In acute subdural hematoma, the trauma
is usually larger and the patient is usually sicker, such as elderly patient in high-speed car accident (>35mph
combined impact). For acute epidural hematomas, 90% of them are ipsilateral to dilated pupil. Hemiparesis on
opposite side. CT scan will show biconvex lens-shaped hematoma with deviation of ventricles/structures to other
side. Emergency craniotomy is needed and prognosis is excellent (high impact disease).
* CNS damage with head trauma occurs in three ways: by initial trauma, by hematoma that develops, and by the
swelling that frequently follows head trauma. Mostly medical measures are done for brain swelling.
* 32yo man involved in head-on high-speed automobile collision, unconscious at site, regains consciousness briefly
during ambulance ride, then arrives in coma with dilated pupil and contralateral hemiparesis. Answer is acute
subdural hematoma. Management is CT scan showing a biconcave semilunar crescent shaped hematoma. Next step
is craniotomy and decompression. Prognosis is grim.
* Man involved in high-speed head-on automobile collision is in a coma. He has never had any lateralizing signs
(unilateral dilated pupil, hemiparesis), CT scan shows small crescent-shaped hematoma without deviation of the
midline structures. This is acute subdural hematoma. Management is not operative since hematoma is not pushing
structures. Management is to control swelling, so minimize ICP via hyperventilation, avoid fluid overload, mannitol,
furosemide. Don’t dehydrate to the point of hypotension of course. Monitoring ICP is helpful too.
* Head-on high-speed MVC in deep coma with bilateral fixed pupils, CT scan shows diffuse blurring of the gray-
white mass and multiple small punctate hemorrhages, no single large hematoma. This is diffuse axonal injury (DAI).
No operation is indicated, as there is no one single hematoma. Treatment centers on managing ICP.
* 77yo man becomes senile over a period of 3-4 weeks. He use to be active and managed all is financial affairs.
Now he stares at the wall, barely talks, and sits all day. His daughter recalls he fell from a horse a few weeks ago.
This is a chronic subdural hematoma. It is only seen in very old or alcoholics, as there is brain atrophy. The skull
does not shrink (“size 7 brain, size 8 skull”). So brain moves more in head and can tear off a venous sinus. This
disease could be easily missed, thinking it is Alzheimer or senility, but those do not occur over a short period.
Management is to evacuate the hematoma and the patient can be back to managing their finances in a week.
* 45yo man involved in high-speed MVC, coma, fixed dilated pupil. He has multiple injuries including fractures, BP
70/50, 130bpm, what kind of intracranial bleeding is responsible? Answer is none. Blood loss source is elsewhere.
--------------------------------------------------------------------------------------------------------------------------------------------
Trauma: Neck Injuries
* Any penetrating trauma of the neck where the patient is rapidly deteriorating, answer is go to the OR.
* All gunshot wounds to the middle part of the neck, regardless of vital signs, always go to the OR.
* If patient is spitting up or coughing up blood, go to the OR.
* 42yo man shot once with 42-caliber revolver, shot is anterior middle of neck at level of thyroid cartilage, coughing
up blood, expanding hematoma, BP responding to fluids. Answer is go to the OR.
* Young man shot in upper part of neck, trajectory is all above the level of the angle of the mandible, steady blood
flows from wound, pressure does not completely control the bleeding. This injury is high in the neck with the base

DO NOT DISTRIBUTE - 13 -
Study Notes – Surgery James Lamberg 28Jul2010

of the skull close, so it is difficult to get both proximal and distal bleeding control. The preferred management of this
type of injury is an emergency angiogram with bleeding vessel embolization.
* Gunshot wound to base of the neck, wound is above clavicle but below cricoid cartilage. The base of the neck is a
boundary area, so the surgical approach may differ depending on location. So an arteriogram, esophagogram, and
bronchoscopy are indicated first.
* Bar fight, stab once in neck in front of right sternocleidomastoid muscle, man is asymptomatic and normal vital
signs. Here we can clinically observe the patient. If it were a gunshot wound to the middle of the neck, operate.
* Unbelted man thrown from car going 30mph, hitting pole. Shows up in ED neurologically intact on backboard
with multiple lacerations on face. Posterior midline of the neck has pain on palpation. Assume a bony injury to
cervical spine. Answer is to look at cervical spine first, such as three-view x-ray (AP, lateral, odontoid), or CT scan
of head and cervical spine.
* 18yo street fighter gets stabbed in back to the right of midline. Has paralysis and loss of proprioception distal to
the injury on the right side and loss of pain sensation distal to the injury on the left side. This is a hemisection of the
spinal cord (Brown-Séquard syndrome). Only way to get a hemisection is with a clean cut, so not a patient who dove
into a shallow pool.
* Car accident, burst fracture of vertebral bodies, loss of motor function and pain/temperature sensation on both
sides distal of injury, with preservation of vibratory sense and position. Spinal cord is behind vertebral bodies, so
burst of the bodies affect the anterior part of the cord. Vibratory and position sense are in the posterior portion of the
spinal cord. This injury is anterior cord syndrome. It can also be seen in anterior vascular injury.
* Rear-end collision with hyperextension of neck, has paralysis and burning pain of upper extremities. Has
preservation of motor function in legs. This is central cord, as the ligamentum flavum buckles into the spinal cord
bundle, affecting the middle fibers.
* Best method for examining the spinal cord and degree of damage is MRI. There is some controversy about giving
high dose steroids in spinal cord injury, so it may be an answer choice before orthopedic surgeons get involved.
--------------------------------------------------------------------------------------------------------------------------------------------
Trauma: Chest Injuries
* 75yo man slips and falls at home, hitting chest on counter. Exquisite pain at 7th rib midaxillary line. Normally rib
fractures and not a big deal, but in a 75yo it may be a problem. Patients don’t breath due to pain, they get atelectasis,
then pneumonia, then can die. Management is to take away the pain in a way so the patient can breath. Answer is not
to bind the chest, which alleviates pain but prevents breathing. Don’t pick the answer with a large dose of narcotic.
Answer is a local anesthetic or nerve block that alleviates pain and allows for breathing.
* 25yo man stabbed in right chest, moderately short of breath, stable vital signs, no sounds on one side, tympanic on
other side. This is a pneumothorax, without tension. First step of management is chest x-ray. If it were a tension
pneumothorax, you wouldn’t waste time. Here you don’t rush for the needle decompression and chest tube. First
step is x-ray, then if chest tube is needed you go high in the chest (2nd intercostal space).
* 25yo man stabbed in right chest, moderately short of breath, stable vital signs, no sounds at base on right, feint
sounds at apex, dullness to percussion at base. This is suggestive of hemothorax. Next step is a chest x-ray to
confirm hemothorax. Lung in not collapsed, bleeding is likely from low-pressure source like the atrium of the heart.
Bottom line is the bleeding usually stops by itself. Contaminated blood in the pleural cavity is an invitation for
infection and empyema. Management is to evacuate the blood in the pleural space with chest tube at base.
* 25yo man stabbed in right chest, moderately short of breath, stable vital signs, no sounds at base on right, feint
sounds at apex, dull to percussion at base, x-ray taken. Chest tube placed at right pleural space recovers 120cc of
blood and another 10cc the next hour. This confirms blood is coming from lung as it is a small amount. Bleeding is
stopping (<100cc/hour) so chest tube is all that was needed.
* 25yo man stabbed in right chest, moderately short of breath, BP 95/70, pulse 100, no breath sounds over right
chest and is dull to percussion, chest tube at right base recovers 1200cc of blood. This patient is bleeding from
systemic vessel, like an intercostal. The answer here is thoracotomy for control of bleeding.
* 25yo man stabbed in right chest, moderately short of breath, stable vital signs, no sounds at base of right chest,
dull to percussion at base, chest tube at right base recovers 350cc of blood. Over the next four hours, he continues to
drain 200-300cc per hour. So this is not from the lung and is from systemic vessel. Needs thoracotomy.
* 25yo man stabbed in right chest, moderately short of breath, stable vital signs, no sounds at base of right chest,
hyperresonant to percussion at apex, air fluid level seen on chest x-ray. We need a chest tube. Either a two tubes
(one for base, one for apex) or a tube with a hole at the top and hole at the bottom to drain both the blood and air.
* If multiple air-fluid levels seen in chest, usually implies bowel is up in chest.
* 33yo woman involved in high-speed MVC, arrives at ED gasping for breath, cyanotic lips, flaring nostrils,
bruising over both sides of chest, tenderness suggestive of multiple rib fractures. BP 60/45, pulse 160, JVD,

DO NOT DISTRIBUTE - 14 -
Study Notes – Surgery James Lamberg 28Jul2010

diaphoretic, left hemithorax has no breath sounds and dull to percussion. This is tension pneumothorax. Penetrating
injury is from broken ribs. Patient needs needle decompression.
* If myocardial contusion is suspected, do blood gases, ECG, and cardiac enzymes.
* Search for traumatic aortic transection, especially if severe deceleration is the mechanism. Look at chest x-ray for
wide mediastinum, which gives a high index of suspicion. Next step of management is spiral CT scan of chest. If
spiral CT is negative, don’t stop. Do an arteriogram. If there is no widened mediastinum on chest x-ray, do the spiral
CT scan but stop there if that is negative.
* Worker injured during explosion at factory, multiple cuts due to flying debris, obviously short of breath, has large
flap-like 5cm injury to chest wall, sucks air through it with every inspiratory effort. If left alone, this patient will
develop a tension pneumothorax. This is a sucking chest wound. An occlusive dressing is needed, such as a large
white petroleum (Vaseline) gauze. One detail is the dressing is taped on three of four sides (semi-occlusive),
allowing for some air to leave during expiration.
* 55yo woman crashes car into telephone pole at high speed. She arrives at ED with respiratory distress, multiple
chest bruises, and multiple chest fractures. On the left side, part of the chest wall caves in with inspiration and
bulges out with expiration. This is paradoxical breathing with flail chest. Even though the chest wall is moving in an
abnormal way, it is not affecting breathing in general. So the management is treatment of the underlying contusion.
Answer is fluid restriction, possibly diuretics, if fluids needed use colloid (not crystalloid). More important is
treatment for ensuing pulmonary failure, which is monitoring blood gases. If PO2 goes down and PCO2 goes up,
patient may need intubation and a respiratory ventilator. However, small punctures sites that are not a problem when
the patient is breathing on their own may become a problem with positive pressure ventilation. So, they may need a
chest tube or tubes on both sides. Also monitor ECG and cardiac enzymes.
* 55yo woman crashes car into telephone pole at high speed. Arrives at ED breathing well, multiple bruises on
chest, x-ray shows multiple rib fractures, lung parenchyma is clear, lungs are expanded. Two days later, her lungs
white-out on x-ray and is in respiratory distress. These are signs of pulmonary contusion. So we need blood gases,
fluid restriction, and diuretics.
* 54yo woman crashes car into telephone pole at high speed. Arrives at ED breathing well, multiple bruises on
chest, exquisitely tender of sternum to the point she feels bone grinding on bone. Sternum fracture is likely, so
monitor ECG and cardiac enzymes as it may be cardiac contusion. Look for traumatic aortic transection.
* Only survivor of four people in car implies very major crash. Three bones in chest that are very difficult to break
are the sternum, first rib, and scapula. Fractures of these means big time trauma.
* 53yo man involved in high-speed MVC. Moderate respiratory distress, no breath sounds on right side, percussion
unremarkable. Multiple air-fluid levels seen on x-ray. This is traumatic rupture of the diaphragm, almost always on
the left side. Air-fluid levels are bowel. This needs surgical correction, typically done through the abdomen. To
make it more obvious, it may say a nasogastric tube is placed and tip curls up into chest.
* Motorcycle daredevil attempts to jump over the 12 fountains in front of Cesar palace in Las Vegas. As he leaves
the ramp at high speed, his motorcycle turns sideways and he hits the retaining wall on the other end like a ragdoll.
At the ED he is found to be remarkably stable although he has multiple fractures. Left first rib fracture is seen with
widened mediastinum. So, we likely have traumatic aortic transection. Begin with a spiral CT, if positive then begin
with repair. If negative, go for arteriogram.
* 34yo woman suffers severe blunt trauma in car accident, multiple extremity injuries, head trauma, and left
pneumothorax. Shortly after exam she develops progressive subcutaneous emphysema over upper chest and neck.
Causes of subcutaneous emphysema are perforation of esophagus (e.g. post-endoscopy), tension pneumothorax, and
major tracheal-bronchial injury. So, this patient needs fiber optic bronchoscopy for diagnostic confirmation and to
guide the insertion of an airway.
* A patient who received a chest tube for traumatic pneumothorax puts out lots of air through tube (large air leak).
The collapsed lung is not expanding. This indicates a major bronchial injury. Indicates fiber optic bronchoscopy.
* Patient sustains penetrating chest injury, intubated, placed on respirator, chest tube placed in appropriate pleural
cavity. Patient was hemodynamically stable, then suddenly goes into cardiac arrest. This is air embolism. Injury to
major bronchus and major vessel next to each other. Patient is fine breathing alone, but once on ventilator the air is
forced into lung and leaks out into the major vessel nearby.
* During the performance of a supraclavicular node biopsy under local anesthesia, a hissing sound is heard and the
patient drops dead. This happens during inhalation with negative pressure in chest. 120cc is enough air.
* A patient is receiving total parenteral nutrition (TPN) through a central venous line, becomes frustrated because
the nurses are not answering his call button, so he gets up out of bed, disconnects the central line, takes two steps
toward the door while the catheter is dangling, and drops dead. The treatment is to put patient into trendelenburg

DO NOT DISTRIBUTE - 15 -
Study Notes – Surgery James Lamberg 28Jul2010

position with the left side down (right side up), hoping the air stays in the atrium rather than the ventricle. If tube is
present in central line, suck air out, maybe do cardiac pressure/massage to move air to atrium as well.
* Patient sustains severe blunt trauma including multiple fractures of long bones, become disoriented, develops
petechia rashes in neck and axilla, has tachycardia. Few hours later is in respiratory distress with hypoxemia. Chest
x-ray shows patchy infiltrate with a low PO2. This is a fat embolism. Multiple long bone fractures followed by
respiratory distress a few hours later implies fat embolism. Fat droplets may be in fundus of eye. Management is
respiratory support, not IV heparin. So, monitor blood gases and possibly a ventilator.
--------------------------------------------------------------------------------------------------------------------------------------------
Trauma: Abdominal Injuries
* Every gunshot wound to the abdomen gets an exploratory laparotomy. A stab wound where it is clear the
abdominal cavity has been penetrated (e.g. bowel hanging out), must go to OR. Also, any abdominal trauma where
the patient develops acute abdomen (e.g. pain, guarding, rebound tenderness) they get exploratory laparotomy.
* 21yo gang member shot in abdomen, epigastrium, left of midline. Patient is stable. Answer is exploratory
laparotomy. Preparation includes large bore venous lines, indwelling venous catheter, high dose antibiotics.
* Exploratory laparotomy for patient with gunshot wound shows clean punched-out entrance and exit wounds in
transverse colon. Answer is to repair injuries caused by bullet. Answer is not to remove the bullet. If bullet is lodged
in peritoneal muscles, no time is spent dissecting and removing the bullet. Answer is not diverting colostomy,
especially in civilian population (vs. military).
* 19yo gang member shot once with 38-caliber revolver, left of midclavicular line, 2” below nipple, bullet lodged in
paraspinal muscles. Every gunshot wound below nipple line includes abdomen. Patient may need chest tube but
answer is exploratory laparotomy.
* 24yo man stabbed in belly, lateral to umbilicus to the left, omentum is protruding. Do exploratory laparotomy.
* During a domestic fight, a 38yo obese woman is attacked with a 4” long switchblade. In addition to several
superficial lacerations, she was stabbed in the abdomen, is hemodynamically stable, has no peritoneal signs. Here
there is no evidence of penetration. Do a gloved finger introduction into the wound, gently, to determine if the
wound has penetrated. So, use digital exploration.
* 31yo woman smashes car into wall, has multiple injuries, upper/lower extremity fractures, BP 75/55, pulse 110,
CVP is zero. So, she’s bleeding somewhere. Tender abdomen is found with guarding and rebound. This indicates an
exploratory laparotomy.
* 31yo woman smashes car into wall, has multiple injuries, upper/lower extremity fractures, BP 135/75, pulse 82.
Physical exam shows tender abdomen, guarding, rebound. This also indicates exploratory laparotomy.
* 26yo woman involved in car wreck, upper/lower extremity fractures, facial lacerations. Chest x-ray normal. She
develops hypotension, tachycardia, low hematocrit, and low CVP. So she’s bleeding somewhere. We know it’s not
in the head (not 1.5L space). Neck bleeding would be seen. Pericardial sac would show pericardial tamponade with
high CVP, so not that. Pleural cavity would be seen on chest x-ray. Arms are too small to accommodate that much
blood. Last three places are abdomen, pelvic, or femur fracture. Pelvic fractures or femur fractures are always
looked for during rapid head to toe trauma assessment. Last place left is abdomen. However, surgery would not be
performed without verification. So the next step in management is Focused Assessment with Sonogram for Trauma
(FAST). Previously was diagnostic peritoneal lavage (DPL), but had shortcomings including being invasive and
only gave a yes/no blood/no blood answer. CT scan of abdomen is excellent way to detect blood, however it takes
too much time while the patient is bleeding out. So, patient must be hemodynamically stable for CT scan. The best
option here is FAST (sonographic) exam.
* 27yo intoxicated man smashes his car into tree, tender over left chest wall, x-ray shows 8-10 rib fractures, BP
85/68, pulse 128. Ruptured spleen is most common solid organ injured in blunt trauma. Causes major bleeding.
* 27yo intoxicated man smashes his car into tree, tender over left chest wall, x-ray shows 8-10 rib fractures, BP
85/68, pulse 128 that do not respond to fluid administration. Positive peritoneal lavage and ruptured spleen is found
on exploratory laparotomy. Management is attempted repair of spleen, not immediate splenectomy. Spleen is
important for immunological function, particularly in children. If unable to be repaired (shattered spleen or
salvageable spleen with other major time-important injuries), remove it. The patient will need to be immunized
against encapsulated bacteria (pneumovax, hemophilus influenzae, meningococcus) that could cause sepsis.
* Multiple trauma patient receiving massive blood transfusions as surgeons are attempting to repair abdominal
injuries. It is noted that blood starts oozing from all surfaces and IV site (developing coagulopathy), core temp is
normal. This is predictable with >10-12 units of blood. Studies could be done to determine cause of coagulopathy,
but there is no time. Answer is give fresh frozen plasma (FFP) and platelets.

DO NOT DISTRIBUTE - 16 -
Study Notes – Surgery James Lamberg 28Jul2010

* Multiple trauma patient gets laparotomy, develops coagulopathy, has low core temp, and acidosis. Answer is stop
operation immediately. Packing should be done for all bleeding areas and abdomen sealed in temporary fashion. The
patient should then be re-warmed, treat coagulopathy (FFP, platelets), then continue operation.
* Exploratory laparotomy for multiple abdominal injuries has lasted 3.5 hours and many transfusions have been
given as well as many lactated Ringer solutions. When closing, there is too much tension to close the abdomen. This
is abdominal compartment syndrome. Forced closure prevents breathing and compresses kidneys. Answer is close
abdomen in temporary method, such as with mesh or plastic stapled around abdominal opening. Close later.
* First post-op day, trauma patient develops very tense and tender abdomen. Tension sutures are cutting through
abdominal wall and is developing hypoxia. Answer is go back to operating room, open abdomen, temporary closure.
--------------------------------------------------------------------------------------------------------------------------------------------
Trauma: Pelvic Fractures
* Rollover car accident, 42yo woman thrown from car, car rolls over her. Pelvic fracture is found in ED. She is
hypotensive and responds to fluid, CT shows no intra-abdominal bleeding but a pelvic hematoma is present. Leave
pelvic hematoma that is not expanding alone. Do not operate on it or stick a needle in it. Answer is to do a
proctoscopic/rectal exam, female pelvic exam, inject dye into male urethra to rule out injury.
* Rollover car accident, 42yo woman thrown from car, car rolls over her. Pelvic fracture is found in ED. She is
hypotensive and did not respond to fluid resuscitation, sonogram shows no intra-abdominal bleed, hemodynamics
keep dropping. Here there is no good answer, but operation is not preferred. Arteriogram may work but bleeding is
usually from veins. So, answer is do external fixation of the pelvis. This prevents fragments from moving.
--------------------------------------------------------------------------------------------------------------------------------------------
Trauma: Genitourinary Injuries
* Blunt trauma, blood in urine, rib fractures: think kidney. Blunt trauma, blood in urine, pelvic fractures: think
bladder or male urethra.
* Young male is shot point-blank in lower abdomen just above the pubis, blood in urine, no evidence of rectal
injury. Answer is exploratory laparotomy for bladder.
* Woman shot in flank, foley inserted and urine bloody. So, kidney is source. Go to the OR, exploratory laparotomy.
* 22yo male involved in high-speed automobile collision with multiple injuries including pelvic fracture, blood
found at meatus and blood in urine. Answer is retrograde urethrogram, do not insert a Foley catheter.
* 19yo male involved in severe automobile accident, has pelvic fracture, has sensation to need to urinate, scrotal
hematoma, blood at meatus, rectal exam shows high-riding prostate. Answer is posterior urethral injury. Study to do
is retrograde urethrogram.
* 19yo man involved in motorcycle accident, scrotal hematoma, blood at meatus, urethralgram shows injury.
Urologists prefer to defer the repair of a posterior urethral injury but to immediate repair of anterior injury.
* 42yo male involved in car crash, pelvic fracture, no blood at meatus, Foley attempted but resistance felt. Answer is
do not continue to insert, remove the catheter. Do retrograde urethrogram.
* 22yo male involved in high-speed car crash, pelvic fracture, initial physical exam shows no blood at meatus, Foley
is easily inserted, gross hematuria found. Answer is bladder injury. Do retrograde cystogram while bladder full with
dye and while emptying the bladder. Most common injury location is at base near trigone. Dye would then be
retroperitoneal and covered by dye in bladder.
* High-speed MVC, abdominal injuries, rib fractures, contusions, no pelvic fracture, gross hematuria, retrograde
cystogram is normal. Answer is CT scan. Most kidney injuries are non-surgical management.
* Patient involved in high-speed MVC, multiple injuries, rib fractures, abdominal contusions, hematuria, normal
retrograde cystogram, CT scan shows renal injuries that do not require surgery. Six weeks later the patient develops
acute shortness of breath and flank bruit. Answer is arteriovenous (AV) fistula. Do arteriogram and repair it. If the
patient became hypertensive, it would be renal artery stenosis.
* 35yo male is about to be discharged from the hospital after being observed for multiple trauma. He is found to
have microscopic hematuria. In an adult, this is inconsequential. So, no management needed.
* Boy falls of bicycle and is found to have microscopic hematuria. This may be the first sign of congenital anomaly
of the urinary tract. Answer is perform a urological evaluation.
* 14yo boy slides down banister and smashes scrotum into knob at the end. Shows up in the ED with grapefruit-
sized scrotum, normal urination. Could also be guy kicked in groin during street fight. The question is about
testicular fracture. Answer is perform a sonogram.
* 41yo male reports to ED reporting he slipped in the shower and injured his penis. Exam shows large penile shaft
hematoma with normal glands. This is a fracture of the tunica albuginea, corpora cavernosum. This typically
happens during intercourse, acrobatic acts, woman on top. So recognize this was a cover story and likely occurred
while the penis was erect. This is an urologic emergency and the penis needs immediate repair.

DO NOT DISTRIBUTE - 17 -
Study Notes – Surgery James Lamberg 28Jul2010

--------------------------------------------------------------------------------------------------------------------------------------------
Trauma: Extremity Injuries
* 25yo man shot with 22-caliber gun in anterolateral aspect of thigh. X-ray shows bullet embedded posterolateral to
femur. We know from anatomy that the major vascular structures are medial, so this man does not need exploratory
surgery or angiogram. He needs tetanus shot and wound cleaning, bullet can stay unless near vital structure.
* 25yo man shot in anteromedial aspect of the upper thigh. Exit wound in posterolateral aspect of thigh. Normal
pulses, no hematoma, x-ray shows bone intact. However, anatomy dictates we are close to the femoral artery and
vein. Answer is arteriogram and possibly exploratory laparotomy. Don’t assume pulses mean no injury.
* 25yo man shot in anteromedial aspect of upper thigh, exit wound in posterolateral aspect of thigh, large expanding
hematoma in upper inner thigh. Answer is exploratory laparotomy to obtain proximal and distal control.
* Man shot through arm with 38-caliber revolver, path through extremity from medial to lateral side, large
hematoma on inner aspect of arm, no distal pulses, radial nerve palsy, fractured humerus. What should we manage
first? The vascular injury is most important from a biological viewpoint. Practically, this is a very delicate surgery.
Gross motion is seen in orthopedic surgery. So, by necessity, answer is fix the bone first. Nerve is last.
* Forearm and lower leg most likely site of compartment syndrome, which can be caused by prolonged ischemia
followed by reperfusion.
* Hunting accident, a man is shot in leg with high-powered big-game hunting rifle (rhino, elephant, not deer). 1cm
hole in upper outer thigh, exit wound is 8cm posteriolateral thigh, femur is shattered. Injury caused by bullet is
directly proportional to mass of bullet and the square of the velocity. Less than 1500ft/sec are considered low
velocity (most civilian weapons), injury limited to back of bullet. High velocity injuries create a cone of destruction
as they pass through. Answer is wide debridement and likely amputation.
* 6yo girl has hand and forearm caught in old-fashioned washing machine, bruise seen, hand not working. This is a
crush injury, like bus running over leg. First concern is myoglobinemia issue, as crushed tissue sends myoglobin
into blood and can cause renal failure. Answer for this is give lots fluid, alkalization of urine, monitor serum
potassium. Second concern is swelling and compartment syndrome. Answer for that is fasciotomy to relieve
pressure.
--------------------------------------------------------------------------------------------------------------------------------------------
Trauma: Burn Injuries
* For chemical burns, as long as the agent is still there the burn will continue. So answer is massive irrigation.
* Common acid is battery acid and alkaline is drain cleaner. Resist the temptation to play chemist.
* 7yo girl spilled Draino all over arms and legs, girl is screaming in pain. Answer is not rush girl to Emergency
Department. Answer is to take off clothes and put girl in shower for 30-minutes then bring to the ED. Answer is not
washing with vinegar or some other acidic substance.
* Exception: Depressed teenage girl attempts to commit suicide by drinking Liquid Plummer. She changes her mind
and is franticly seeking help. By necessity, we give diluted vinegar, lemon juice, or orange juice. If acid were
responsible, we give milk, antacids, or egg whites. Something gentle.
* Man comes into contact with high-tension electrical power line. Entrance burn wound on upper outer thigh, exit
wound lower. Assume everything in between is cooked to a crisp. Answer is extensive surgical debridement and
possible amputation. Watch for myoglobinemia and myoglobinuria, give osmotic diuretics and lots of fluid.
Posterior dislocation of the shoulder and compression of vertebral bodies may be seen due to intense contraction of
all muscles in the body. Demyelination may occur down the road.
* Man rescued from burning building, burns around mouth and nose, inside of mouth looks like inside of chimney.
If smoke involved and confined environment, think respiratory burn. Answer to confirm diagnosis is fiber optic
bronchoscopy. Answer for handling the patient is by blood gases. Only effective therapy is respiratory support.
Patient may have breathed carbon monoxide, so also monitor for carboxyhemoglobin.
* Patient has 3rd degree burns to both arms while shirt caught on fire upon lighting backyard fire pit. Burn areas are
dry, white, leathery, anesthetic, and circumferential around forearm. The worry is not airway burn as we are in an
open space. The key word here is circumferential, edema occurs and there is nowhere to expand. Answer is to
monitor circulation, monitor pulses, monitor capillary refill, Doppler study. If no pulse/flow, must perform an
escharotomy, dramatic procedure but don’t need anesthetic (no nerves) or cleaning (already contaminated).
* Toddler is brought to ED with burns on both buttocks, area is moist blisters present (2nd degree), story is child
pulled pot of burning water from stove. Important answer is not about silver sulfadiazine for the treatment. The
answer is recognizing this is child abuse. Answer is referring to proper authorities.
--------------------------------------------------------------------------------------------------------------------------------------------
Trauma: Burns & Fluid Resuscitation

DO NOT DISTRIBUTE - 18 -
Study Notes – Surgery James Lamberg 28Jul2010

* Adult male sustains 2nd and 3rd degree burns (drawing shows areas). Fluid is no longer is vascular space, it has
moved into tissues, patient requires fluid for about two days after burn. Estimate fluid requirements first with
formula, then assess patient via urinary output and give fluids accordingly. Modified Parkland formula uses body
weight, extent of burn, then a factor (which is different between formulas).
* Rule of Nines for adult: Head and each arm = 9%, Anterior, posterior, and each leg = 18%. Genitals = 1%.
* Rule of Nines for children: Each arm = 9%, Head, anterior, posterior = 18%, Both legs = 27%. Genitals = 1%.
* Modified Parkland = Body Weight (kg) * % Body Surface Burned (up to 50%) * 4cc (some formulas use 2cc).
* Burns > 50% body surface means fluid loss is at maximum possible rate.
* 70kg male burned over 18% would be: 70kg * 18 * 4cc = 5040cc = 5L fluid
* Half of fluid given over first 8 hours, next half over next 16 hours. 2L may be added since patients are NPO.
* Second day, needs are about half calculated. On day three, no more fluid needed and diuresis should be seen.
* An adult who weights Xkg has third degree burns over (drawings, over 20% BSA), fluid administration should be
started at what rate? Answer is being at about 1000cc/hour. This is a good rate to start at, then monitor urine.
* An adult who weights Xkg has third degree burns over (drawings), how much is the estimated amount of fluid
needed? Answer is to use the Modified Parkland formula.
* After suitable calculations have been made, a 70kg adult with extensive third degree burns is receiving Ringer
lactate at the calculated rate. Over the first three hours, his urinary output is 15cc, 22cc, and 18cc. Normal urinary
output should be 1cc/kg/hr, so you’d expect 70cc/hour. Range is about half that (35cc) to twice that (140cc). Answer
is give more fluid, increase rate of fluid administration.
* After suitable calculations have been made, a 70kg adult with extensive third degree burns is receiving Ringer
lactate at the calculated rate. Over the first three hours, his urinary output is 325cc, 240cc, and 370cc. Answer is to
give less fluid, decrease rate of fluid administration.
* During the first 48-hours after a major burn, a 70kg patient received vigorous fluid resuscitation, maintained
urinary output between 45 and 110cc/hour. On day three, after fluid resuscitation has stopped, his output reaches
270 to 350cc/hour. Answer is not high output renal failure or early fluid mismanagement. Answer is this is to be
expected for the patient, do nothing.
* Eight-month old baby weighing Xkg is burned over (drawings), formula includes more fluid (e.g. 6cc factor).
* Patient with 2nd and 3rd degree burns over 65% of BSA is undergoing proper fluid resuscitation. Must give
tetanus prophylaxis for any burn where patient doesn’t have current tetanus; in fact, tetanus for anything that breaks
the skin surface. Burn area should be cleaned. No parenteral antibiotic that could kill all the bugs that could enter
through the burned area. So topical management is needed and the workhorse is silver sulfadiazine. With deep burns
or if cartilage is involved, use mafenide acetate. Mafenide acetate, unlike silver sulfadiazine, is painful to apply and
can cause acidosis. Cuts near the eyes should be covered with triple antibiotic ointment, this is because silver
sulfadiazine is very painful if gotten in the eyes. Pain medication should be given intravenously, not subcutaneous or
intramuscular. After 2-3 weeks, grafts should be done to areas that did not regenerate. Intensive nutritional support,
preferably via GI with high calorie and high nitrogen, should be given right after burn. Rehabilitation must begin on
day one. Survival is not the only objective, not the endpoint. We need a functioning injured area. The last thing the
patient is going to want to do is move a burned area, however, if you let them go for three weeks with immobile
hands they will have useless hands for the rest of their life.
* In a very circumscribed third degree burn, the best management is immediate excision and grafting. A 42yo
woman drops a hot iron on her lap while doing the laundry. She comes in with the shape of an iron clearly
delineated on her upper thigh. The burn is white, dry, leathery, and anesthetic. Here, it wouldn’t make sense to have
this patient in the hospital for three weeks while waiting for a graft. Answer is to take patient to operating room the
same day, excise area, put on skin graft.
--------------------------------------------------------------------------------------------------------------------------------------------
Trauma: Bites & Stings
* A 6yo child tries to pet a domestic dog while the dog is eating and the child is bitten by the dog. What about
rabies? In a domestic pet, presumably vaccinated, if bite is considered provoked, then don’t worry about rabies
prophylaxis. Animal goes under the care of a veterinarian, if animal shows signs of rabies within a few days, then
we protect the child.
* During a hunting trip, a young man is bitten by a coyote. Wild animals cannot be observed as they will have
aggressive behavior while in captivity. They should be brought in anyway so the coyote can be killed and the brain
examined for rabies. If rabies, prophylax hunter.
* While exploring caves in the Texas hill country, a young man is bitten by bats, which fly away. Answer is give
rabies prophylaxis, which includes both the immunoglobin and vaccination.

DO NOT DISTRIBUTE - 19 -
Study Notes – Surgery James Lamberg 28Jul2010

*During a hunting trip, a hunter is bitten in the leg by a snake. His companion, who is an expert outdoorsman,
reports the snake had elliptical eyes, pits behind the nostrils, big fangs, and a rattler at the tale. The patient arrives to
the hospital one hour after the bite, has two fang marks about 2cms apart, no local pain, no swelling. Your first
reaction might be that the snake is venomous and the man was bitten, so treat. However, up to a third of these bites
do not involve venom. Factor that determines envenomation is burning and redness within 30 minutes. So, in this
case the patient was not injected with venom. Answer is to clean wound and send patient on their way.
* During a hunting trip, a hunter is bitten in the leg by a snake. His companion, who is an expert outdoorsman,
reports the snake had elliptical eyes, pits behind the nostrils, big fangs, and a rattler at the tale. The patient arrives to
the hospital one hour after the bite, has two fang marks about 2cms apart, with local edema, ecchymotic
discoloration, with much pain and redness in the area. Answer is not sucking out the venom or not excising the skin.
Answer is give specific anti-venom in large dosages, at least five vials. In large envenomation, give 10-20 vials.
Blood should be drawn for type and cross-matching as the venom interferes with this, so if something happened
down the road you’d be stuck. Also do coagulation studies, renal function studies, and liver function studies.
* While playing in the backyard of a south Texas home, a 6yo girl is bitten by a rattlesnake. At the ED, she has
severe signs of envenomation. Even in this case, with a child, the dosage does not deal with the patient’s size. It
deals with the amount of envenomation. So, we might still need the 10-20 vials of anti-venom.
* During a picnic out, a young girl inadvertently bumps into a beehive and is stung repeatedly by angry bees. She is
seen 20mins later, wheezing, hypotensive, and madly scratching a urticarial rash. Answer is give epinephrine.
Dosage is 1/3 to 1/2 of 1:1000 solution epinephrine subcutaneous or 1:10000 IV. Remove stingers carefully as to not
inject more venom.
* While rummaging around her attic, a woman is bitten by a large spider she says was black with a red hourglass on
the belly (black widow). She is not vomiting and has muscle cramps. The cramps can be so strong that they mimic
an acute abdomen (guarding). Answer is give IV calcium gluconate. Muscle relaxants could maybe help.
* Patient finds a painful red ulcer on his forearm, was cleaning the attic the day before, and thinks it might be a
spider bite. The ulcer has is 1cm in diameter and has a necrotic center with a surrounding halo of erythema. This is a
brown recluse bite. Answer is do local excision to remove dead skin. Eventually a skin graft may be needed.
* 22yo gang leader comes to the ED with a small 1cm deep sharp cut over the knuckle of his middle finger. He says
he cut himself with a screwdriver while fixing his car. This is a human bite, mostly from hitting someone in the
mouth with a closed fist. Bacteriologically, a human bite is the worst. If it affects the joint, it’s a big deal. Answer is
go to the OR and do massive irrigation and debridement.
--------------------------------------------------------------------------------------------------------------------------------------------
Orthopedics: Children
* In a newborn nursery it is found that a child has unequal gluteal folds. On physical exam, one hip can be easily
dislocated posteriorly with a jerk and click, then relocated with a snapping sound. The family is concerned because a
previous child had the same problem. This is developmental dysplasia of the hip (congenital dislocation of the hip).
Answer is not get an x-ray. Answer is a sonogram; hip has not been calcified enough for an x-ray. Management is
abduction splinting with Pavlik harness. A double diaper may be used possibly, both for up to 6 months.
* 6yo boy has insidious limping with decreased hip motion, pain in the knee on that side. He walks into the office
with an antalgic gait, putting less time of weight on the painful side. Passive motion is guarded. Legg–Calvé–Perthes
disease (avascular necrosis of capital femoral epiphysis). What is the diagnostic test? Answer is x-ray.
* 13yo chubby boy complains of pain in the groin and is noted by the family to be limping. He sits in the office with
the sole of the foot on the affected side facing the other foot. The hip has limited motion, as it is flexed, the leg goes
into external rotation and cannot be rotated. This is slipped capital femoral epiphysis (orthopedic emergency).
Answer for diagnosis is x-ray of hip. Management is pinning of femoral head to the hip.
* Toddler has had the flu for several days, had been walking fine for two days, now refuses to move one of the legs.
He holds it with hip flexed, in slight adduction, external rotation, and you cannot evaluate the hip. He has elevated
SED rate. Answer is septic hip. Diagnosis is general anesthesia with aspiration for pus. Management is open
orthrotomy with drainage.
* A child with a febrile illness but no trauma, has localized pain/tenderness in a localized bone. Answer is acute
hematological osteomyelitis. Answer is not an x-ray as it takes time for osteomyelitis to show. Answer is a bone
scan (radionucleide). Management is antibiotic.
* A 2yo child is brought in by concerned parents because he is bow-legged. Answer, this is normal.
* A 5yo child is brought in by concerned parents because he has knock knees. Answer, this is normal.
* 14yo boy says he injured his knee while playing football. There is no swelling of the knee joint, but he has pain
over the tibial tubercle, physical exam reveals pain at that location. As a rule, with intrinsic injury to joint (e.g.
ACL), there will be swelling. “Swelling of the knee is the poor-man’s MRI of the knee.” Answer is Osgood

DO NOT DISTRIBUTE - 20 -
Study Notes – Surgery James Lamberg 28Jul2010

Schlatter disease (osteochondrosis of the tibial tubercle). Treated with immobilization of the knee, in an extensor
position for 4-6 weeks.
* A baby boy is born with both feet turned inward. Physical exam reveals plantarflexion of ankle, inversion of the
foot, adduction of the forefoot, and internal rotation of the tibia. Answer is clubbed foot (talipes equinovarus).
Answer is serial plaster casts beginning during neonatal period. Sequence begins with most distal deformity, which
is the adducted forefoot, then varus, then equinus. If surgery needed, done after 6 months and before age 2.
* 9yo girl referred to school nurse for potential scoliosis. Thoracic spine curved right, when she bends forward a
hump is noted over right thorax, no menstruation yet. Scoliosis is complex field and has specialized orthopedic
surgeons. Earliest manifestation of scoliosis is presence of hump when bent over. Menstruation is roughly 80% of
maturity. Answer is corset, or brace, or other orthopedic device to stop the disease progression.
* A 4yo falls down stairs and fractures his humerus, he is placed in a cast at local doc-in-the-box (small non-
emergency clinic). Boy seen at pediatrician 4 days later. AP and lateral x-rays show significant angulation of the
broken bone. Answer is not ORIF, replacing cast, or other procedures. Answer is do nothing, counting on the
amazing ability for the child’s bones to remodel.
* An 8yo boy falls on his right hand with the arm extended and he breaks his elbow by hyperextension. X-rays show
supracondylar fracture of the humerus. In an adult, this isn’t a big deal. However, in children, this has a high
incidence of neurovascular compromise. Answer is to pay tremendous attention to vascular supply, like monitor
pulse, monitor capillary refill, Doppler studies, etc. Fracture itself is treated in a simple manner.
* A child sustains a fracture of the long bone involving the epiphysis and growth plate. The fracture is laterally
displaced but in once piece, fracture does not cross epiphysis or growth plate. Answer is to move bone back and
apply cast. Normally Salter-Harris class less than V.
* A child sustains a fracture of the long bone extending through the joint, epiphysis, growth plate, and part of the
metaphysis. So, growth plate in two pieces. Answer is very precise alignment (ORIF) before casting.
--------------------------------------------------------------------------------------------------------------------------------------------
Orthopedics: Tumors
* A 16yo boy complains of low-grade but constant pain in the distal femur for several months. X-ray shows large
bone tumor with sunburst pattern and periosteal onion skinning. Benign tumors typically have a sharply demarcated
edge, distinguishing it from the bone. Primary malignant tumors have a fuzzy, ill-defined, “sunburst” pattern.
Osteogenic sarcoma tends to be older kids and around the knee, Ewing sarcoma typically younger and in diaphysis
(middle) of the bone. Most likely osteogenic sarcoma here. Answer is this is a primary malignant bone tumor.
* A 10yo complains of persistent pain deep in the middle of the thigh, x-ray shows mass in middle of femur with
onion skinning and sunburst pattern. Most likely Ewing sarcoma. Answer is not put needle in bone or anything
invasive. Answer is refer to an orthopedic surgeon.
* A 66yo woman picks up a bag of groceries and her arm snaps broken (pathological fracture). Osteoporosis could
be an option, but more trauma is needed such as a fall to break an osteoporotic leg. Answer is metastatic malignant
tumor to the broken bone in the arm. X-ray will show a lytic, eating away, bone. Assume most likely source for
women is breast, for man assume the lung (not prostate). Bone metastasis for prostatic cancer are osteoblastic, so
building or a mass in the bone.
* 60yo man complains of fatigue and pain at specific places on several bones, found to be anemic, x-rays show
multiple punched-out lytic lesions. This is multiple myeloma (MM). Bence Jones proteins are found in the urine.
Abnormal immunoglobin. Answer is look at urine for proteins and immunoelectrophoresis of blood for abnormal
pattern. Answer chemotherapy for treatment.
* 58yo woman has a soft tissue tumor in her thigh, it has grown steadily for 6 months, is deep in the thigh, firm,
fixed to surrounding structures, 8cm in diameter. This is soft tissue sarcoma. Answer is do an MRI to look at soft
tissue pathology. Answer is not doing invasive things like putting a needle in it. To diagnose, you need a large piece
of tissue so you’d need an open biopsy, but it can compromise future treatment. So, MRI then refer to expert.
--------------------------------------------------------------------------------------------------------------------------------------------
Orthopedics: Upper Extremity
* A man who fell from a 2nd store window has clinical evidence of fracture in the femur. If you think a bone is
broken and you want to verify, you need x-rays at 90-degrees to each other and you need to include the joint above
and the joint below. In this case, there is a high mechanism of injury suggesting multiple trauma. Answer is not only
AP and lateral of femur with 2 joints included, but also x-ray of lumbar spine that may have compression fractures.
* While playing football a college student fractures his clavicle, pain is at middle and distal thirds of the clavicle.
Answer is treat with figure-of-eight device (pulls back shoulders) for a few weeks.
* A 55yo woman falls in the shower and hurts her right shoulder. Arm is held close to body but rotated outwards as
if she were going to shake hands. She is in pain and will not move the arm, there is numbness over the deltoid

DO NOT DISTRIBUTE - 21 -
Study Notes – Surgery James Lamberg 28Jul2010

(axillary nerve). This is an anterior dislocation of the shoulder, most common. Answer is x-ray AP and lateral then
do a closed/external reduction.
* After a grand mal seizure, a 32yo epileptic notices pain in her right shoulder and cannot move it. She goes to a
doc-in-the-box, gets x-rays, and is diagnosed with a sprain and is given pain meds. She comes the next day to the
ED with the same symptoms, arm held close to the body. This is a posterior dislocation of the shoulder, occurring
when all the muscles in the body contract at the same time (epilepsy, electricity). Answer is get x-rays (AP/lat)
including the axillary view and scapular-lateral view.
* An elderly woman with osteoporosis falls on an outstretched hand (FOOSH), comes in with a deformed and
painful wrist that looks like a dinner fork. X-rays show dorsally displaced, dorsally angulated fracture of the distal
radius and possible small fracture of the ulnar styloid. This is a Colles fracture. Answer is treat with closed reduction
and a long-arm cast.
* During a rowdy demonstration with police involvement, a young man is hit with a nightstick on his outer forearm
that he raised to protect himself. He is found to have a diaphyseal fracture of the proximal ulna and anterior
dislocation of the radial head (Monteggia fracture). Answer is closed reduction of the radial head with ORIF (open
reduction internal fixation) of the ulna.
* During a rowdy demonstration with police involvement, a young man is hit with a nightstick on his outer forearm
that he raised to protect himself. He is found to have a fracture of the distal third of the radius and dorsal dislocation
of the distal radial-ulna tract (Galeazzi fracture). Answer is ORIF of broken bone and closed reduction of dislocated
bone. Casting is done in supination to maintain function.
* Young adult falls on an outstretched hand (FOOSH) and complains of wrist pain, there is exquisite pain over the
anatomical snuffbox, AP and lateral x-rays are read as normal. There is a fracture though, of the scaphoid bone
(carpal navicular bone) and is notorious for not showing up on x-ray. Answer is not to send the patient home.
Answer is give patient a thumb-spica cast and do fracture x-rays 3 weeks later.
* A young adult has a FOOSH with wrist pain and anatomical snuffbox pain, AP, lateral, and semipronated oblique
x-rays show a displaced and angulated fracture of the scaphoid. These are notorious for non-union. Answer is do
ORIF and not something like cast and send them home.
* During a barroom fight, a young man tries to punch someone, misses, hits wall. Swollen and tender hand, fractures
of 4th and 5th metacarpal necks. This is a Boxer fracture. Answer is closed reduction and ulnar splint for mild
displacement and Kirshner wire (K-wire) for bad displacements. Splinting should be done with slight wrist
extension and then finger flexion to about 90-degrees.
--------------------------------------------------------------------------------------------------------------------------------------------
Orthopedics: Lower Extremity
* 77yo man falls in the nursing home and hurts his hip. He shows up with the affected leg shortened and external
rotated. X-ray shows displaced femoral neck fracture. Femoral neck has great deal of blood supply, so a fracture
here has low likelihood of femoral neck surviving. Answer would be go to operating room, remove femoral head,
and put in a metal prosthesis. If we said ORIF with femoral head pinning, the elderly patient would be immobilized
for a long time, likely develop hospital-acquired pneumonia, and die.
* 77yo man falls in the nursing home and hurts his hip. He shows up with the affected leg shortened and external
rotated. X-ray shows intra-trochanteric fracture. This can heal. Answer is ORIF with pinning and immobilization.
These patients are prone to DVTs and PEs, so answer also includes post-operative anti-coagulation.
* Unrestrained passenger in a MVC has closed fracture of the femoral shaft. Answer is intramedullary rod fixation.
* Unrestrained front-seat passenger in a MVC has closed comminuted fracture of both femoral shafts. Shortly after
admission he has a BP of 80/50, a pulse rate of 110 and venous pressure of zero. This means hypovolemic
hemorrhagic shock. Rest of physical exam and x-rays are unremarkable including FAST exam. Answer is do some
kind of fixation (external, internal), which helps stop the bleeding.
* Unrestrained front-seat passenger in a MVC has closed comminuted fracture of both femoral shafts. Twelve hours
later he has disorientation, fever, and scleral petechia. Blood gases show PO2 of 60. Answer is fat embolism, which
occurs with long bone fractures. Answer is respiratory support, such as ventilator.
* College student is tackled while playing football and develops knee pain. Knee is swollen and he has pain on
direct palpation of the medial aspect of the knee. With the knee flexed at 30-degrees, passive abduction elicits pain
and abduction occurs further than expected (valgus stress). Answer is medial collateral ligament injury. Answer is
hinge cast if this is the only injury. If several ligaments are injured, do surgical repair.
* College student is tackled while playing football and develops knee pain. Knee is swollen and he has pain on
direct palpation of the lateral aspect of the knee. With the knee flexed at 30-degrees, passive adduction elicits pain
and adduction occurs further than expected (varus stress). Answer is lateral collateral ligament injury. Answer is
hinge cast if this is the only injury. If several ligaments are injured, do surgical repair.

DO NOT DISTRIBUTE - 22 -
Study Notes – Surgery James Lamberg 28Jul2010

* College student is tackled while playing football and develops knee pain. Knee is very swollen, flexed at 30-
degrees the knee can be pulled anteriorly much further than expected. Answer is anterior cruciate ligament injury.
Test of choice is the MRI. Answer is treat with immobilization and rehabilitation for sedentary patient. For athletes,
arthroscopic reconstruction is usually needed for quick healing so they can get back to sports.
* College athlete injures his knee while playing basketball. Several physicians he has seen have given bandages and
pain medications, but he still has a swollen knee and knee pain. He describes catching and locking of his knee.
When the knee is forcefully extended, there is a click and lock. X-rays are normal. This is meniscal tear. Answer is
do an MRI of the knee. Arthroscopic repair is done while saving as much of the meniscus as possible. Extended
meniscectomy can result in degenerative arthritis.
* Young recruit complains of localized pain in his tibia after a forced march at boot camp. He is tender to palpation
at a very specific point in the bone, but x-rays are normal. This is a stress fracture and will not show on day one,
takes a couple of weeks. Treat as if patient has fracture. Answer is put patient in cast. Another example would be a
patient intending on completing a marathon when they are not well trained.
* Pedestrian is hit by a car, physical exam shows angulated between knee and ankle, x-ray shows fractures of the
tibia and fibula at the height of the bumper of the car. Answer for break that can be easily reduced is casting.
Answer for difficult reduction is intramedullary nailing.
* Pedestrian is hit by a car, physical exam shows angulated between knee and ankle, x-ray shows fractures of the
tibia and fibula at the height of the bumper of the car. Satisfactory alignment is achieved and a cast is applied. After
a few hours, there is significant pain and the cast is removed. The pain continues, especially with passive extension
of the toes. Answer is compartment syndrome, commonly in lower leg and forearm. Earliest and most reliable
finding is excruciating pain with passive extension of toes. Answer is fasciotomy now, done by opening all of the
fascial compartment of the limb. In the lower leg, there are 4, so two incisions are made at points of fascial overlap.
* Out of shape, recently divorced 42-yo man tries to impress a woman by challenging her to a game of tennis. In
middle of game, a loud pop is heard like a gunshot and the man falls to the ground clutching his ankle. He leaves the
court with pain and swelling in his lower ankle but is still able to plantarflex his ankle. The next day, an obvious
defect is seen under the skin near the Achilles tendon. This is a rupture of the Achilles tendon. They can plantar flex
because the Achilles is not the only plantar flexor. Answer is casting in an equinus position (tip-toe). For faster
healing, open surgical repair can be done.
* While running to catch a bus, an old man twists his ankle and falls on his inverted (or everted) foot. AP, lateral,
and mortise (ankle) x-rays shows displaced fractures of both malleoli. If fractures are not displaced, do casting. Here
though, we do ORIF to ensure tight ankle joint.
--------------------------------------------------------------------------------------------------------------------------------------------
Orthopedics: Emergencies
* Middle-aged homeless man is brought to the ED because of very severe pain in his forearm. History is he passed
out after drinking a bottle of cheap wine and slept on a park bench for more than 12 hours. There are no signs of
trauma but muscles in forearm are firm and tender to palpation. Passive motion of fingers and wrist elicits
excruciating pain. Pulses at wrist are normal. This is compartment syndrome. Here it is prolonged ischemia followed
by reperfusion, sleeping on twisted arm on hard park bench. Presence of normal pulses does not rule out
compartment syndrome, so don’t exclude it. Pressure at little as 30-35mmHg can damage the muscles, but a pulse
will be present. Answer is emergency fasciotomy.
* Patient presents to the ED complaining of moderate and persistent pain under a long leg plaster cast applied a few
hours ago for an ankle fracture. Answer is remove cast immediately. No other answer is acceptable, medically or
medical-legally. It could be just a wrinkle in the cast or something as bad as compartment syndrome. So answer is
not give analgesics and check the patient in the morning.
* Young man in MCC has obvious open fracture of right thigh, femur is sticking out through jagged skin laceration.
In a multi-trauma patient, orthopedic injuries are typically the last that need to be managed for life threats. However,
open fractures are emergencies and must go to the OR for cleaning within six hours. If the patient gets an infection
with osteomyelitis, it may be osteomyelitis for life as it is difficult to eradicate. Management may be a constant
exercise in diplomacy, deciding what injury has higher priority (neurosurgery for ICP, general surgery for
abdominal bleed, etc). Answer here is this injury must be dealt with within six hours.
* A front seat passenger in a head-on collision relays that he hit the dashboard with his knees and complains of pain
in the hip. He lies on the ED stretcher with a lower extremity shortened, abducted, and internally rotated. Answer is
posterior dislocation of the hip. There is tenuous blood supply to the femoral head, so this is an emergency. Answer
is reduction as soon as possible.
* Healthy 24yo man steps on a rusty nail at the stables where he works as a horse breeder. He arrives to the ED three
days later with a swollen dusty foot and crepitation can be felt, he looks moribund (toxic, septic). This is gas

DO NOT DISTRIBUTE - 23 -
Study Notes – Surgery James Lamberg 28Jul2010

gangrene, a life-threatening soft tissue infection. Most life-threatening soft tissue infections occur in
immunocompromised patients, like mucor mycoses in patients that are extensively burned. Necrotizing fasciitis seen
in AIDS or diabetics. Gas gangrene, however, can occur in a healthy individual with a deep puncture wound. Could
do a Gram stain, but not important immediately. Answer is give large doses of penicillin first, then do surgical
debridement, and finally hyperbaric oxygen therapy as this deactivates the anaerobic bacteria.
--------------------------------------------------------------------------------------------------------------------------------------------
Orthopedics: Neurovascular Injuries
* 48yo man breaks his arm when he falls down the stairs. X-ray shows oblique fracture from middle to distal 2/3rds
of the humerus. He cannot dorsiflex (extend) the wrist. Answer is reduce fracture, splint, and see if that returns nerve
function. If there is paralysis after the reduction, do an open reduction to remove nerve from between bony
fragments.
* Football player hit straight in right leg, suffering posterior dislocation of right knee. Popliteal artery right there has
poor collateral circulation. Answer is ensure good blood supply, check pulses, Doppler studies, arteriogram. Answer
is also do immediate reduction of the dislocation.
--------------------------------------------------------------------------------------------------------------------------------------------
Orthopedics: Injury Patterns With Secondary Injuries
* Window cleaner falls from third story scaffold and lands on feet, x-ray shows comminuted fractures of both
calcaneus. Answer is not to first manage the ankle/foot fractures. Answer is to look for compression fractures of
spine, so do x-rays of thoracic and lumbar spine.
* In a head-on MVC, unrestrained front seat passenger hits dashboard and windshield. Has facial lacerations, upper
extremity fractures, blunt chest trauma. Answer it to look for posterior dislocation of the hip as this is an orthopedic
emergency, so x-rays of the hip.
* Unrestrained front-seat passenger of MCV crashing at high speed has multiple facial lacerations and closed head
injury. Answer is look for cervical spine damage, so x-ray cervical spine and CT scans, especially if unconscious.
--------------------------------------------------------------------------------------------------------------------------------------------
Orthopedics: Common Hand Problems
* 43yo female secretary who does a lot of typing complains about numbness and tingling in her hand, particularly at
night. When she hands her hand limply in front of her (Phalen test), the numbness and tingling are reproduced along
the distribution of the median nerve. Pressing over the nerve at the carpal tunnel elicits symptoms (Tinel test). This
is carpal tunnel syndrome, affecting primarily women. Answer is wrist x-ray including carpal tunnel views,
primarily to rule-out other problems. Management is splinting and anti-inflammatories first. Do not inject anti-
inflammatories first. Next step is electromyography (EMG) prior to surgery.
* 58yo woman describes that she wakes up with her right middle finger acutely flexed and is unable to extend it. She
can pull the finger extended with her other hand, but feels a painful snap. This is trigger finger, seen mostly in
women. Answer is steroid injections. If those don’t work, then surgery.
* Young mother complains of pain along radial side of wrist and first dorsal compartment. She relates that the pain
is often caused by wrist flexion and thumb extension, while carrying the head of her baby. Physical exam reproduces
pain by holding thumb into closed fist and forcing hand into ulnar deviation (Finkelstein test). Answer is De
Quervain tenosynovitis (tendons of the extensor pollicis brevis and abductor pollicis longus muscles). Answer is
steroid injection, possible splint and anti-inflammatory. Surgery rarely.
* A 72yo man of Norwegian ancestry has a contracted hand that can no longer be extended and placed flat on the
table. Palmar fascial nodules can be found. This is Dupuytren contracture, seen mainly in men and alcoholics.
Answer is surgery. Do not do splinting, steroid injections, or anti-inflammatories.
* 33yo carpenter accidentally drives a small nail into the pulp of his small finger. He has throbbing pulp pain, fever,
and all the signs of an abscess within the pulp of the affected finger. Answer is felon and is important because it
requires immediate surgical decompression. Left untreated, tissue will necrose.
* Young man falls while skiing and jams thumb into snow, has collateral laxity at thumb-metacarpal phalangeal
joint. This is Gamekeeper thumb (ulnar collateral ligament injury, skier thumb). Historically named for landed
gentry gamekeepers that were only allowed to eat rabbits and would do so by holding them by the tail and
dislocating the neck with their thumb, producing this injury. Answer is casting.
* Two hoodlums grab a woman’s purse and run away with it. She tries to grab one of the offenders by the jersey, but
the offender pulls away, hurting the woman’s hand. She makes a fist and cannot flex the distal phalanx of the distal
ring finger. This is jersey finger (flexor digitorum profundus tendon injury). Answer is splinting.
* While playing volleyball a young lady injures her middle finger but cannot extend her distal phalanx. This is
mallet finger (extensor tendon injury). Answer is splinting.

DO NOT DISTRIBUTE - 24 -
Study Notes – Surgery James Lamberg 28Jul2010

* While working at a workshop, a young man suffers a traumatic amputation of his index finger. Finger was cleanly
severed at its base. Answer is managed severed finger by cleaning severed finger with sterile saline, wrap in saline-
moistened gauze, put in plastic bag, then put that bag on a bed of ice. Do not put the finger in alcohol, cleaning it, or
putting in formaldehyde. Do not put finger directly on ice or dry ice. This finger can be replanted, as there was a
clean separation of the nerve.
--------------------------------------------------------------------------------------------------------------------------------------------
Orthopedics: Back Pain
* 45yo male gives a history of aching back pain of several months, he has been told he has muscle spasms, was
given analgesics and muscle relaxants. He has a sudden onset of severe back pain when trying to lift a heavy object.
The pain feels like an electrical shock that shoots down his leg. It is aggravated by sneezing, coughing, and
straining. He keeps the left flexed, does not ambulate, and straight leg raise yields excruciating pain. Answer is
lumbar disc herniation. Discogenic pain is the aching pain, neurogenic pain is the electrical feeling pain. Most
common location is L4-L5 or L5-S1. To determine which of those, if pain exits at big toe it’s L4-L5 (heel walking)
and if the pain exits at small toe it’s L5-S1 (toe walking). For diagnosis, answer is MRI. Answer, for the vast
majority, is bed rest for about three weeks. If progressive weakness, need neurosurgical intervention. If patient has
flaccid rectal sphincter, cannot empty bladder, then immediate surgical intervention.
* 46yo man has sudden onset of severe back pain when he tried to lift a heavy object. Pain is like an electrical shock
that shoots down leg, prevents ambulation, keeps leg flexed, straight leg raise gives excruciating pain. Has distended
bladder, flaccid rectal sphincter, and saddle anesthesia. Answer is immediate surgery for cauda equina syndrome.
* Young man began to have chronic back pain at age 34, pain and stiffness is progressive, morning stiffness/pain
worse at rest but improved with activity. This is ankylosing spondylitis. X-ray will eventually show a bamboo spine.
Answer is anti-inflammatory agents and physical therapy.
* 72yo man has had 20lb weight loss and low back pain. Pain is worse at night and unrelieved with rest or positional
pain. Answer is metastatic malignancy (e.g. prostate). Diagnostic test is radionuclide bone scan, most sensitive but
not most specific. If scan is positive, then do x-rays to rule out other reasons for scan to be positive.
--------------------------------------------------------------------------------------------------------------------------------------------
Orthopedics: Leg Ulcers
* 70yo diabetic with indolent, un-healing ulcers of the heel of the foot. Leg ulcer likely began due to neuropathy as
they could not notice the pain. Typical example is diabetic buys new pair of shoes and does not notice the pain.
Classical location is on the heel or head of metatarsal. Answer is control diabetes, keep leg horizontal to prevent
swelling, and long-term management. Eventually, they will likely need an amputation.
* 67yo smoker with high cholesterol and CAD has indolent and un-healing ulcer on tip of toe. Toe is blue and no
peripheral pulses in that extremity. Arterial occlusive disease like this seen at most distal extremities. If damage to
large vessels, we can bypass them. If small vessels, we cannot. So, answer is to determine blood pressures at several
locations in the circulation to determine pressure gradients (blockage); answer is Doppler study. If pressure gradient
found, answer is do arteriogram to find obstructions.
* 44yo obese woman has an indolent, un-healing ulcer above the right medial malleolus. Skin around it is thick and
hyperpigmented, has episodes of synovitis and has varicose veins. Answer is venous stasis ulcer. Always found
above medial malleolus and always around thick skin. Answer for management is providing support to veins, such
as compression stalkings, elastic bandages, and possibly varicose vein surgery.
* 40yo man has a chronic draining sinus in his lower leg since an episode of osteomyelitis. There is an indolent,
dirty looking ulcer at the site with heaped-up tissue growth at the location. Answer is squamous cell carcinoma
(Marjolin ulcer). Answer is biopsy of the edge of the ulcer. Treatment is wide resection.
* Ever since she had an untreated third degree burn to her lower leg at age 14, a 30yo immigrant from Latin America
has shallow ulcerations at the scar site that heal and breakdown all the time. Over the past few months there is an
indolent, dirty looking ulcer at the site with heaped-up tissue growth at the edges. Answer is squamous cell
carcinoma (Marjolin ulcer). Answer is biopsy of the edge of the ulcer. Treatment is wide resection.
--------------------------------------------------------------------------------------------------------------------------------------------
Orthopedics: Foot Pain
* Older overweight man complains of sharp heel pain every time his foot strikes the ground. The pain is worse in the
morning, preventing him from putting weight on the heel. X-ray shows a bony spur matching the location of pain.
Physical exam shows exquisite tenderness over spur. Answer is plantar fasciitis. Culprit is not bony spur, but is
inflammation of plantar fascia. Management is not surgical excision of bony spur. Management is supportive,
including analgesics and devices that prevent heel strike during walking. Usually goes away within a year or two.
* Woman who usually wears high-heel pointed shoes (or Texas cowboy who wears pointed boots) complains of pain
in the forefoot after prolonged standing and walking. Very tender spot is found at third interspace. Answer is Morton

DO NOT DISTRIBUTE - 25 -
Study Notes – Surgery James Lamberg 28Jul2010

neuroma, inflammation of common digital nerve. Management is wear more sensible shoes. Further management is
to excise the neuroma.
* 55yo obese man suddenly develops swelling, redness, and exquisite pain at the first metatarsal joint. Answer is
gout. Diagnosis is via serum uric acid or uric acid found in joint fluid. Management is colchicine, allopurinol,
indomethacin (for acute pain).
--------------------------------------------------------------------------------------------------------------------------------------------
Pre-Operative Assessment: Cardiac Risk
* 72yo man with a history of multiple myocardial infarctions is scheduled to have an elective sigmoid resection for
diverticular disease. Pre-op radionuclide ventriculography shows EF less than 0.35. This is a no-go situation. If
EF<35%, answer is do not do surgery as there is risk of intra-operative myocardial infarction. Or, answer is non-
operative management.
* 72yo chronically bedridden man is being considered for emergency cholecystectomy for acute cholecystitis that is
not responding to medical management. He had a transmural MI 4 months ago, has Afib, 8-10 PVCs per minute, and
JVD. Answer is prohibitive operative risk, as he has most of the Goldman risk factors. Anesthesiologists are
expected to assign numbers and quantify risk. So again, answer is avoid surgery.
* 72yo man scheduled to have an elective sigmoid resection for diverticular disease and pre-op assessment shows
venous jugular distention (JVD). The assumption is congestive heart failure. So, answer is managed the CHF with
calcium channel blockers, beta-blockers, digitalis, diuretics, etc. Answer is do not operate.
* 72yo man scheduled to have an elective sigmoid resection for diverticular disease and pre-op assessment shows he
had a transmural MI 2 months ago. A recent MI (under 6 months) tremendously increases operative mortality, to
nearly 40%. After 6 months though, mortality is about 6%. Answer is to wait at least 4 months from now.
* 72yo man needs elective repair of large AAA has history of severe progressive angina. This is high risk. So
answer is first evaluate for coronary vascularization. If CAD, operate on coronary arteries first.
--------------------------------------------------------------------------------------------------------------------------------------------
Pre-Operative Assessment: Pulmonary Risk
* Deals mostly with smokers that have ventilatory, not oxygenation, problems.
* 61yo man with a 6 pack-year history and COPD needs elective AAA repair, currently smokes 1 pack/day. This is
high risk, relating to ventilation (not oxygenation). Answer for laboratory test is blood gases (PCO2 high) or
pulmonary function studies (FEV1 for ventilation). Answer is also cessation of smoking for 8-weeks with intensive
pulmonary therapy to decrease risk. Don’t pick the answer that says stop smoking for a week and operate, as you
first get bronchorrhea, which would make it worse.
--------------------------------------------------------------------------------------------------------------------------------------------
Pre-Operative Assessment: Hepatic Risk
* A cirrhotic is bleeding from a duodenal ulcer and surgical intervention is considered. Bilirubin is 3.5, prothrombin
time (PT) is 22 seconds, serum albumin is 2.5, has encephalopathy. Answer is do not operate as this patient will die.
All of these factors alone preclude operation. Bilirubin at 10 or even 20 is alright if due to obstruction, but if it is due
to hepatic damage then we cannot operate. Mortality, quantified, is about 100%.
* A cirrhotic with a blood ammonia concentration above 150ng/dl, albumin level below 2, bilirubin above 4…any
one of these precludes operation due to degrade of hepatic dysfunction and inability to accept the anesthesia dose.
Death incidentally occurs due to high-output cardiac failure with low peripheral resistance. Answer for expected
outcome for this patient is high-output cardiac failure.
--------------------------------------------------------------------------------------------------------------------------------------------
Pre-Operative Assessment: Nutritional Risk & Metabolic Complications
* Elderly gentleman needs palliative surgery for cancer of the esophagus, has lost 20% of his body weight over the
past two months, serum albumin is 2.7. Has anergy to injected skin test antigens and a serum transferrin level of less
than 200mg/dl. This man is severely malnourished, judged by weight-loss exceeding 20% body weight and serum
albumin below 3. Anergy to tuberculin skin test is another sign, serum transferrin may be used too if less than 200.
Mortality risk is many fold higher than those nourished. Answer is a brief period of intensive preoperative
nutritional therapy, preferably 7-10 days delivered in the gut (e.g. feeding tube), to drastically reduce the risk.
* Elderly diabetic male presents with acute cholecystitis for a few days. Is profoundly dehydrated, in coma, with a
blood sure of 950, severe acidosis, and ketone bodies everywhere. Answer is do not operate now. Answer is
rehydrate patient, correct diabetic ketoacidosis at least to 7.4, and normalize blood sure. So, answer is standard
medical management of diabetes before surgical management of gallbladder. As long as this inflamed gallbladder is
in place, it should be noted that the diabetes parameters may not reach normal levels with management.
--------------------------------------------------------------------------------------------------------------------------------------------
Post-Operative Assessment: Fever

DO NOT DISTRIBUTE - 26 -
Study Notes – Surgery James Lamberg 28Jul2010

* Shortly after the onset of general anesthetic with inhaled halothane and paralytic succinylcholine, a patient
develops a rapid rise in body temp exceeding 104F. Metabolic acidosis and hypercalcemia are noted. History reveals
a family member died under general anesthesia years ago. This is malignant hyperthermia, due to congenital enzyme
absence needed for succinylcholine breakdown. Answer is treated with the muscle relaxant IV dantrolene and
expected support measures such as oxygen, correcting acidosis, cooling blanket, and checking for myoglobinemia
and myoglobinuria with appropriate treatment.
* 45mins after completion of a cystoscopy, a patient develops chills and a fever spike to 104F. Answer is
bacteremia. Answer for management is blood cultures times three and empiric antibiotics then switching to most
narrow-spectrum antibiotic that shows susceptibility with blood culture results.
* In the first post-op day after an abdominal procedure, a patient develops a fever of 102F. Answer is atelectasis.
Answer for exam is chest x-ray even though we’d look at the wound, IV sites, and inquire about UTI symptoms.
Answer for therapy is improving ventilation, coughing, deep breathing, postural drainage, incentive spirometry, and
ultimate therapy is bronchoscopy (but rarely needed).
* In the first post-op day after an abdominal procedure, a patient develops a fever of 102F. Patient is not complying
with atelectasis therapy and by day three he still has fever. Answer is pneumonia. Treatment of atelectasis relies on
the patient. Exercises are not sufficient. Answer is get chest x-rays, sputum cultures, and antibiotics.
* Patient had major abdominal surgery, is afebrile during first two post-op days, but on day three has a fever spike to
103F. Answer is UTI. Answer for testing is urinalysis and urinary culture. Answer for management is antibiotics.
* Patient had major abdominal surgery, is afebrile during first four post-op days, but on day five has a fever spike to
103F. Answer is DVT. Could be early wound infection or late UTI. Physical exam here is mostly worthless,
including Homan sign (50% accurate at best). Answer is do Doppler studies.
* Patient had major abdominal surgery, is afebrile during first six post-op days, but on day seven has a fever spike to
103F. Answer is wound infection. Physical exam of wound is likely to demonstrate infection, even at the level of
redness without pus. A sonogram could differentiate between pus or no pus. Answer is management of
* Mnemonic (5Ws): Wind, Water, Walking, Wound, (Wonder drugs). These are days 1, 3, 5, 7. Wonder drugs, or
medications, is an option when everything else has been ruled out.
* Patient had major abdominal surgery had normal post-op course until the tenth day, begins to spike temps to 102F
to 103F daily. Answer is likely subphrenic abscess, or subhepatic abscess, or even pelvic abscess. Answer for
diagnosis is sonogram. As an aside, before sonogram this was another W for “Wonder Where” as clinicians would
wonder where the abscess was located.
--------------------------------------------------------------------------------------------------------------------------------------------
Post-Operative Assessment: Chest Pain
* Second post-op day for abdominal peritoneal resection for cancer, a 72yo complains of severe retrosternal pain
radiating to the left arm, has shortness of breath and tachycardia. Answer is either myocardial infarction (quick) or
pulmonary embolism (slow). So, answer here is myocardial infection as less than three days. Answer for
management is ECG and cardiac enzymes, then follow MI management except do not use fibronlytics as the patient
is freshly post-op and thus we’d make things worse. Peri-op MIs have higher mortality than non-peri-op MIs.
* During the performance of an abdominal peritoneal resection for rectal cancer in a 72yo, unexpected severe
bleeding is encountered and the patient is hypotensive on and off for an hour. Anesthesiologist notes ST depression
and T-wave flattening on the monitor. Answer is peri-operative MI, hypotension is the triggering factor. Answer is
cardiac enzymes (troponin, CK), treatment follows MI management without clot busters.
* On seventh post-op day after pinning of a broken hip, 76yo man develops severe pleuritic chest pain and shortness
of breath, found to be diaphoretic, tachycardic, and has JVD and prominent forehead veins. Answer is pulmonary
embolism. Answer is do blood gases to find hypoxemia and hypocapnia. If it were respiratory failure, we’d have
hypoxemia and hypercarbia. In PE, patient is likely hyperventilating. Answer for definitive diagnosis is pulmonary
angiogram, but rarely done as they are cumbersome in this setting. Practical choice is ventilation/perfusion scan
(V/Q scan). However, V/Q scan only works with normal lungs such as no atelectasis or pneumonia. Newer test and
the correct answer for this scenario is the spiral CT scan of the chest. Answer for management is anticoagulation,
such as heparin, to prevent new clots. This won’t do anything for the clot that’s there, but we don’t need to do
anything for that as the lungs has the most active, naturally occurring system for lysing clots. If the patient cannot be
anticoagulated, we do a vena cava filter (e.g. conical Greenfield filter). Or, if patient gets PE even with
anticoagulation then we do the vena cava filter.
--------------------------------------------------------------------------------------------------------------------------------------------
Post-Operative Assessment: Pulmonary Complications
* An awake intubation is being attempted in a drunk and combative man who has sustained gunshot wounds to the
abdomen. In the ongoing struggle, the patient vomits and aspirates a large amount of gastric contents with

DO NOT DISTRIBUTE - 27 -
Study Notes – Surgery James Lamberg 28Jul2010

particulate matter. Answer for management is lavage and removal of particular matter via bronchoscopy. Answer
after bronchoscopy is bronchodilators and ventilatory support. Steroids are not necessary.
* A man with advanced pulmonary tuberculosis is undergoing laparotomy for a perforated terminal ileum,
secondary to intestinal TB. Halfway through the case it becomes progressively difficult to bag him and his BP
steadily declines as his CVP steadily rises. Here, a pulmonary bleb broke and the air is being injected into the
pleural space, causing a tension pneumothorax. Answer for management is to empty the pleural space. In a draped
patient, the surgeon may make a tiny needle hole in the diaphragm, then place a chest tube after surgery.
* 18-hours after major surgery a patient becomes disoriented. In all post-op patients who become disorientated,
check blood gases. If a patient arrived in ED in coma, we’d suspect diabetic coma, alcoholic coma, or drug
overdose. So answer is check blood gases.
* In the second week of a stormy complicated post-op period in a young patient with multiple GSWs to the
abdomen, he become progressively disoriented and unconscious. He has bilateral pulmonary infiltrates and a PO2 of
65 while breathing 40% oxygen, no evidence of CHF. This is acute respiratory distress syndrome (ARDS). Answer
for management is mechanical respiratory support with positive-end expiratory pressure (PEEP). PEEP can damage
lung the same way oxygen can, allowing some degree of hypercapnia so we’re not pushing too far. Also, with
ARDS, there is a good chance sepsis is present. So, answer also involves CT scan of areas where an abscess may be
hiding and drainage of any found abscess.
* Alcoholic man checks in to have elective colon resection for recurrent diverticular disease. He swears to everyone
that he has not touched a drop of alcohol for the past 6-months. In the 3rd post-op day, he becomes disoriented,
combative, claims to see elephants crawling up the walls, and the wife reveals the patient drank heavily up to the
day of the operation. Answer is delirium tremens (DTs), alcohol withdrawal. In post-op setting, DTs have a high
mortality. Answer is give 5% IV alcohol in 5% dextrose, although this is very controversial. Psychiatrists will
disagree and point out incongruity in treatment. But here the surgeons are not treating the alcoholism, the objective
is survival during the post-op period so it is acceptable to defer the treatment of the addiction. Immediate treatment
would be benzodiazepines like diazepam (Valium) or lorazepam (Ativan).
* 12-hours after completion of an abdominal hysterectomy, a 42yo woman becomes confused, lethargic, complains
of severe headache, has grand mal seizure, and goes into coma. Review of the chart shows an order for 5% dextrose
in water (D5W) run in at 125cc/hour was mistaken as 525cc/hour. Answer is acute water intoxication. Post-op
patients release ADH in response to trauma, and if they get fluids containing no sodium, they retain the water.
Answer for diagnosis is serum sodium concentration, with low findings (e.g. 120). Answer for management is
controversial as there is high morbidity/mortality regardless. So, give very careful administration of hypertonic
saline. If we reverse too quickly in chronic hyponatremia, we are likely to produce central pontine myelinolysis.
Mannitol may also be used with the hypertonic saline.
* 8-hours after completion of a transphenoidal hypophysectomy for a prolactinoma, a young woman become
lethargic, confused, and comatose. Records show urinary output has averaged 600cc/hour while her IV fluids are
going in at 100cc/hour. Answer here is inability to produce ADH, maybe posterior pituitary stalk transection. So,
surgically induced diabetes insipidus. Answer for testing is serum sodium concentration, showing high levels (e.g.
155). Answer is replace lost water and we can do so rapidly without fear of inducing cerebral edema. Give a few
liters of D5W (most diluted). We could also give 1/3 normal saline or 1/4 normal saline if we’re worried. We could
also give ADH for the patient, absorbed via nasal mucosa.
* Cirrhotic patient goes into coma after an emergency portacaval shunt for esophageal varices. Ammonium is found
to be high (ammonium not converted to ammonia) or patient has hypokalemic alkalosis, high cardiac output, and
low peripheral resistance (overt liver failure). This patient has ammonium intoxication. Answer is clean out bowel
with enema and do locally acting antibiotics to get rid of the source of the ammonium.
--------------------------------------------------------------------------------------------------------------------------------------------
Post-Operative Assessment: Urinary Complications
* 6-hours after undergoing a hemorrhoidectomy under spinal anesthesia, a 62yo man complains of suprapubic
discomfort and fullness. He feels the need to void but has not been able to do it since the operation. Suprapubic mass
is dull to percussion. Answer is urinary retention. Standard protocol is to alert physicians post-op if the patient has
not voided within 6-hours. Answer for treatment is do straight-catheter of bladder. If it needs to be done again, we
may think about leaving in the catheter (Foley). If an operation is predicted to cause urinary problems, an indwelling
urinary catheter may be placed pre-op.
* A man has had an abdominal perineal resection for rectal cancer and an indwelling Foley catheter was left in place
after surgery. His urinary output has been zero over the past two hours. Vital signs are stable, so kidneys are
perfused. When we see urinary output of zero, think mechanical problem. Answer is kinked catheter (straighten) or
plugged catheter (flush). Answer is not insufficient fluid administration or acute tubular necrosis.

DO NOT DISTRIBUTE - 28 -
Study Notes – Surgery James Lamberg 28Jul2010

* Several hours after multiple surgeries for blunt trauma in an average sized adult, the urinary output per hour is
12cc, 17cc, and 9cc. BP is 95 to 130 systolic over those three hours. Patient is not in shock and there is oliguria, so
either we are behind on fluids or acute renal failure. Answer for testing is bolus fluid (500cc) as it would increase
urinary output if we were behind on fluid. If renal failure, this fluid bolus would not change the oliguria. However,
that is a low-tech way and there is an elegant way…Answer is measure urinary sodium. If low fluid volume, kidney
will hold on to salt so the urinary sodium would be below 20. If kidney failure, urinary salt will be high, >40. We
could also calculate the fractional excretion of sodium (FENa), which exceeds 1 in renal failure. If we are behind on
fluid, the FENa is below 1. Answer for management is either more fluid (if we’re behind) or fluid restriction (if
renal failure).
--------------------------------------------------------------------------------------------------------------------------------------------
Post-Operative Assessment: Abdominal Distension
* 4 days after exploratory laparotomy for blunt abdominal trauma, with resection and anastomosis of damaged small
bowel, patient has abdominal distension without pain, has no bowel sounds, has not placed flatus, and x-ray shows
dilated loops of small bowel without air-fluid levels. Answer is paralytic ileus, even at four days out. Answer for
laboratory test is serum potassium, as hypokalemia may compound and perpetuate paralytic ileus. If a few more
days (day 6-8) go on with same inability to pass flatus, we must think about mechanical obstruction such as
adhesions. Diagnosis for mechanical obstruction is small amount of oral/nasal barium and we follow it via serial x-
ray every few hours.
* Elderly gentleman with Alzheimer disease who lived in a nursing home is operated on for a fracture femoral neck.
On the 5th post-op day, his abdomen is grossly distended, tense, and not tender, with occasionally bowel sounds, x-
ray shows very distended colon and a few small bowel dilated proximal loops. Answer is Ogilvie syndrome
(pseudo-obstruction). Typically seen in elderly patients with poor activity who then are further immobilized by
operation, even a non-abdominal procedure. Answer for management is colonoscopy, to decompress the colon and
to examine for colon cancer, and a long rectal tube may be left in place so there is an avenue for gas decompression.
--------------------------------------------------------------------------------------------------------------------------------------------
Post-Operative Assessment: Wound Complications
* Fifth post-op day after laparotomy, noted that large amounts of salmon-colored clear fluid is soaked in the
dressings. The skin is found to be intact. Answer is wound dehiscence. The pink fluid is peritoneal fluid that is
seeping through and soaking the dressing, pink due to the little bit of blood. If we do not recognize this, the patient
could move or cough and tear through skin into evisceration. Answer for management is careful protection of the
wound, kept in bed, not lots of movement, taping wound together, abdominal binders, etc. At a later convenient date,
re-operate for ventral hernia.
* The nurses report that a patient on his fifth day post-op for a laparotomy has been draining clear pink fluid from
his abdominal wound. A medical student removes the dressing and asks the patient to sit-up so the patient can be
helped out of bed and moved to the treatment room. When the patient complies, the wound opens widely and a
handful of small bowel rushes out. Answer is evisceration (emergency), with high morbidity and mortality. Answer
for management is cover bowel with large moist dressings soaked in warm saline (prevent hypothermia) while
arrangements are being made to rush the patient to the OR for re-closure.
* On the seventh post-op day, the inguinal incision of an open herniorrhaphy is found to be red, hot, tender, and
boggy. The patient has had a fever for the past two days. Answer is wound infection. Answer for management if just
redness (cellulitis) is antibiotics geared toward skin flora. Answer for management with an abscess is drainage.
Answer for detecting abscess is sonogram.
* Nine days after a sigmoid resection for cancer, the wound drains a brown fluid that everyone recognizes as feces.
The patient is afebrile and otherwise doing quite well. By definition, this is a fistula. We know it isn’t draining from
a partially drained collection (e.g. anastomosis leak) cause they’d be febrile. If fistula is high volume (2-3L/day) and
from high up (duodenum), a lot of things would be wrong (severe fluid and electrolyte problem). If it were high up,
the fluid would be loaded with enzymes and would digest the abdominal wall. In this case, this is just inconvenient
for the patient but is not life threatening and will eventually heal. Answer is not do a colostomy or parenteral
nutrition. Answer is leave fistula as it is, it will probably get better. If the patient had a fever and leukocytosis, you’d
have to go in and drain the contents. If there was an acute abdomen, surgery immediately.
* Eight days after a difficulty hemigastrectomy and gastroduodenostomy for gastric ulcer, a patient begins to leak 2-
3L of green fluid per day through the right corner of his subcostal wound. Answer is not immediate operation for re-
anastomosis, as fresh fistula will be a nightmare of an operative field. Answer is fluid and electrolyte replacement,
good replacement option here is Lactated Ringer, delivery of elemental nutrient solution (Vivonex) distal to fistula
or parenteral nutrition if enteral not an option, and protection of abdominal wall from digestive fluid so compulsive
dressing changes or suction device hooked to ileostomy bag and protecting skin with zinc oxide or powder.

DO NOT DISTRIBUTE - 29 -
Study Notes – Surgery James Lamberg 28Jul2010

* Mnemonic for things that prevent fistula from healing: FETIIID, Foreign body (sponge, catheter), Epithelialization
(fills lumen before granulation tissue connects), Tumor (fistula through tumor, or tumor creating fistula),
Infection/Irradiated tissue/Inflammatory bowel disease, Distal obstruction (e.g. adhesive band in bowel).
--------------------------------------------------------------------------------------------------------------------------------------------
Post-Operative Assessment: Fluids & Electrolytes
* 8-hours after completion of a transphenoidal resection of a prolactinoma, a young woman becomes lethargic,
confused, and eventually comatose. Urinary output since surgery has been 600cc/hour while IV fluids are going in at
100cc/hour, serum sodium is 152mEq/L. This is due to the diabetes insipidus. Anytime sodium is high, it is most
likely due to loss of water. Gaining sodium would be seen with edema and ascites. Answer is treat patient with
diluted fluid to replace the loss, such as D5W or 1/3 or 1/4 normal saline. For every 3mEq that the serum sodium is
above normal represents roughly 1L of water lost. So if they went to surgery at 140 and are at 152, they lost about
4L of water. This patient would also need ADH.
* A group of Mexican nationals are smuggled into the U.S. in a closed metal truck, when chased by border control
the smugglers abandon their travelers in locked truck with no water. The victims are found and rescued 5 days later.
One arrives at the hospital in obvious dehydration with a serum sodium of 155mEq/L. So, this patient has
hypernatremia and dehydration, lost approximately 5L of fluid for above 140mEq. Do not pick the answer that gives
the patient 5L D5W over the next couple hours, cause it would kill the patient (cerebral edema). Answer is not to
give 5L of D5W over five days. Answer is to reverse volume loss first, within a few hours, but not the tonicity
correct immediately. So, answer is give 5L of 1/2 normal saline over next few hours (pick solution in between
patient’s tonicity and what’s normal).
* 12-hours after an abdominal hysterectomy, a 42yo woman becomes confused, lethargic, has grand mal seizure,
and goes into coma. Chart shows 125cc/hour of D5W but was implemented as 525cc/hour. Serum sodium is
122mEq/L, acute water intoxication. Answer is treat with 3% hypertonic saline (or 5% NS) and mannitol. This is
high morbidity and high mortality situation, avoided by not giving dilute fluids is high volume. Do about 250cc each
time then re-evaluate the situation.
* 62yo woman comes in for her scheduled chemotherapy administration for metastatic breast cancer. Although she
is quite asymptomatic, the lab reports that her serum sodium is 122mEq/L. This is a very chronic situation. The
metastatic cancer can be creating ADH, and normal fluid intake leads to extra fluid retention. We know this
happened slowly because she is asymptomatic. If we corrected this quickly, we could kill the patient or cause central
pontine myelinolysis and cripple her. Answer, if we are compelled to correct her, is water restriction. We could also
use medications to counteract ADH (e.g. demeclocycline, lithium).
* Patient has lower than normal sodium concentration. Answer is never loss of sodium, as it can never be lost as a
solid. Also, there are no hypertonic fluids that can be lost from the body except the kidney, but in that case the
kidney is doing what is suppose to do. Fluids from skin are almost pure water, fluids from lungs are pure water.
There is one exception, sorta, with sequential development…
* 68yo woman comes in with obvious incarcerated umbilical hernia, has gross abdominal distension, is clinically
dehydrated, and reports persistent fecaloid vomiting for past 5 days. She is awake and alert with serum sodium of
118mEq/L. So she began by losing isotonic fluids from the GI tract. She has been severely volume depleted for
several days. She likely got thirsty and drank water or tea or soda or other things with low sodium. Answer for
management is correct volume depletion within a few hours (normal saline or Ringer lactate) and correct
hyponatremia over a few days. If alkalotic (vomiting acid) then normal saline, if vomiting small bowel contents or
diarrhea and is acidotic, then Ringer lactate.
* Patient with severe diabetic ketoacidosis comes in with profound dehydration and serum potassium of 5.2. After
several hours with insulin and IV fluid administration, serum potassium is 2.9. So what’s the deal with the
potassium? In acidosis, potassium comes out of the cells to exchange hydrogen in the blood. On the other side, the
kidney sees the high potassium in the blood so it dumps it into the urine. After we correct the problem, the cells say
“give me back my potassium” but that potassium is in the toilet. So, we have a profound hypokalemia. In the
treatment of diabetic ketoacidosis, we should begin to give potassium. This is an example of a time we give
potassium when potassium is not low in the serum, and also at a high rate. Normally, we don’t exceed 10mEq/h IV
of potassium, which is 240mEq/day where we normally get 75mEq/day from food. Answer for this case is
20mEq/hour of potassium for several hours.
* 18yo woman slips and falls under a bus and her right leg is crushed. On ED arrival she is hypotensive and gets
several units of blood. Over the next several hours she is in and out of hypovolemic shock and she develops
acidosis. Serum potassium was 4.8 at admission is now 6.1 a few hours later. Crush injury destroys cells and thus
dumps potassium. Blood transfusion has high potassium as hemolysis takes place while the blood sits at the blood
bank. In and out of shock, kidneys cannot eliminate potassium as they normally would. Above 6 is dangerous

DO NOT DISTRIBUTE - 30 -
Study Notes – Surgery James Lamberg 28Jul2010

hyperkalemia, so the only true way to fix this is hemodialysis, but that takes times. Answer here is give 50% glucose
and insulin, creating an anabolic phase that puts potassium into cells. We could do nasogastric suction, which may
help a little, and Kayexalate (sodium polystyrene sulfonate) to exchange potassium in GI tract. The use of sodium
bicarbonate in this instance has fallen out of favor. Lethal event here is cardiac arrest due to hyperkalemia, so we
can protect the myocardium with calcium. So, answer is calcium, glucose, and insulin with calcium protecting the
patient most effectively and most quickly.
* An elderly alcoholic diabetic male with marginal renal function sustains multiple trauma while driving under the
influence of alcohol. During resuscitation, he is in and out of shock, blood gases show pH of 7.1, PCO2 of 36,
electrolytes show sodium of 138, chloride of 98 and bicarbonate of 15. This is metabolic acidosis. We would likely
see high lactic acid in the blood. Management is by correcting the underlying problem. Giving bicarbonate or a
precursor (acetate, lactate) we could tip the balance, but this doesn’t help here because the underlying problem is a
low-flow state. Answer is vigorous rehydration with a fluid that helps, like Ringer lactate (not saline).
* Patient who has had subtotal gastrectomy for cancer with Bilroth II reconstruction has a blowout of the duodenal
stump and creates a duodenal fistula. Past ten days has been draining 750 to 1500cc/day of green fluid. Sodium is
132, chloride is 104, bicarbonate is 15, pH is 7.2, PCO2 is 35. This is metabolic acidosis. Reason here is loss of
bicarbonate fluid, bile. Answer here is give bicarbonate. Bicarbonate is not in IV bag form because it causes
precipitation, so it is found in amps containers.
* Patient with severe peptic ulcer disease develops pyloric obstruction and has vomiting of clear gastric contents (no
bile) for several days. Sodium is 134, chloride is 82, potassium is 2.9, bicarbonate is 34. This is hyperchloremic
hypokalemia metabolic alkalosis secondary to loss of acid gastric juice. Answer is rehydration with saline (not
Ringer lactate). Answer also may include giving a hydrogen donor, such as amino acid buffered diluted hydrochloric
acid or ammonium chloride (very rarely). Answer is to give generous intake of potassium chloride, between 5 and
10mEq/hour of KCl. A drip of 5mEq/h would be enough for mild metabolic alkalosis and kidney will do the job.
--------------------------------------------------------------------------------------------------------------------------------------------
General Surgery: Esophagus & Stomach
* 62yo man describes epigastric and substernal pain that he cannot characterize well. At times it sounds like reflux,
at times it does not. Sonogram of gallbladder, ECG, and cardiac enzymes are negative. Answer to determine reflux
is esophageal pH monitoring, which can tie the pH to the pain symptoms.
* Patient has horrible crushing pain each time they swallow. This is nutcracker esophagus (esophageal spasm).
Answer for testing is monometry.
* 54yo obese man gives history of burning retrosternal pain and heart burn brought about by bending over, wearing
tight clothing, or lying flat at night. He gets symptomatic relief from antacids but has not been formally treated,
although it seems to be progressing over the years. This is clearly gastroesophageal reflux (GERD). Answer though,
since it has been occurring for many years, is to examine lower esophagus for damage, e.g. Barrett disease. Answer
therefore is endoscopy and biopsies. Answer is not simply to treat this patient and let them go.
* 54yo obese man gives history of burning retrosternal pain and heart burn brought about by bending over, wearing
tight clothing, or lying flat at night. He gets symptomatic relief from antacids but has not been formally treated,
although it seems to be progressing. Endoscopy shows peptic esophagitis and Barrett esophagus. Answer for
management is Nissen fundoplication. It is also possible to relieve symptoms by intensive therapy, such as
omeprazole (most effective PPI). However, this requires high doses for a long time, which can effect the stomach
and there are reports of carcinoid tumors developing. So favored treatment is fundoplication, which is like a valve.
* 54yo obese man gives history of burning retrosternal pain and heart burn brought about by bending over, wearing
tight clothing, or lying flat at night. He gets symptomatic relief from antacids but in spite of strict adherence to
therapy, the problem seems to be progressing. Endoscopy shows severe peptic esophagitis with no dysplastic
changes, surgical treatment has been recommended. We have to be sure the motility is normal to ensure we can push
past this new one-way valve via fundoplication. Answer is monometry, gastric emptying studies, and barium
swallow prior to surgery.
* 47yo woman describes difficulty swallowing which she has had for many years. She says liquids are more difficult
to swallow than solids. She has learned to sit up straight and wait for the fluids to make it through. Occasionally she
regurgitates large amounts of undigested food. Answer is achalasia of the esophagus, key is liquids are more
difficult to swallow. If the problem were mechanical (e.g. long standing reflux with stricture), the problems would
be solids. Here it is a functional problem, similar to denervation. Answer is not barium study or endoscopy showing
megaesophagus. Answer for diagnosis is monometry, shows nature of the problem. Answer for treatment is
medically with repeated dilatations or surgically with Heller myotomy.
* 55yo black man with a history of smoking and drinking describes progressive dysphagia with difficulty
swallowing meat, then soft foods, now liquids. There is a place near the sternum he can locate where food sticks and

DO NOT DISTRIBUTE - 31 -
Study Notes – Surgery James Lamberg 28Jul2010

he has lost 30lbs of weight. Answer is cancer of the esophagus. Most likely squamous cell carcinoma, seen in
African Americans, smokers, drinkers. Adenocarcinoma would be with GERD. Answer for diagnosis is endoscopy
and biopsy. However, answer for next step is barium swallow. Barium swallow doesn’t diagnose, but a perforation
during endoscopy could be deadly. So the barium study is done first before we put scope down. Next step after
endoscopy is CT scan to determine operability. Most are not operable for cure, but done for palliation. Answer for
surgery is transhiatal esophagectomy (THE), with blunt esophagus dissection and stomach anastomosis. For long-
term survival of the esophagus is colon interposition, but this is more palliative care so do THE.
* 24yo man spends a night cruising bars and drinking heavily. In the wee hours of the morning he is very drunk and
vomits heavily and repeatedly. Eventually he vomits bright red blood. Answer is Mallory Weiss longitudinal
mucosal tear. Answer for diagnosis is endoscopy, where we could also do photocoagulation for treatment.
* 24yo man spends a night cruising bars and drinking heavily. In the wee hours of the morning he is very drunk and
vomits heavily and repeatedly. Eventually he has a particularly violent episode of vomiting and feels a severe
wrenching epigastric pain with low sternal pain of sudden onset. He arrives at ED one hour later in pain, is
diaphoretic, has a fever, has leukocytosis, and looks quite ill. Answer is Boerhaave syndrome, lower esophagus
perforation, rarely occurs. Answer is not anything that delays diagnosis like giving antibiotics and watching the
patient overnight. Answer is not endoscopy cause it could make things worse. Answer is not radiologic studies that
involve barium. Answer is gastrografin swallow. Gastrografin is water-soluble contrast that is safer than barium if it
leaks out, although gastrografin gives low-quality pictures. If gastrografin is negative, then we do a barium study. If
positive for perforation, answer is immediate surgical repair.
* 66yo man has upper GI endoscopy done as an outpatient to check on the progress of medical therapy for a gastric
ulcer. Six hours after the procedure he returns complaining of severe, constant, retrosternal pain that began when he
got home. He looks ill, is diaphoretic, has RR of 30, temp of 104, subcutaneous emphysema at neck. Answer is
instrumental perforation of the esophagus, most common cause of perforation. Answer is gastrografin swallow, if
positive we do surgical repair. If negative we do barium swallow, if barium positive do surgical repair.
* 72yo man has lost 40lbs of weight over a two or three-month period. Has several months of anorexia and vague
epigastric discomfort for past two weeks. Answer is do endoscopy and biopsy or stomach. If cancer is found, next
step is CT scan to determine how resectable. After that, surgery.
--------------------------------------------------------------------------------------------------------------------------------------------
General Surgery: Small Bowel
* 54yo man has had colicky abdominal pain and protracted vomiting for several days. He has developed progressive
abdominal distension and has not passed gas for 5 days. He has high-pitched bowel sounds that coincide with the
abdominal pain. X-ray shows distended loops of the small bowel with air-fluid levels. Five years ago he had an
exploratory laparotomy for a GSW to the abdomen. Answer for problem is mechanical obstruction. Answer for
cause is adhesions. Answer for treatment is not to rush to the operating room, as there is a good chance that the way
the bowel got into trouble it will get out of trouble. Answer is nasogastric suction, NPO, IV fluids, and wait for
about 24-hours for complete obstruction. If stuff makes it to the colon, we can wait for 3-5 days. The major
complication we have to worry about is strangulation of the bowel, ischemia due to blood supply loss. First sign of
this is fever and leukocytosis, followed eventually by constant pain, acute abdomen, and eventually septic shock.
* 54yo man has had colicky abdominal pain and protracted vomiting for several days. He has developed progressive
abdominal distension and has not passed gas for 5 days. He has high-pitched bowel sounds that coincide with the
abdominal pain. X-ray shows distended loops of the small bowel with air-fluid levels. Five years ago he had an
exploratory laparotomy for a GSW to the abdomen. Six hours after hospitalization and being placed on NGT and IV
fluids, he has abdominal pain and signs of an acute abdomen. Answer is strangulated obstruction. Answer for
treatment is emergency surgery.
* 54yo man has had colicky abdominal pain and protracted vomiting for several days. He has developed progressive
abdominal distension and has not passed gas for 5 days. He has high-pitched loud bowel sounds that coincide with
the abdominal pain. X-ray shows distended loops of the small bowel with air-fluid levels. On physical exam a groin
mass is noted. He explains he could push it back at will but has not been able to for 5 days. Answer is incarcerated
hernia. If fever and leukocytosis this would be strangulated hernia (emergency surgery). For incarcerated hernia,
answer is operate, unlike how we don’t operate on adhesions because adhesions return. Most hernia repairs don’t
have reoccurrence. So repair is elective repair, like waiting overnight for the next day.
* If the patient is dehydrated, such as with hypovolemia and high sodium on an electrolyte profile, fix the
dehydration first before rushing to the OR.
* 55yo woman is being evaluated for protracted diarrhea. She gives a bizarre history with episodes of facial flushing
and respiratory wheezing. Jugular venous pulse is noted in her neck. This is not a common problem. Answer is
carcinoid syndrome. Tumor most likely located in small bowel, such as ileum (appendix). Normally, serotonin

DO NOT DISTRIBUTE - 32 -
Study Notes – Surgery James Lamberg 28Jul2010

metabolized by the liver. If metastasis to liver, serotonin can be dumped directly through the IVC into the
circulation, causing flushing and wheezing. Serotonin can damage heart valves, but the lung is another serotonin
metabolism site. So right sided valves are the ones that get affected, thus the JVD from tricuspid damage. Answer
for diagnosis is serum testing of 5-HIAA, a byproduct of 5-HT (serotonin). Answer for treatment is remove primary
tumor. Carcinoid tumors are slow growing so palliation is worthwhile.
* 22yo man develops anorexia followed by vague periumbilical pain, that several hours later becomes severe, sharp,
and constant pain that is well localized to the RLQ to the abdomen. Has tenderness, guarding, and rebound to the
right and below the umbilicus, temp of 99.6, WBC 12500, neutrophils in immature forms (bands/stabs). Answer is
acute appendicitis. Do not waste time ordering time-consuming studies; operate immediately before the appendix
perforates. Perforated appendicitis can lead to ICU admission with a lengthy hospital course. Answer for the given
scenario is immediate surgery. If we’re not sure of acute appendicitis, we can do either an abdominal ultrasound
(sonogram) or a CT scan. Sonogram is cheaper but very much operator independent, so CT scan more common.
Correct answer for non-typical acute appendicitis is CT scan. If sonogram is an option, pick it because it’s cheaper.
--------------------------------------------------------------------------------------------------------------------------------------------
General Surgery: Colon
* Right-sided colon cancer has less luminal impingement and occult (not obvious) blood, so the common finding
initially is anemia then occult blood found. Left-sided colon cancer has luminal impingement (constipation, stool
shape changes such as flat or pencil-thin) and visible blood.
* 59yo man is referred for evaluation because he has been fainting at his job where he operates heavy machinery. He
is pale but physical exam is remarkable only for 4+ blood in stool. CBC shows hemoglobin of 5. This is a cancer of
the cecum or ascending colon. Answer for diagnosis is endoscopy and biopsy, with the endoscopy being a full-
length colonoscopy. Answer for management is blood transfusions, CT scan to determine resectability, then right
hemicolectomy.
* 56yo man has bloody bowel movements. The blood coats the outside of his stools and has been presents on and off
for the past few weeks. For the past two months he has been constipated and his stools have become of narrow
caliber. This is descending colon, sigmoid, or rectal cancer. Answer for diagnosis 45cm or 60cm flexible
sigmoidoscopy, biopsy for diagnosis. Answer for management is CT scan to determine resectability with full-length
colonoscopy as about 3% have a second primary and 10% will develop a second primary, then resection.
* 77yo man has a colonoscopy for rectal bleeding, villous adenoma is found in rectum, several adenomatous polyps
are found in the sigmoid and descending colon. Answer for next step depends on if the polyps are pre-malignant or
not. Familial polyposis is virtually 100% predictable of cancer, so treatment is full resection of territory covered by
polyps. Villous adenoma is 50% likely to be cancerous, resection is mandatory. Adenomatous polyp should also be
snared and removed endoscopically, unless there are many then full resection. However, if the patient has juvenile
polyps or Peutz-Jeghers disease or inflammatory polyps or hyperplastic polyps, none are pre-malignant so do not do
surgery.
* 42yo man has suffered from chronic ulcerative colitis for twenty years. He weighs 90lbs and has had at lease 40
hospital admissions for relapsing disease. After a recent visit, he was placed on high dose steroids and azathioprine
(Imuran). For the past 12-hours he has had severe abdominal pain, temp of 104, and leukocytosis. He looks ill, toxic,
abdomen is tender in epigastric area with rebound. X-ray shows massively distended transverse colon and there is
gas within the wall of the colon. These are all the indications for surgery in chronic ulcerative colitis, which would
cure the disease, something that we cannot normally do with Crohn disease. Indications are >20years as there is a
high risk of carcinoma, disease interferes with nutrition/lifestyle, long-term steroids and immunosuppresants, and
last is the toxic megacolon. Answer also includes removal of rectal mucosa.
* 27yo man is recovering from an appendectomy for gangrenous appendix with perforation. He has been receiving
clindamycin and tobramycin for seven days. Eight hours ago he developed watery diarrhea, crampy abdominal pain,
and leukocytosis. Answer is pseudomembranous colitis from Clostridium difficile. Answer for diagnosis is stool
cultures (takes time), proctosigmoidoscopic exam, and identification of toxin in the stool. Answer for management
is stop offending antibiotic (clindamycin) and do not use antidiarrheal medications as they keep the toxin in the
colon. Answer for treatment is metronidazole, vancomycin, or replenish normal flora.
--------------------------------------------------------------------------------------------------------------------------------------------
General Surgery: Ano-Rectal
* Answer for management of ano-rectal disease always begins by ruling out cancer.
* 60yo man known to have hemorrhoids reports bright red blood on the toilet paper after use. Answer for diagnosis
is internal hemorrhoids (internal bleed, external hurt). Answer for management is rectal exam and
proctosigmoidoscopic exam to rule out cancer and verify diagnosis. Treatment is rubber band ligation or by laser.

DO NOT DISTRIBUTE - 33 -
Study Notes – Surgery James Lamberg 28Jul2010

* 60yo man known to have hemorrhoids complains of anal itching and discomfort, particularly towards the end of
the day. He has mild discomfort when sitting down and finds himself sitting sideways to avoid the discomfort.
Answer for diagnosis is external hemorrhoids, or possibly a prolapse internal hemorrhoid. Answer for management
is rectal exam and proctosigmoidoscopic exam to rule out cancer and verify diagnosis. Treatment is operation with
anesthesia because they are innervated for pain.
* 23yo woman describes exquisite pain with defecation and blood streaks outside the stools. She avoids bowel
movements due to pain, but when she does have them the stools are hard and even more painful. Physical exam
cannot be done because she refuses to have anyone get near the area. Answer for diagnosis is anal fissure. Answer
for management is physical exam under anesthesia, meaning rectal exam and endoscopy. Answer for management is
stool softeners and topical agents like nitroglycerine cream to relax the sphincter; theory is these patients have very
tight anal sphincters and normal mucosal tears have less blood supply, don’t heal, then become painful. If more
aggressive treatment is needed, go for the sphincter; such as a lateral sphincterotomy or forceful dilation or possibly
injections of botulism toxin to paralyze the sphincter.
* 28yo man is brought to the office by his mother. Beginning four months ago he has had three operations done
elsewhere for a perianal fistula, but each has resulted in poor healing and a large surgical area with multiple
unhealed ulcers and fissures with purulent discharge but no palpable masses. Answer is Crohn disease. Management
involves rectal exam and endoscopy to rule out cancer, as it could be squamous cell carcinoma that is necrotic with
discharge. Treatment follows what we do for Crohn disease, such as sulfasalazine.
* 44yo man shows up in the ED at 11pm with exquisite perianal pain and cannot sit. He reports painful bowel
movements with chills and fever. Physical exam shows hot, tender, fluctuating mass between the anus and ischial
tuberosity. Answer is ischiorectal (perirectal) abscess. Answer for management is incision and drainage, but since
this is in the rectal area we must also examine for cancer via rectal exam and endoscopy. If patient is a brittle
advanced diabetic or immunosuppressed, treatment is aimed at preventing fulminating tissue fasciitis such as close
follow-up over next few days.
* 62yo man complains of perianal discomfort and reports fecal streaks soiling his underwear. Four months ago he
had surgical drainage of a perirectal abscess. Physical exam shows a perianal opening and a cord-like tract can be
palpated from the opening to the inside of the anal canal. Brownish discharge can be expressed from the tract.
Answer is fistula in anu. Management includes ruling out cancer.
* Adenocarcinoma of the rectum metastasizes only to lymph nodes within the abdomen. Squamous cell carcinoma
does as well, but also to groin nodes.
* 55yo HIV positive man engages in receptive anal sex has a fungating mass growing out of the anus and rock-hard
and large lymph nodes in both groin areas. He has lost a lot of weight. Answer is squamous cell carcinoma of the
anus. Diagnosis is confirmed on biopsy. Answer for management is to first shrink them before surgery. So, answer
is chemotherapy first then surgery, also called the Nigro protocol or neoadjuvant chemotherapy.
--------------------------------------------------------------------------------------------------------------------------------------------
General Surgery: GI Bleeding
* 3/4 GI bleeds are from the upper GI tract, which is from tip of nose to the ligament of Treitz. The other 1/4 are
usually from the colon with very few before the ileocecal valve. For those that bleed from the colon, typically bleed
from hemorrhoids, polyp, cancer, angiodysplasia, or diverticulosis; these usually affect the older patient. So if
patient is 25yo and has GI bleeding, very likely to be upper GI tract. Older patient could be from anywhere. If
patient is vomiting blood, it is from the upper GI tract. Management is endoscopy.
* 33yo man vomits a large amount of bright red blood. Answer for next step is upper GI endoscopy. Management
when the bleeder is found could be photocoagulation with the endoscope.
* 33yo man has had three large bowel movements recently that he describes as made up entirely of dark red blood.
He is diaphoretic, pale, BP 90/70, pulse rate of 110. Answer for next step of management is place a nasogastric tube
and aspirate. This would confirm upper GI bleed and then we can do the endoscopy. As a note, blood is a GI irritant
so the transit may be quick and thus not have full digestion, leading to blood per rectum and not necessarily melena.
* 65yo man has had three large bowel movements recently that he describes as made up entirely of dark red blood.
He is diaphoretic, pale, BP 90/70, pulse 110. NG tube returns clear green fluid without blood. If the fluid were
white, this excludes up to the pylorus. Here it is green (bile) so we exclude up through the duodenum. Assumption is
fluid replacement to correct hemodynamics. Answer for management would not involve a colonoscopy, as the
bleeding in this case is going right into the instrument. Answer for next step is anoscopy, as diagnosing hemorrhoids
via angiography would not be a good idea. If bleeding is at a large rate, 2cc/min or more, an emergency arteriogram
may be used. 2cc/min is 120cc/hour so a unit of blood every 4 hours would be a sign of bleeding at a fast rate. If the
patient is bleeding less than 0.5cc/min, 16hours for a unit of blood, an arteriogram would not be useful. Answer here
would be colonoscopy once bleeding stops. If we’re in between though, from 0.5cc/hour to 2cc/hour, there is some

DO NOT DISTRIBUTE - 34 -
Study Notes – Surgery James Lamberg 28Jul2010

controversy involving the radioactive tagging of red blood cells. Nuclear medicine can then give an approximate
idea of where the bleeding is occurring and some physicians do this before an arteriogram.
* 72yo man had three large bowel movements that he describes as made up entirely of dark red blood, the last one
was two days ago. He is pale with normal vital signs. NG tube returns clear green fluid without blood. So, this
patient is not actively bleeding and the NG tube isn’t too useful because it was two days ago. Answer for
management is endoscopy of both ends of the GI tract, with upper GI (3/4 of bleeds) and lower GI (1/4 of bleeds).
* 7yo boy passes a large bloody bowel movement. Answer is Meckel diverticulum. Answer for diagnosis is
radioactively labeled Technetium-99m exam, to recognize gastric mucosa.
* 41yo man has been in the ICU for two weeks being treated for idiopathic hemorrhagic pancreatitis. He has had
several percutaneous drainage procedures for pancreatic abscess, chest tubes for pleural effusions, and
bronchoscopies for atelectasis. He has been in and out of septic shock and respiratory failure several times. He
recently vomited bright red blood and as you enter the room he vomits what looks like a pint of blood. Answer is
stress ulcers, multiple shallow furiously bleeding ulcers. This is the reason all ICU patients are always on H2
blockers, antacids, and/or PPIs like pantoprazole (Protonix) to keep pH above 4. Answer for diagnosis is endoscopy.
Answer for treatment is angiogram with selective catheterization and embolization, typically the left gastric artery.
--------------------------------------------------------------------------------------------------------------------------------------------
General Surgery: Abdominal Pain (Acute Abdomen)
* Three causes are perforation (sudden onset with constant severe generalized pain), obstruction (sudden onset with
localized colicky pain), and inflammatory (gradual onset with localized constant pain, fever, and leukocytosis).
* 59yo man arrives at the ED at 2am accompanied by his wife who is wearing curlers in her hair and a robe over her
nightgown. He has abdominal pain that began suddenly an hour ago, is now generalized, constant, and severe. He
lies motionless on the stretches, is diaphoretic, and has shallow rapid breathing. His abdomen is rigid, is tender to
palpation, and has guarding and rebound in all quadrants. Answer is perforation, causing peritonitis. Most likely
duodenal ulcer. Answer for management is exploratory laparotomy, we don’t know the cause but we’re very close to
finding out (as close as the thickness of the abdominal wall). Answer first though is a CXR, ECG, plain abdominal
x-rays, and amylase/lipase for diagnosis.
* 62yo man with cirrhosis of the liver and ascites has generalized abdominal pain starting 12 hours ago. He has
moderate tenderness over the abdomen with some guarding and rebound. He has mild fever and leukocytosis.
Answer is primary/spontaneous bacterial peritonitis (SBP). Answer for diagnosis is get ascites and grow cultures
expecting to get a single bacteria (versus multiple with a perforation). Answer for treatment is antibiotics.
* 43yo man develops excruciating abdominal pain at 8:18pm. When seen in the ED soon after, he has a rigid
abdomen, lies motionless on the table, has no bowel sounds, and is in great pain described as constant. X-ray shows
free air under the diaphragm. Answer is perforation of the GI tract. Answer for management is emergency
exploratory laparotomy.
* 44yo alcoholic man presents with severe epigastric pain that began shortly after a bout of heavy alcohol intake. It
reached maximum intensity over two hours. The pain is constant, radiates to back, is accompanied with nausea,
retching, and vomiting. He had an episode two years ago. Answer is acute pancreatitis. Answer for next step is order
serum amylase or lipase (if soon after problem) or urinary amylase/lipase if three days later.
* 43yo obese mother of six children has severe right upper quadrant abdominal pain that began six hours ago. The
pain was colicky, radiated to the right shoulder and back, accompanied by nausea/vomiting, pain has been constant
for past two hours, has RUQ tenderness/guarding, temp is 101F, WBC is 16000, episodes in past brought about after
eating fatty foods. Answer is acute cholecystitis. Answer for next step of management is sonogram (ultrasound).
* 52yo man has right flank colicky pain of sudden onset that radiates to the inner thigh and scrotum. Microscopic
hematuria is found. Answer is acute nephrolithiasis, stone in ureter. Answer for next step is intravenous pyelogram
(IVP), sonogram, or CT scan.
* 59yo woman has a history of three prior episodes of LLQ abdominal pain for which she was hospitalized and
received antibiotics. She has discomfort starting 12 hours ago, now constant LLQ pain, tenderness, vaguely palpable
mass, fever, leukocytosis. Answer for management is CT scan, which could show the likely diverticulitis or unlikely
things like twisted tubo-ovarian cyst.
* 82yo man develops severe abdominal distension, nausea, vomiting, and colicky abdominal pain. He has not passed
any gas or stool for the past 12 hours, has a typanitic abdomen with hyperactive tinkling bowel sounds, x-ray shows
distended loops of bowel, and very large gas shadow in RUQ tapering to LLQ with the shape of a parrot’s beak.
Two conditions to think of are mesenteric ischemia and sigmoid volvulus. Answer here is sigmoid volvulus, as seen
by the “beaking” on the x-ray at the location of the twist. Answer for diagnosis is protosigmoidoscopic exam, as we
can untwist the bowel on the way in and then leave a rectal tube/stent to prevent recurrence.

DO NOT DISTRIBUTE - 35 -
Study Notes – Surgery James Lamberg 28Jul2010

* 79yo man with atrial fibrillation develops an acute abdomen and comes to the ED two days later. He has a silent
abdomen with diffuse tenderness and mild rebound, there is a trace of blood on rectal exam, he has acidosis and
looks quite sick. X-ray shows distended small bowel and distended colon up to the transverse colon. Two conditions
to think of are mesenteric ischemia and sigmoid volvulus with elderly patient. Answer here is mesenteric ischemia
from the atrial appendage in a patient with atrial fibrillation or mural thrombi from patient who had a recent MI. Up
to transverse colon is territory of the SMA and acidosis due to dead bowel. Answer for diagnosis is exploratory
laparotomy. Answer for management is resection of ischemic segment. Vascular surgery can possibly do an
arteriogram with clot removal if the pain started immediately prior to being seen.
--------------------------------------------------------------------------------------------------------------------------------------------
General Surgery: Liver
* 53yo man with cirrhosis of the liver develops malaise, vague RUQ abdominal discomfort, and 20lb weight loss.
Physical exam reveals a palpable mass that seems to be from the left lobe of the liver, alpha-fetoprotein is
significantly elevated. Answer is hepatocellular carcinoma.
* 53yo man develops vague RUQ abdominal discomfort and 20lb weight loss. Physical exam reveals a palpable
liver with nodularity, two years ago he had a right hemicolectomy for cancer of the ascending colon. His
carcinoembryonic agent (CEA) have been within normal limits right after the hemicolectomy, but now he is ten
times normal for CEA. Answer for next step is CT scan to see how bad the cancer is. Answer for management is
resection of primary and even metastatic if it comes from a relatively slow-growing primary tumor (e.g. colon). This
may not be a cure but can prolong life. Other options include radio-ablation of the tumor. Metastatic cancer to the
liver outnumbers primary liver cancer 20 to 1.
* 24yo woman develops moderate generalized abdominal pain of sudden onset and shortly thereafter faints. In the
ED she is pale, tachycardic, and hypotensive. Abdomen is mildly distended and tender, hemoglobin of 7, no trauma.
On pregnancy inquisition, she denies due to birth control pills since age 14 without missing doses. Answer is hepatic
adenoma, which can break and bleed, and are due to long-term birth control use. Answer for next step is CT scan
and for management is resection. This is very uncommon and thus is not a reason to stop taking birth control pills as
complications of normal pregnancy and delivery outweigh these complications.
* 7month pregnant woman suddenly bleeds into abdomen and goes into shock, answer is small visceral aneurysm of
hepatic or splenic artery.
* 44yo woman is recovering from an episode of acute ascending cholangitis secondary to choledocholithiasis. She
develops fever and leukocytosis, RUQ tenderness, sonogram reveals liver abscess. Answer is biogenic abscess, a
complication of biliary disease. Management is percutaneous drainage, such as with interventional radiology (IR).
* 29yo migrant worker from Mexico develops fever and leukocytosis with tenderness over liver on percussion. He
has mild jaundice and an elevated alkaline phosphatase. Sonogram shows a normal biliary tree and an abscess in the
liver. Answer is amoebic (amebic) abscess, common in Mexico. Answer for diagnosis is serology for antibodies but
we begin empiric treatment right away. Answer for treatment is not drainage first, it is metronidazole. If the patient
does not respond to metronidazole, then we drain it. Do not pick the answer that involves draining the abscess pus
and growing it in the lab. The amoeba grows in the wall of the abscess, not pus, which is why metronidazole works.
--------------------------------------------------------------------------------------------------------------------------------------------
General Surgery: Jaundice
* Jaundice is due to three general groups of conditions, hemolytic (<10mg unconjugated/indirect bilirubin),
hepatocellular (high transaminases like SGOT and SGPT with a modest elevation of alkaline phosphatase), and
obstructive (high level of alkaline phosphatase without very high transaminases).
* 42yo woman is jaundiced, total bilirubin of 6, lab reports show unconjugated bilirubin is 6, direct is 0, no blood in
urine. Answer is hemolytic jaundice, work-up involves finding cause of red cell hemolysis.
* 19yo college student returns from a trip to Cancun, two weeks later develops malaise, weakness, and anorexia.
One week later he notices jaundice. Total bilirubin is 12, 7 indirect, 5 direct, ALP is mildly elevated, SGOT and
SGPT is very high. Answer is hepatocellular jaundice, work-up involves determining which hepatitis he has.
* Patient with progressive jaundice over four weeks has total bilirubin of 22 with 16 direct and 6 indirect with
minimally elevated SGOT. ALP was twice normal value a couple weeks ago, now he is six times above. Answer is
obstructive jaundice. Answer for first step is sonogram to find dilated intrahepatic (and possibly extrahepatic) ducts.
Sonogram can also be helpful in determining stones versus cancer (dilated thin-walled gallbladder, Courvoisier
sign), common duct stone may not be seen (50% accuracy) but stones sitting in the contracted thick-walled
gallbladder likely will.
* 40yo obese mother of five children has progressive jaundice over four weeks, has total bilirubin of 22 with 16
direct and 6 indirect with minimally elevated SGOT. ALP is about six times normal upper limit. Has history of
multiple episodes of colicky RUQ pain after eating fatty foods. Answer is gallstones, cholelithiasis. Answer for next

DO NOT DISTRIBUTE - 36 -
Study Notes – Surgery James Lamberg 28Jul2010

step is sonogram, showing dilated intrahepatic ducts and thick-walled gallbladder. Answer for next diagnosis step is
endoscopic retrograde cholangiopancreatography (ERCP), which can also allow for removal of the stone. Answer
for management now is cholecystectomy to prevent future stones.
* 66yo man presents with six weeks progressive jaundice, has total bilirubin of 22 with 16 direct and 6 indirect with
minimally elevated SGOT. ALP is about six times normal upper limit. He has lost ten pounds over the past few
months but is otherwise asymptomatic. Sonogram shows dilated intrahepatic ducts, dilated extrahepatic ducts, and
very distended thin-walled gallbladder. Answer is malignancy at head of pancreas (adenocarcinoma),
cholangiocarcinoma in lumen of common duct, or adenocarcinoma at the ampulla vater. Answer for next step is CT
scan (non-invasive). Answer for management if positive CT scan is percutaneous biopsy. Answer for management if
negative CT scan is ERCP.
* 66yo man presents with six weeks progressive jaundice, has total bilirubin of 22 with 16 direct and 6 indirect with
minimally elevated SGOT. ALP is about six times normal upper limit. He is otherwise asymptomatic. Sonogram
shows dilated intrahepatic ducts, dilated extrahepatic ducts, and very distended thin-walled gallbladder. CT scan is
unremarkable. ERCP shows narrow area in distal common duct and normal pancreatic duct. Answer is
cholangiocarcinoma at the distal common duct, maybe showing an apple-core appearance. Answer for diagnosis is
brushings and cytology. Answer for treatment is pancreaticoduodenectomy (Whipple procedure), which has high
survival. Pancreatic tissue tumors do not have high survival with Whipple however.
* 64yo woman presents with progressive jaundice she noticed two weeks ago, total bilirubin of 12, 8 direct, 4 direct,
minimally elevated SGOT, ALP ten times upper limit of normal. She is otherwise asymptomatic but is found to be
slightly anemic and to have positive occult blood in the stool. Sonogram shows dilated intrahepatic ducts, dilated
extrahepatic ducts, and very distended thin-walled gallbladder. Answer is not pancreatic head cancer as well as
cecum cancer; find a single explanation for both. Answer is ampullary cancer, adenocarcinoma growing in the
lumen of the duodenum, which can bleed. Answer is not CT scan because the cancer would be too small to see.
Answer is endoscopy up to ampulla vater, not ERCP. Answer for treatment is pancreaticoduodenectomy.
* 56yo man presents with progressive jaundice over six weeks, total bilirubin of 22, 16 direct, 6 indirect, minimally
elevated SGOT, ALP eight times upper limit of normal. He lost 20lbs over past two months and has persistent
nagging pain in the deep epigastrium and upper back, sister died at age 44 of pancreatic cancer. Sonogram shows
dilated intrahepatic ducts, dilated extrahepatic ducts, and very distended thin-walled gallbladder. Answer is cancer
of the head of the pancreas. Answer for next step is CT scan to see cancer, then percutaneous biopsy, then likely
only palliative treatment and not Whipple.
--------------------------------------------------------------------------------------------------------------------------------------------
General Surgery: Biliary Tract Disease
* Asymptomatic gallstones have a very low rate of conversion to symptomatic, about 2%, so they are left alone.
* Colicky pain due to stone, worse with fatty foods, CT scan shows stones, gallbladder wall thickening, and
pericholecystic fluid. Start with antibiotics then elective cholecystectomy. 15% will not get better with medical
management, with males and diabetics most likely to fail medical management.
* If gallstone impacts past cystic duct to common duct and produces partial obstruction allowing for acute ascending
cholangitis, this can be deadly. These are older patients, fever high (e.g. 104F), and even chills. Will have a huge
elevation of alkaline phosphatase, which is produced in the lining of the hepatic ducts. Obstruction increase ALP but
pus really increases ALP. Answer for next step is ERCP to emergency decompress the duct.
* White obese 40yo mother of five children has RUQ pain with fatty foods and relieved with anticholinergic
medications. Pain is intermittent in RUQ and goes to right shoulder and back with nausea and vomiting. This is
biliary colic. Answer for next step is sonogram. Answer for management is elective cholecystectomy.
* 43yo obese mother of six children has severe RUQ pain that began six hours ago, was colicky at first radiating to
right shoulder. For past two hours the pain has been constant, has tenderness, muscle guarding, and rebound. Temp
is 101, WBC 12000, normal liver functions. This is acute cholecystitis. Answer for next step is sonogram, hopefully
showing stones in gallbladder, thickened gallbladder wall, and pericholecystic fluid. If sonogram is inconclusive, we
could do an hepatobiliary iminodiacetic acid (HIDA) scan, which radioactively labels bile. Cholecystokinin (CCK)
may be given during a HIDA scan to show gallbladder contraction. IDA scan would show radioactive label
everywhere except gallbladder in acute cholecystitis. Answer for next step is IV fluids, NPO, antibiotics, then
eventually elective cholecystectomy.
* 73yo obese mother of six children has severe RUQ pain that began three days ago, was colicky at first but constant
over last two days, has tenderness, guarding, and rebound. Temp spikes to 104 and 105, chills, WBC 22000 with left
shift, bilirubin 5, ALP 2000 (20 times normal). Answer is acute ascending cholangitis. Answer for management is
hospitalization, antibiotics, and emergency decompression of biliary tract preferably by ERCP.

DO NOT DISTRIBUTE - 37 -
Study Notes – Surgery James Lamberg 28Jul2010

* White obese 40yo mother of five children has repeated episodes of RUQ pain with fatty foods and relieved with
anticholinergics. Pain is colicky and radiates to right shoulder and to back with nausea and vomiting. This time she
has chills with colicky pain lasting longer than usual. Mild tenderness to palpation at RUQ, bilirubin 3.5, ALP five
times normal, amylase three times normal. Answer is biliary pancreatitis, stone stuck at ampulla vater. Most of these
patients pass the stone. Next step is sonogram. Management is watchful waiting, IV fluids, antibiotics, NPO. In the
rare case that the pancreatitis doesn’t get better, amylase and bilirubin get higher, then emergency ERCP with
removal of the stone.
--------------------------------------------------------------------------------------------------------------------------------------------
General Surgery: Pancreas
* 33yo alcoholic man arrives in ED with 12 hours of mid-epigastric pain beginning after a large meal. The pain is
constant, severe, radiates to back, vomited twice, retching, tenderness, muscle guarding, afebrile, mild tachycardia,
serum amylase is 1200, hematocrit is 52. Choice of amylase versus lipase isn’t extremely important, lipase may be
more specific, amylase has been used for many years. The choice between serum and urinary amylase/lipase
depends on time, serum is for within 12hours or so, urinary is for two days out up to 4-5 days. To determine acute
edematous (benign) versus hemorrhagic (less common, devastating), we look at hematocrit. Hematocrit of 52 is
indicative of plasma leaving blood and concentrated hematocrit is left, so acute edematous pancreatitis. Management
is rest, NPO, NG suction, IV fluids, wait for pancreas to recover.
* 56yo alcoholic man is admitted with the clinical picture of acute upper abdominal pain. Pain is severe, constant,
radiates to back, serum amylase is 800, hematocrit is 40, WBC 18000, glucose 150, calcium 6.5, given IV fluids,
NPO, NG suction. Next morning hematocrit is 30, calcium is low despite being given calcium, BUN up to 32,
metabolic acidosis, low arterial PO2. Answer is hemorrhagic pancreatitis. Ranson’s criteria for prognosis of
pancreatitis would put this patient at 100% chance of dying. Ranson’s criteria here at admission are low hematocrit,
elevated WBC, elevated blood glucose, low calcium, and by next morning that hematocrit is lower, calcium still
low, BUN up, and PaO2. Final pathway leading to death is often pancreatic abscesses, so we must recognize these
immediately and drain. Answer for next step is ICU admission and daily CT scans.
* 57yo alcoholic man is being treated for acute hemorrhagic pancreatitis, was in the ICU for one week, required
chest tubes for pleural effusion, was on a respirator for several days, eventually improved enough to be transferred
to the floor. Two weeks after initial disease, has fever spikes and leukocytosis. Answer is pancreatic abscess.
Answer for next step is confirm with CT scan and management is percutaneous drainage.
* 49yo alcoholic man presents with ill-defined upper abdominal discomfort and early satiety. Physical exam shows
large upper abdominal mass that is deep in the abdomen and hard to define. He was discharged from the hospital
five weeks ago for successful treatment of pancreatitis. Answer is pancreatic pseudo-cyst secondary to disruption of
a pancreatic duct. Pancreatic juice/cyst collects in lesser sac. Answer for confirmation is sonogram or CT scan.
Management is based on possible outcomes and most that go away by themselves occurs within first six weeks.
Complications occur after six weeks, so for the first few weeks the answer is careful observation with repeated
sonogram or CT scans. If we’re at six weeks and the cyst is still there, then we intervene. Answer is endoscopic
(gastroscopy) internal cystogastrostomy. Other options include percutaneous drainage (interventional radiology).
* 55yo woman presents with vague upper abdominal discomfort, early satiety, and large ill-defined abdominal mass.
Five weeks later she was involved in a MVC where she hit the upper abdomen against the steering wheel. Answer is
pancreatic pseudo-cyst. Answer for confirmation is sonogram or CT scan. Management is the same as above.
* Disheveled malnourished individual shows up in the ED requesting pain medication. He smells of alcohol and
complains bitterly of constant epigastric pain radiating to the back that he says he had for several years. He has
diabetes, steatorrhea, and calcifications in the upper abdomen in plain x-ray. Answer is chronic pancreatitis. We
would not expect amylase to be useful here as the pancreas is burnt out. These patients are typically a wreck, lost
their jobs, on the streets, plagued by constant pain, are known in the ED for seeking pain medications. Answer for
management is attempt to control diabetes and give pancreatic enzymes to control steatorrhea. Answer for next step
if therapy is being suggested is an ERCP. Even a pancreatectomy doesn’t take care of this problem, as these patients
usually cannot control the difficult diabetes created by the pancreatectomy.
--------------------------------------------------------------------------------------------------------------------------------------------
General Surgery: Hernias
* Standard recommendation for any abdominal hernia is elective repair to prevent future strangulation.
* 9month old baby girl is brought in with an umbilical hernia. Defect is 1cm in diameter and contents are freely
reducible. Specifically, umbilical hernias below age 2 may close on their own, so this is a case where elective repair
is not warranted.
* 18yo man has a routine physical exam for college admission and a right inguinal hernia is found. External inguinal
ring is about 2.5cm in diameter, hernia bulge is felt in scrotum when he strains. Answer is elective repair.

DO NOT DISTRIBUTE - 38 -
Study Notes – Surgery James Lamberg 28Jul2010

* 72yo farmer is forced by his insurance company to have a physical exam to be issued a life insurance policy. He
has been healthy his whole life and never been to the doctor. Physical exam shows large left inguinal hernia
reaching down to hernia. Bowel sounds can be heard over it. Hernia is not reducible, he use to be able to push it
back, but not for the last 10-20years. Normally, if the hernia is irreducible it should be an emergency surgery. In this
case, the answer is elective surgery.
* The presence of a sliding hiatal hernia is not an indication for surgery. Parahiatal esophageal hernias are though.
--------------------------------------------------------------------------------------------------------------------------------------------
General Surgery: Breast Disease
* Answer for management should always begin with ruling out cancer, meaning only via the pathologist. Most
important predictor for cancer of the breast and thus treatment is age. So non-invasive tests for younger patients. As
a note, mammography is a screening tool, not a diagnostic tool, but it can help find other areas for scrutiny if we find
a breast mass. FNA (fine needle aspiration) is not considered invasive. Contraindications for mammogram include
age below 20 years and lactating woman. Pregnancy is not a contraindication.
* 18yo woman has firm rubbery mass in left breast, moves easily with palpation. Answer is fibroadenoma. Answer
for management is FNA or sonogram, then removal at the option of the patient.
* 14yo girl has a firm moveable rubbery mass in left breast felt one year ago, now is 6cm in diameter. Answer is
giant juvenile fibroadenoma. Answer for management is recommend removal.
* 27yo immigrant from Mexico has a 12x10x7cm mass in her left breast. Present for 7 years, slowly growing to
current size. Mass is firm, rubbery, completely movable, not attached to chest wall or overlying skin, no palpable
axillary nodes. Answer is cystosarcoma phyllodes. It is benign with malignant potential to sarcoma. FNA is not
sufficient. Answer requires tissue diagnosis (biopsy) and management is removal.
* 35yo woman has 10-year history of tenderness in both breasts related to menstrual cycle, multiple lumps in both
breasts that come and go. She has firm, round, 2cm mass that has not gone away for six weeks. Answer is fibrocystic
disease (mammary dysplasia) and next step is mammogram. However, there is a mass that hasn’t gone away.
Answer now is likely cyst but could be a tumor/cancer, so we do a mammogram then aspiration of the suspected
cyst, not an FNA, if fluid is clear then we’re done. If we aspirate and get bloody fluid then it goes to cytology. If we
aspirate and the mass stays or comes back in a day or two, we do tissue sampling via biopsy.
* 34yo woman has been having bloody discharge from the right nipple on and off for several months. Answer is
intraductal papilloma, a small 2-3mm benign tumor in young woman, but it could be carcinoma. So, answer for next
step is mammogram. If lesion found, we biopsy and proceed. Intraductal papilloma is likely not seen so we would
then do a galactogram or retroareolar surgical exploration then remove that segment of the breast to stop discharge.
* 26yo lactating mother has cracks in the nipple and develops a fluctuating red, hot tender mass with a fever and
leukocytosis. Answer is breast abscess, seen only in lactating women. Answer for next step is not mammogram
since patient is lactating. Answer is operating room incision and drainage and a biopsy of the wall of the abscess to
rule out infected cancer.
* 49yo woman has firm 2cm mass in breast present for two months. Answer for next step is mammogram to confirm
sole area, then do tissue sampling. We could begin with an FNA, but if negative you still need the core biopsy.
* 34yo woman in her fifth month of pregnancy has 3cm firm ill-defined mass in her right breast that has been
growing for three months. Answer for next step is mammogram (shield fetus) and tissue sampling. Only two
limitations are no chemo in first trimester of pregnancy and no radiation therapy to the breast at any point. It is not
necessary to terminate the pregnancy as it does not increase cancer growth.
* 69yo woman has 4cm hard mass in right breast with ill-defined borders, movable from the chest wall but not
moveable within the breast, skin overlying the mass is retracted and has orange-peel appearance. Answer is cancer
of the breast.
* 69yo woman has 4cm hard mass in right breast with ill-defined borders, movable from the chest wall but not
moveable within the breast, nipple became retracted six months ago (desmoplastic reaction, but note some women
have retracted nipples all their life). Answer for next step is mammogram, tissue sample, and don’t stop until you
have generous tissue for testing.
* 72yo woman has red swollen breast, skin over the area looks like orange peel, she is not particularly tender and
unsure if the area is hot or not, no fever, no leukocytosis. Answer is inflammatory cancer of the breast. Next step is
mammogram, tissue sampling (skin would have cancer cells too).
* 62yo woman has eczematoid lesion in the areola for three months, has not gone away with a variety of skin
products. Answer is Paget disease, breast cancer. Answer for next step is mammogram and biopsy.
* 42yo woman hits her breast with a broom handle while doing housework. She noticed a lump in the area at that
time and one week later it is still there. 3cm hard mass deep in breast and superficial overlying ecchymosis. History

DO NOT DISTRIBUTE - 39 -
Study Notes – Surgery James Lamberg 28Jul2010

of trauma does not exclude the history of cancer. She could have hit her breast, felt the area, and then felt a lump
that was originally there. Answer for next step is mammogram and then tissue sampling.
* 58yo woman discovers a mass in her right axilla, discrete hard moveable 2cm mass, breast physical exam is
negative, no enlarged nodes elsewhere. If she were 22yo with other nodes in neck then you’d think lymphoma.
Answer in this case is breast cancer, metastatic to the axilla. Primary is too small to palpate, which is why
mammography is important. Answer for next step is mammography and biopsy of axillary node.
* 60yo woman has routine screening mammogram, radiologist reports irregular area of increased density with fine
microcalcifications not present two years ago in previous mammogram. Answer for next step is radiologically-
guided core biopsies.
* 44yo woman has 2cm palpable mass in upper outer quadrant of right breast. Core biopsy shows infiltrating ductal
carcinoma (most common). Mass is freely moveable, breast is of normal size. No nodes, mammogram shows no
other lesions. Answer for management for small cancer in large breast, far away from nipple/areola, then
lumpectomy with axillary sampling and radiation to remaining breast that was operated on. Next option if small
breast, or near nipple, then modified radical mastectomy with axillary sampling (not dissection) via sentinel node
biopsy, meaning injection of radioactive substance then sampling of the first node. Negative palpation of the axilla
has at best 50% accuracy.
* 62yo woman has a 4cm hard mass under the nipple and areola in her smallish left breast. Core biopsy diagnosis
infiltrating ductal carcinoma, no other lesions on mammogram, no axillary nodes. Answer for management is
modified radical mastectomy. No need for post-op radiation.
* 44yo woman has 2cm palpable mass in the upper outer quadrant of the right breast, core biopsy shows lobular
cancer of the breast (or medullary cancer). Note lobular cancer has high incidence of bilaterality, but not high
enough to required a bilateral mammogram. Management for these is the same as infiltrating ductal.
* 44yo woman has 2cm palpable mass in the upper outer quadrant of the right breast, core biopsy shows
inflammatory cancer of the breast. In this case, answer is do radiation and chemotherapy first, then attempt surgical
resection.
* 52yo woman has suspicious area on mammogram, multiple radiologically guided biopsies show infiltrating ductal
carcinoma in situ. In situ meaning not capable of metastasis here, so we don’t need to go to the axillary nodes. One
answer for management is simple mastectomy as there is high risk of recurrence. Another answer is lumpectomy
and radiation therapy, like the infiltrating ductal carcinoma.
* 30yo woman in the seventh month of pregnancy has 2cm mass in breast, core biopsy shows infiltrating ductal
carcinoma. Answer for next step is lumpectomy or modified radical mastectomy. However, if we do lumpectomy
then the radiation must wait until after the pregnancy.
* 44yo woman shows up in the ED because she is bleeding from the breast, examination shows huge fungating
breast mass occupying the entire right breast and firmly attached to chest wall, patient says it has only been there for
two weeks, relative says it has been there for at least two years. Tragic case of neglect and denial. Answer is breast
cancer. Answer for next step is biopsy. Answer for management is chemotherapy and radiation, even though it is
clearly inoperable.
* 37yo woman has lumpectomy and axillary dissection for 3cm infiltrating ductal carcinoma, pathologist notes clear
margins and metastatic cancer in four of seventeen axillary nodes. Answer is cancer with systemic disease. Answer
for management is systemic treatment, general rule is if patient has positive axillary nodes then we do systemic
therapy, if pre-menopausal we do chemotherapy, if post-menopausal we do hormonal therapy (tamoxifen), and
chemo therapy for any distant metastasis. Answer in this case is chemotherapy.
* 66yo woman has a modified radical mastectomy for infiltrating ductal carcinoma, pathology reports 4cm mass
with seven of twenty-two nodes positive for metastasis, tumor is estrogen and progesterone positive. Answer is
tamoxifen.
* 61yo woman has lumpectomy and axillary dissection for infiltrating ductal carcinoma of the breast, 7 of 22 nodes
are positive for metastasis, liver and bone metastasis are found. Answer is chemotherapy.
* 44yo woman complains bitterly of severe headaches for several weeks that have not responded to OTC meds. She
is two years post-op for modified radical mastectomy for T3N2M0 cancer of the breast with several courses of post-
op chemotherapy but she discontinued because of the side effects. Answer for next step is CT scan of head, likely
showing brain metastasis. Resect if they are resectable, else treat some other way. For TNM classification, just know
T is for size from 0-4, N is nodal metastasis from 0-2, and M is metastasis either 0 or 1.
* 39yo woman completed her last course of post-op adjuvant chemotherapy for breast cancer. Comes to clinic with
constant back pain for three weeks, tender to palpation over well-circumscribed areas in thoracic and lumber spine.
Assume bony metastasis, typically to pedicle of the vertebra. Answer for next step is not x-ray, it is radionuclide

DO NOT DISTRIBUTE - 40 -
Study Notes – Surgery James Lamberg 28Jul2010

bone scan. Bone scan is most specific but not sensitive, so negative rules out, but if positive then we need x-rays to
rule-out other things that light-up the bone scan, like arthritis or old fracture.
--------------------------------------------------------------------------------------------------------------------------------------------
General Surgery: Endocrine Disease
* 62yo woman was drinking her morning cup of coffee while applying her makeup, she noticed a lump in her lower
neck as she swallowed. Exam shows dominant 2cm left thyroid mass as well as two smaller masses on the right
lobe. They are all soft, no lymph nodes. Note that no hyperthyroid symptoms are mentioned, like tachycardia, heat
intolerance, or other signs of a hot/hyperfunctional node. Answer for next step is FNA. If FNA is negative, leave
lady alone. If tumor, operate. If indeterminate, operate (thyroidectomy or lumpectomy). Most thyroid masses are
benign, thus we don’t want to operate for benign disease.
* 21yo man found on routine physical to have single 2cm nodule in thyroid gland, had head and neck radiation at
age 13 for persistent acne, thyroid function tests normal, FNA is indeterminate. Answer for next step is operation.
Concerning factors for cancer are younger patients, male, single lump, previous radiation.
* 33yo woman has palpable mass in thyroid gland, describes weight loss despite ravenous appetite, palpitations, heat
intolerance. She is thin, constantly moving, moist skin, pulse of 105. Answer is hyperthyroidism, could be hot
adenoma in neck. Answer for next step is T4 (free T4) or TSH, with high T4 and low TSH. Answer for test is
radioactive iodine scan to see if mass is hyperfunctioning (“hot”). Remove hot adenoma if that is the case, else if the
thyroid itself is hyperfunctioning we can treat with radioactive iodine.
* 22yo man has 2cm round, firm mass in lateral aspect of neck, present for 4 months. Clinically this is assumed to be
a large jugular lymph node and it is removed surgically. Pathologist reports the tissue is normal thyroid tissue.
Thyroid migration is only in the midline, so if it is on the lateral side of the neck it is metastasis of a follicular cancer
of the thyroid, replacing a lymph node. Follicular cancer of the thyroid is difficult to recognize pathologically from a
small sample. Answer for next step is thyroid scan to identify primary, then surgery.
* Automated blood chemistry panel done during the course of a routine medical exam, indicates that an
asymptomatic patient has a serum calcium of 12.1 where upper limit of normal is 9.5 Repeated determinations are
between 10.6 and 12.5, serum phosphorus is low. Answer is parathyroid adenoma, which today is found in
asymptomatic patients mostly during routine screening. Symptomatic patients have “stones, bones, groans” for
kidney stones, bone pain from cystic degeneration, and GI complaints causing groans. There is about a 20% rate of
cancer conversion here, so we treat asymptomatic patients. Answer for diagnosis is PTH (parathyroid) determination
read in conjunction with the serum calcium. So, high PTH and high calcium is confirmation. 90% are adenoma
rather than hyperplasia. Answer for localization is sonogram, CT scan, or radionuclide scan (Technetium-99m
sestamibi scan) prior to surgical removal.
* Picture of pretty lady then picture next to it of “monster”. Question says it is the same patient a year later. There is
truncal obesity, thin spider extremities, abdominal wall striae, round face with hair and pimples, buffalo neck hump.
Answer is Cushing disease. Other manifestations are HTN, osteoporosis, diabetes, amenorrhea, wide mood swings.
Answer for next step is measure cortisol at AM and PM. Then, dexamethasone suppression test. Suppress patient
with small dose of dexamethasone to suppress the adrenal gland, if suppressed at small dose then no Cushing. If she
does not suppress at low dose, it is Cushing but maybe pituitary adenoma or adrenal gland adenoma. Now we do the
suppression at a higher dose, if suppressed it is micro-adenoma of the pituitary associated with large adrenal glands.
If no suppression even at high dose, the problem is in the adrenal gland itself or possibly paraneoplastic syndrome.
Answer depending on finding is pituitary MRI scan or adrenal CT scan, then surgical removal of tumor.
* 28yo woman has virulent peptic ulcer disease. Extensive management including eradication of H. pylori fails to
heal her ulcers in the first and second portions of the duodenum. She has watery diarrhea. Answer is Zollinger-
Ellison syndrome (ZES), gastrinoma. Answer for next step is measure serum gastrin, later do CT scan of pancreas to
see primary tumor, eventually resection. Octreotide may be used for symptomatic relief.
* 3rd year medical student is hospitalized for a neurological workup for a seizure disorder of recent onset. During
one of the convulsions it is determined that his blood sugar is very low. Further workup shows high levels of insulin
in the blood with low levels of C-peptide. For neurological symptoms, we are to believe that hypoglycemia is the
cause. DDx is insulinoma, reactive hypoglycemia, or insulin injection. Insulinoma would have attacks occurring
during fasting, such as skipping breakfast or late for lunch. If reactive hypoglycemia, attacks are after big meal when
pancreas over-reacts. Insulin injector often knows what insulin does, like a nurse, physician, medical student, or
relative or a diabetic. Answer is self-administration of insulin, as C-peptide is not elevated with insulin. Answer for
next step is not CT of pancreas, it is referral to psychiatrist.
* 48yo woman with severe migratory necrolytic dermatitis for several years, unresponsive to all kinds of herbs and
oils, she is thin, mild stomatitis, mild diabetes mellitus. Answer is glucagonoma. Answer for next step is measure
glucagon, CT scan of pancreas to see tumor, resect tumor.

DO NOT DISTRIBUTE - 41 -
Study Notes – Surgery James Lamberg 28Jul2010

--------------------------------------------------------------------------------------------------------------------------------------------
General Surgery: Hypertension
* 45yo woman comes in for regular check-up, repeated exams confirm hypertension, BP 3 years ago was normal,
lab studies now show Na 144, Bicarb 28, K 2.8, on no medications. Answer is hyperaldosteronism. Most commonly,
it would be a hypertensive patient on diuretics, but here we specify that the patient is not on medications. Answer for
next step is measure aldosterone and renin, looking for high aldosterone with low renin. Answer if hyperplasia is
medical management, answer if adenoma is surgical management. Do aldosterone and renin tests with patient lying
down and with patient standing up. Physiologic is higher aldosterone standing up, which would be associated with
hyperplasia. If not normal or lower aldosterone standing, likely adenoma. Next step is CT scan and surgical removal.
* Thin hyperactive 38yo woman is frustrated with her physician’s inability to treat her bouts of severe pounding
headaches with profuse palpitations and pallor. By the time she makes it to the doctor’s office, she feels normal.
Answer is pheochromocytoma with paroxysmal hypertension. Could be persistent hypertension, but this variant
makes the patient feel like they are going to die. Answer for next step is 24hour urinary collection for metanephrine
or vanillylmandelic acid (VMA), VMA is easier to measure, not all labs measure metanephrine but it’s better.
Follow with adrenal gland CT scan, expecting a big mass like 5-6cm. Anesthesiologist is very important during this
surgery as they must pharmacologically block the catecholamine release from the tumor, such as with phentolamine
(alpha1 blocker).
* 17yo man is found to have a BP of 190/115, checked repeatedly in both arms, found to be normal in the limbs.
Answer is coarctation of the aorta. Answer for next step of management is chest x-ray, seeing collateral circulation
causing rib scalloping or rib notching. Answer for diagnosis is spiral CT scan, better than transesophageal
echocardiogram and MRI.
* 23yo woman has had severe hypertension for two years, unresponsive to usual medical treatment. Bruit can be
faintly heard over upper abdomen. Answer is renal vascular hypertension, due to fibromuscular dysplasia. Answer
for next step is noninvasive Doppler study or duplex scan, moving to arteriogram, and finally to dilation with a stent.
* 72yo man with multiple manifestation of atherosclerotic occlusive disease, has hypertension of relatively recent
onset, refractory to usual treatment. Faint bruit over abdominal flank. Answer is renal vascular hypertension, due to
plaque. Answer for next step is possibly nothing in an older individual as they likely have several other underlying
problems. There is debate of if a workup is necessary and which workup would be used.
--------------------------------------------------------------------------------------------------------------------------------------------
General Surgery: Pediatrics
* Within 8 hours after birth, it is noted that a baby has excessive salivation. Answer is esophageal atresia, likely
tracheoesophageal fistula. Could also say there is a choking episode with first feeding. A small soft nasogastric tube
is inserted and the child is taken to x-ray to have a “babygram” done; called a babygram because the child is the size
of the x-ray plate so they shield the gonads and do an x-ray of the entire baby. Film shows the NG tube coiled back
upon itself in the upper chest. There is air in the GI tract. Answer is blind pouch at upper end of the esophagus and a
fistula between tracheoesophageal tree and lower fistula. Answer for next step is not immediate operation. This is
one sign of a constellation of symptoms that exist, so we must rule out the others. Mnemonic is VACTRR, vertebral
anomalies, anal anomalies, cardiac anomalies, tracheoesophageal fistula, and renal and radial anomalies. So next
step is examine radial bones, examine vertebrae, examine for anal imperforation, sonogram, and echocardiogram. If
no other anomalies preventing fistula prevention, repair is done at first day of life. If other anomalies exist, we place
a gastrostomy tube.
* A newborn baby is found on physical exam to have an imperforate anus. Answer is not go to OR. Answer for next
step is insert a soft NG tube, babygram to see if tube is in stomach, sonogram for renal anomalies, echocardiogram
for heart, x-ray for bony anomalies. If no other anomalies, is there an anal fistula nearby. If a fistula exists such as to
the vagina, we do not need emergency surgery. If no fistula exists, we need to know how high up the blind pouch is.
This is done via x-ray with the child upside down, little marker taped to anus, measure distance from gas to marker.
If gap is short, we can do simple repair immediately. If gap is large, might opt for diverting colostomy with repair at
a later, more developed date, before toilet training.
* Newborn baby is tachypneic, cyanotic, abdomen scaphoid, bowel sounds over left chest. X-ray shows bowel in left
thorax. Shortly later the baby has hypoxia and aspirates. Answer is congenital diaphragmatic hernia, always on the
left side. This is a problem as the lung on the left is hypoplastic, thus answer is not go to the OR and fix diaphragm.
Answer is wait 2-3 days for pulmonary maturation, give extensive respiratory support, put on respirator with care
taken to prevent over-inflation. Most likely, this mother had no prenatal care else this would be diagnosed in utero.
If diagnosed in utero, planning could be made and birth could occur at a location with ECMO (extra-corporeal
membrane oxygenation).

DO NOT DISTRIBUTE - 42 -
Study Notes – Surgery James Lamberg 28Jul2010

* At the time of birth, it is noted that a child has a large abdominal wall defect to the right of the umbilicus. There is
a normal cord, but protruding from the defect is a matted mass of angry-looking edematous bowel loops. Answer is
gastroschisis, defect with no membrane protecting it. Answer is go to OR and create a silastic container to house and
protect the bowel (looks like a silo), gradually squeezing the bowel back over a few days. Answer also includes
providing vascular access for long-term parenteral nutrition.
* A newborn baby is noted to have a shiny thin membranous sac at the base of the umbilical cord, with the cord
going to the sac and not the baby directly. Inside the sac one can see part of the liver and loose abnormal bowel.
Answer is omphalocele. Answer for next step is create silastic silo and move bowel in over a few days.
* Newborn is noted to have a moist medallion of mucosa occupying the lower abdominal wall, above the pubis and
below the umbilicus. It is clear that urine is constantly bathing this congenital anomaly. Answer is exstrophy of the
urinary bladder. Associated with epispadias. Answer for management is repair within the first two days of life,
possibly transporting to a highly specialized facility to perform this.
* 30mins after first feed, baby vomits greenish fluid. Normally, vomiting is white from babies. Mother has
polyhydramnios and baby has Down syndrome. X-ray shows a double-bubble sign, with large air-fluid level in
stomach and smaller level in duodenum. No gas can be seen in the rest of the bowel. Answer could be three
conditions, duodenal atresia, annular pancreas, or malrotation. Answer is barium enema (or swallow). If not
malrotation, operate without urgency.
* If baby has green vomiting after first feed, double-bubble sign, and gas in small loops of distal bowel, answer is
malrotation. This is rotationally the most devastating of the three (duodenal atresia and annular pancreas). Answer to
diagnose malrotation is barium enema or barium swallow. Barium enema is safer, but less diagnostic, likely the
better answer as it has less risk. If malrotation diagnosed, operate immediately.
* Newborn baby has repeated green vomiting during first day of life and passes no meconium. Baby is normal
except for distension. X-ray shows multiple air-fluid levels and distended loops of bowel. Answer is intestinal
atresia. This occurs via vascular accident in utero, so no need to search for other congenital anomolies. Answer is to
operate to fix this.
* Very premature baby develops feeding intolerance, abdominal distention, and rapidly dropping platelet count.
Baby is 4yo and was treated with indomethacin for a patent ductus. Answer is necrotizing enterocolitis. Answer for
next step is stop feedings, intravenous broad-spectrum antibiotics, and surgical intervention only if there are signs of
death of the bowel. Signs of bowel death are abdominal wall erythema, air in biliary tree, and pneumoperitoneum.
* 3-day-old full term baby is brought in for feeding intolerance and bilious vomiting. X-ray shows multiple loops of
dilated small bowel, ground glass appearance in lower abdomen, mother has cystic fibrosis. Answer is meconium
ileus, we also know baby has cystic fibrosis from this. Answer for next step is gastrografin enema to make diagnosis
by showing unused microcolon and pellets of meconium. Gastrografin is a hypertonic solution, drawing fluid into
lumen, so it can help dissolve the meconium pellets, thus it is also therapeutic.
* 3-week-old baby has trouble feeding, is not growing well, has bilious vomiting. X-ray shows double-bubble with
normal gas pattern in rest of the bowel. Answer is malrotation. Answer is barium enema then surgery.
* Three-week-old baby, first born baby boy, full term, vomited three days ago and is projectile. Projectile vomiting
in adults is a sign of brain tumor. Projective vomiting at age three weeks is hypertrophic pyloric stenosis. Vomiting
here has no bile, follows each feeding, and baby is hungry and eager to keep eating, looks dehydrated, has visible
gastric peristaltic waves, and palpable olive-sized mass in right upper quadrant (the is the hypertrophic pylorus). No
more diagnostic tests are needed, but if one was needed pick sonogram and not barium. Answer for treatment is not
surgical first, answer is fix hypochloremic hypokalemic metabolic alkalosis and dehydration. Answer after that is
operation, Fredet-Ramstedt pyloromyotomy.
* 8-week-old baby brought in because of persistent, progressive jaundice. Bilirubin is significantly elevated, about
2/3rds is conjugated direct bilirubin, serology negative for hepatitis, sweat test is normal. Answer is biliary atresia.
Answer for diagnostic test is IDA radionuclide scan done one week after phenobarbital therapy. Phenobarbital is a
powerful choleretic, stimulating liver to create bile. IDA scan showing material only in liver is diagnostic for atresia.
If bile if found in liver, ducts, and duodenum, it is not atresia. Answer is operate, and we find during surgery that 1/3
cannot be operated and required liver transplant. Of the 2/3 that can be operated on, half remain well after the repair,
so another 1/3 of the original require transplant.
* 2-month-old baby boy brought in because of chronic constipation, has distended abdomen, x-ray shows gas
through dilated loops of bowel in the abdomen. Rectal exam is followed by explosive expulsion of stool and flatus,
with remarkable improvement to the distension. Answer is aganglionic megacolon (Hirschsprung disease). Answer
for diagnosis is full-thickness biopsy of the rectal mucosa.
* If kid has chronic constipation, goes to bathroom once a week, rectal exam shows rectal vault full of feces,
episodes of fecal soiling, answer is likely psychological problem and not aganglionic megacolon.

DO NOT DISTRIBUTE - 43 -
Study Notes – Surgery James Lamberg 28Jul2010

* 9-month-old chubby healthy-looking boy who has episode of colicky abdominal pain that make him double-up
and squat. The pain lasts for one minute and kid looks happy until he gets another colic. Physical exam shows a
vague mass on the right abdomen, empty right lower quadrant, and current jelly stools. Answer is intussusception.
Answer for next step is contrast enema, which is also therapeutic in most cases.
--------------------------------------------------------------------------------------------------------------------------------------------
Cardiothoracic Surgery: Congenital Heart Disease
* 6-month-old baby has occasional stridor and episodes of respiratory distress with crowing respiration in which he
assumes a hyperextended position. The family notes he has difficulty swallowing. If just respiratory distress with
stridor and crowing only, answer is tracheomalacia. Here there is difficulty swallowing, so answer is vascular ring,
of the aorta around the trachea and esophagus. Answer for next step is barium swallow, showing extrinsic
compression from the abnormal vessel. Bronchoscopy is done next to confirm segmental tracheal compression and
also rules out tracheomalacia. Answer for treatment is surgical repair to divide the smaller of the two aortic arches.
* Patient known to have a congenital heart defect with a hemodynamically non-significant shunt requires extensive
dental work. Any morphological abnormality of the heart is a weak point where bacteria can setup shop. Answer is
prophylactic antibiotics for this patient.
* Non-cyanotic heart defects are left to right, cyanotic heart defects are right to left (less oxygen in circulation).
There will always be a heart murmur, will be increased or decreased pulmonary vascular markings, and will include
respiratory problems such as congestion or infections.
* During a school physical exam, a 12-year-old girl is found to have a heart murmur. A cardiology fellow notes a
pulmonary flow systolic murmur and a fixed split second heart sound. A history of frequent colds and upper
respiratory infections is elicited. Answer is atrial septal defect (ASD). Fixed split second heart sound is nearly
diagnostic. Answer for workup of any morphological abnormality is echocardiogram.
* 3-month-old boy is hospitalized for failure to thrive (FTT). He has loud pansystolic murmur best heard at left
sternal border, CXR shows increased pulmonary vascular markings. Answer is ventricular septal defect (VSD). Note
the child is in trouble here by age three months versus the atrial defect going up through grade school. FTT implies
here the child is so busy breathing that he cannot drink much milk. Answer for next step is echocardiogram, then
surgical repair. If left alone, a left-to-right shunt will damage the lungs so it must be repaired.
* Because of a heart murmur, an otherwise asymptomatic 3-month-old baby, is diagnosed with a small restrictive
VSD located low in the muscular septum (assume via echocardiogram). This particular variant can close on its own.
So, answer is not surgical repair, it is watchful waiting, including antibiotics as necessary. If there is a VSD question
with a murmur, where the murmur was not heard at birth, this doesn’t change anything. Due to fetal flow, the
murmur may not be heard on day one.
* 3-day-old premature baby has trouble feeding and pulmonary congestion. Physical exam shows bounding
peripheral pulses and a continuous machinery-like murmur. Shortly after the baby goes into overt heart failure.
Answer is patent ductus arteriosus (PDA). Answer for next step is echocardiogram. For management, can be closed
with indomethacin (“endomethacin ends PDA”), surgical closure, or embolization for closure. Indomethacin
treatment works best for premature baby and also takes time, so it cannot be used if the baby is in congestive failure.
Thus, in this case the answer is surgical division of the patent ductus.
* Premature baby girl has mild pulmonary congestion, signs of increase pulmonary blood flow, wide pulse pressure,
and a precordial machinery-like murmur. Answer is indomethacin.
* For cyanotic heart conditions, there will be decreased vascular markings. Mnemonic is 5Ts, tetralogy of Fallot,
transposition of great vessels, truncus arteriosus, total anomalous pulmonary venous connection, tricuspid atresia.
* 6-year-old boy is brought to the U.S. by new adoptive parents from orphanage in Eastern Europe. The boy is small
for his age and has a bluish hue in his lips and tips of fingers, has clubbing, and spells of cyanosis relieved by
squatting. He has a systolic ejection murmur in left 3rd intercostal space, CXR shows small boot-shaped heart and
diminished vascular markings, ECG shows right ventricular hypertrophy. Answer is tetrology of Fallot (TOF).
Specific location is not important, just an excuse for the chronically neglected health condition. Answer for next step
is echocardiogram and then surgical repair.
* Baby born with cyanosis, 2-days old, looks extremely sick. CXR shows the heart appearing like an egg on a string.
Answer is transposition of the great vessels. Answer for next step is echocardiogram and then surgical repair must
be done within the first few days of life.
--------------------------------------------------------------------------------------------------------------------------------------------
Cardiothoracic Surgery: Acquired Heart Disease
* 72yo man has history of angina and exertional syncopal episodes. Has harsh midsystolic murmur, best heard over
2nd intercostal space and over left sternal border. Answer is aortic stenosis. Answer for next step is echocardiogram.

DO NOT DISTRIBUTE - 44 -
Study Notes – Surgery James Lamberg 28Jul2010

Surgical valvular repair is indicated if there is a gradient of >50mmHg across a valve or at the first indication of
congestive heart failure, angina, or syncope. Answer for repair is a prosthetic valvular replacement.
* 72yo man has been known for years to have a wide pulse pressure and a blowing high-pitched diastolic heart
murmur best heart at 2nd intercostal space and left lower sternal border with expiration. He has had periodic
echocardiograms and the most recent one showed evidence of beginning left ventricular dilatation. Answer is
chronic aortic insufficiency. Indication for intervention is LV dilatation. Answer is aortic valve replacement.
* 26yo drug addicted man develops congestive heart failure over a few days. He has a loud diastolic murmur at right
2nd intercostal space. Physical exam done a few weeks ago when he attempted to enroll in a detoxification program
was completely normal. Answer is acute aortic insufficiency due to endocarditis. Right valves get bacteria first, but
those are less critical to overall health. Here the patient has CHF, so it is left sided. Answer for next step is
emergency valve replacement, IV antibiotics, and continued antibiotics for a long time.
* 35yo lady has dyspnea on exertion, orthopnea, paroxysmal nocturnal dyspnea, cough, and hemoptysis. She has had
these progressive symptoms for 5 years, looks thin, cachectic, atrial fibrillation, and low pitched rumbling diastolic
apical murmur. At age 15 she had rheumatic fever. Answer is mitral stenosis. Answer for next step is
echocardiogram. Answer for management is repair preferred (mitral commissurotomy), replacement if necessary.
* 55yo lady has been known for years to have mitral valve prolapse (MVP), has developed exertional dyspnea,
orthopnea, and atrial fibrillation. Has apical high-pitched holosystolic murmur radiating to axilla and back. Answer
is mitral regurgitation (insufficiency). Most common cause of mitral valve regurgitation and MVP. Answer for next
step is echocardiogram. Answer for management is repair preferred (annuloplasty), replacement if necessary.
--------------------------------------------------------------------------------------------------------------------------------------------
Cardiothoracic Surgery: Coronary Disease
* 55yo man has progressive unstable disabling angina that does not respond to medical management. His father and
two older brothers died of heart attacks before the age of 50. The patient stopped smoking 20 years ago, but is a bit
overweight, has sedentary lifestyle, has type 2 diabetes mellitus, and high cholesterol. Angina is typically handled
medically, but intervention is indicated here as there is unstable angina not responding to medical treatment. Answer
is cardiac catheterization to determine if this patient is a candidate for coronary revascularization.
* 55yo man has progressive unstable disabling angina that does not respond to medical management. His father and
two older brothers died of heart attacks before the age of 50. The patient stopped smoking 20 years ago, but is a bit
overweight, has sedentary lifestyle, has type 2 diabetes mellitus, and high cholesterol. Cardiac catheterization
demonstrates 70% occlusion of three coronary arteries, with good distal vessels, and left ventricular ejection fraction
is 65%. Indication for coronary artery bypass graft (CABG) is >70% occlusion and especially if multiple vessels,
plus has good ventricular function still. So, answer for management is CABG using internal thoracic (internal
mammary) artery for critical vessels and saphenous vein for bypass of the others.
* Post-op patient who underwent open-heart surgery is determined to have a cardiac index of 1.7L/min/m^2 and a
left ventricular end diastolic pressure of 3mmHg. Identify that the cardiac index is low, we expect 5L/min in normal
adult and average adult is 1.7m^2, thus normal cardiac index is about 3-3.5L/min/m^2. If end diastolic pressure is
high, it means the ventricle cannot push out more fluid. Here, the pressure is low so the answer is give more fluids.
* Cardiac catheterization shows square root sign, equalization of pressures (RA, RV diastolic, PA diastolic, PCWP,
LV diastolic), in a patient with dyspnea on exertion, hepatomegaly, and ascites. Answer is chronic constrictive
pericarditis. Answer for next step is surgical repair.
--------------------------------------------------------------------------------------------------------------------------------------------
Cardiothoracic Surgery: Lung Cancer
* If central cancer of the lung patient needs pneumonectomy. If pulmonary function compromised already, we can’t
do the pneumonectomy. So, we should not do the diagnostic study in the first place. FEV1 should be at least 800mL
after a pneumonectomy, thus we do pulmonary function studies to find FEV1 of both lungs and a
ventilation/perfusion scan to determine what percentage of FEV1 comes from each lung. If distal metastasis, other
lung metastasis, or sternal metastasis at level of carina, we cannot due curative surgery.
* On a routine pre-employment physical examination, a CXR is done on a 45yo chronic smoker, coin lesion found
in upper lobe of the right lung. Answer for next step is look for recent CXR to compare with; if coin lesion existed
two years ago then this is not cancer of the lung as it would be growing. In this case we just do follow-up with
physical exam and chest x-rays occasionally.
* 75yo man with a 40pack*year history of smoking gets a CXR due to persistent cough. A peripheral, 2cm coin
lesion is found in the right lung, CXR two years ago was normal. This is high likelihood of cancer; above age 50,
coin lesion has >80% chance of malignancy. Answer for next step is sputum cytology (might give diagnosis, low
yield) and CT scan including chest and upper abdomen (e.g. liver). If lesion is peripheral coin lesion then we would
have to do a percutaneous lung biopsy, if central lesion then we would have to do bronchoscopy and biopsy.

DO NOT DISTRIBUTE - 45 -
Study Notes – Surgery James Lamberg 28Jul2010

* 66yo man with a 40pack*year history of smoking gets a CXR due to persistent cough. A peripheral, 2cm coin
lesion is found in the right lung, CXR two years ago was normal. CT scan shows no calcifications in the mass
(calcified mass in the lung is likely to be benign). CT scan shows no liver mets and no enlarged peri-bronchial or
peri-tracheal lymph nodes. Sputum cytology, bronchoscopy, and percutaneous needle biopsy have not been
diagnostic. The man has good pulmonary function and is otherwise in good health. So, we know there is a high
likelihood of cancer, a high likelihood of cure with lobectomy, and nothing preventing treatment (e.g. mets). Answer
for next step is thoracotomy and wedge resection.
* 72yo chronic smoker with severe COPD is found to have a central hilar mass on CXR. Sputum cytology
establishes a diagnosis of squamous cell carcinoma of the lung. Answer is evaluate pulmonary function to determine
if a candidate for pneumonectomy. His FEV1 is 1100 and a ventilation perfusion scan shows 60% of his pulmonary
function comes from the affected lung. If we did a pneumonectomy, we would have 440 FEV1 left, which is <800.
Thus, this patient cannot undergo a pneumonectomy. Answer for next step is palliative treatment, meaning radiation
and chemotherapy.
* 62yo chronic smoker has an episode of hemoptysis, CXR shows central hilar mass, bronchoscopy and biopsy
diagnosis a squamous cell carcinoma of the lung. FEV1 is 2200 and V/Q scan shows 30% of function comes from
the affected lung. Here, we remove 30% of 2200, and are still above 800 for the remaining lung. Thus, a
pneumonectomy could be done. We must also check for metastasis, so CT scan of liver including looking for other
lung nodes and tracheal nodes. Next step is cervical mediastinal exploration then pneumonectomy.
* 33yo woman is undergoing a diagnostic workup because she appears to have Cushing syndrome. CXR shows a
central 3cm round mass on the right lung. Bronchoscopy and biopsy confirms small cell carcinoma of the lung.
Answer for small cell cancer of the lung is go straight to radiation and chemotherapy. Answer is not pulmonary
function tests, bronchoscopy, or surgery including pneumonectomy. Cushing due to paraneoplastic syndrome.
--------------------------------------------------------------------------------------------------------------------------------------------
Peripheral Vascular Surgery
* In atherosclerotic occlusive disease, we treat the cause of the current problem (e.g. stroke, MI).
* A 54yo right-handed laborer notices coldness and tingling in his left hand as well as pain in his forearm when he
does strenuous work. What concerns him, during the last few episodes he notices transitory vertigo, blurred vision,
and difficulty articulating his speech. If the patient had problem when he used his common hand, answer is likely
thoracic outlet syndrome. Here there are neurologic complications involved as well. Answer is subclavian steal
syndrome, with plaque in artery before takeoff of the artery. During exercise, blood is siphoned away from
ipsilateral vertebral artery. Answer for next step is angiogram, demonstrating retrograde flow through the vertebral
artery. So, symptoms again are upper limb claudication with neurologic symptoms associated with the back of the
head (e.g. visual cortex, cerebellum, equilibrium). Answer for management is bypass graft to bring blood to the arm
without having to steal it from the brain.
* 62yo man is found on physical exam to have a 6cm pulsatile mass deep in the abdomen between the xiphoid and
umbilicus. Answer is abdominal aortic aneurysm (AAA). Typically they are asymptomatic, found on physical exam,
sonogram, or CT scan done for another reason. As a rule, if AAA is 6cm or larger, it must be repaired. If 4cm or
smaller, we can safely watch and wait. Answer for next step is precisely measure the AAA, done by sonogram
(cheap) or CT scan (more expensive). Answer is elective repair of AAAs that are at risk (>6cm). As a note, an
arteriogram is not needed and may even show a normal looking artery.
* 62yo man has vague poorly described epigastric and upper back discomfort. Physical exam shows 6cm pulsatile
mass deep in abdomen between xiphoid and umbilicus. The mass is tender to palpation. In this case, the AAA will
likely rupture within a couple days. Answer for next step is sonogram and vascular consultation for surgery.
* 68yo man is brought to the ED with excruciating back pain that began suddenly 45mins ago, is diaphoretic,
systolic of 90, 8cm pulsatile mass deep in abdomen above umbilicus. Typically, the AAA leaks blood into
retroperitoneal space before blowing up into the peritoneal space. This aneurysm is rupturing now. Answer is not x-
ray of lumbar spine, myelogram, etc. Answer is get vascular surgeons now, quick sonogram, immediate surgery.
* Wealthy retired man has claudication when walking more than 15 blocks. We do not know if claudication will get
better or not, thus we do not do prophylactic surgery. The only thing we know is if the patient smokes the
claudication will get worse. Claudication is operated on when it interferes with the lifestyle of the patient. Answer in
this situation is tell the patient to walk only 14 blocks then rest; the point is do not to risky surgery to palliate a
symptom that in itself is not devastating. Answer does not even include a diagnostic workup.
* 56yo postman describes severe pain in his right calf when he walks 2-3 blocks. The pain is relieved when he rests
for a couple minutes but recurs when he walks the same distance again. He cannot perform his job duties with this
pain and does not require for retirement. He does not smoke. Answer is to do a workup, which starts with Doppler
studies to find pressure gradient. If we no gradient found, disease is affecting small vessels and we have no surgical

DO NOT DISTRIBUTE - 46 -
Study Notes – Surgery James Lamberg 28Jul2010

solution. If a gradient found, then we do arteriogram to show vessels and obstructions. Finally, appropriate surgical
repair is done; if long segment in iliac area we do a bypass such as an aortobifemoral bypass with Dacron (plastic)
prosthesis. If long occlusion is in femoral area, like superficial femoral, then femoropopliteal bypass with saphenous
veins. If small segment of occlusion, we can do angioplasty and stent.
* Patient consults you because he cannot sleep. On questioning he has pain on the right calf that prevents him from
falling asleep, pain goes away when he sits at edge of bed and dangles the leg. Wife has seen this and notices leg is
pail when lying down and gets deep purple when he sits up. On physical exam the leg is shiny, no hair, and there are
no palpable peripheral pulses. This is resting pain, the penultimate stage (ultimate stage is gangrene). The patient has
pain when lying down because during they day they ignore the pain and have learned to be sedentary and not use the
affected leg. Answer for next step is Doppler studies of the leg and appropriate bypass as needed.
* 45yo man shows up in ED with a pale, cold, pulseless, paraesthetic, painful, and paralytic lower extremity. The
process began suddenly two hours ago, pulse in wrist is 95 and is grossly irregular. Irregular pulse (Afib) or post
myocardial infarction are the two situations where a clot can occur. Answer for next step is Doppler study. Answer
for management is emergency embolectomy with a Fogarty catheter, threading catheter past clot, filling a balloon,
and retrieving the clot.
* If there is an incomplete occlusion of a vessel, you might resort to clot-busters instead of Fogarty catheter. If the
patient had the symptoms for a long time like 6-8hours, then we will likely get compartment syndrome when we
reperfuse the ischemic limb. So answer in that case is embolectomy and fasciotomy.
* 74yo man has sudden onset of extremely severe, tearing chest pain, that radiates to the back and migrates down
shortly after onset. His BP is 220/110, unequal pulses in upper extremities, CXR shows wide mediastinum. Cardiac
enzymes and ECG are negative for myocardial infarction. Answer is thoracic aortic dissection. Answer for next step
is CXR if not done, to show widened mediastinum. Answer for next step is spiral CT scan, transesophageal echo
(more intrusive), or MRI angiogram (less common). Answer for management, if in ascending aorta is emergency
surgery, if in descending aorta is medical management such as control hypertension.
--------------------------------------------------------------------------------------------------------------------------------------------
Surgery: Dermatology
* 65yo West Texas farmer of Swedish ancestry has indolent raised waxy 1.2cm skin mass on the bridge of the nose
that has been slowly growing over the past three years. No enlarged nodes found in the head or neck. Answer is
basal cell carcinoma.
* 71yo West Texas farmer of Irish ancestry has a non-healing indolent punched-out clean looking 2cm ulcer of the
left temple that has slowly grown over the past two years. No enlarged nodes found in the head or neck. Answer is
basal cell carcinoma or squamous cell carcinoma. Basal cell carcinoma grows slower. Basal cell carcinoma does not
go to lymph nodes. Basal cell carcinoma occurs above the mouth line and squamous cell is below the mouth line.
So, this is most likely basal call carcinoma. Answer for first step is full-thickness biopsy at the edge of the lesion,
containing normal skin, tumor, and transition zone. Basal cell carcinoma does not require going far past the margin
of the tumor for removal.
* Blonde blue-eyed 69yo sailor has non-healing indolent 1.5cm ulcer on the lower lip that has been present and
slowly growing for the past 8-months. He is a pipe smoker and has no other findings. Answer is squamous cell
carcinoma. Answer for first step is full-thickness biopsy at the edge of the lesion, containing normal skin, tumor, and
transition zone. We must include a margin of normal skin at least 0.5cm to 1 or 2cm depending on location. We also
have to do a good physical exam and determine if a lymph node dissection is required.
* A redheaded 23yo lady who worships the sun and happens to be full of freckles, consults you for a skin lesion on
the shoulder that concerns her. She has a pigmented lesion that is asymmetric with irregular borders and of different
colors, measuring 1.8cm. Mnemonic is ABCD, asymmetry, border irregularity, color differences, diameter > 0.5cm.
Answer is melanoma.
* 35yo blonde blue-eye man, left his native Minnesota at age 18, has been living his life in the Caribbean as a yacht
crewmember. He has multiple nevi all over his body, but one has changed recently. Answer for next step is full-
thickness biopsy at edge of lesion, looking for diagnosis of melanoma as well as the depth of the melanoma.
Treatment depends on how invasive the melanoma is and that is dependent on depth. Invasive melanoma requires a
much larger excision at primary site and care taken toward lymph nodes. Previously, the Clark classification was
used for depth, using histological classification, with level 1-2 being excisable and 4+ requiring invasive
intervention including nodes. New classification being used is Breslow, which uses millimeters of depth at 0.75mm,
1.5mm, and 3mm. For up to 0.75 is superficial melanoma, and more than 3mm is invasive. However, some changes
in millimeter depth are being made; Australia is pushing to use <1mm and >4mm for conservative treatment and
poor prognosis, respectively. For invasive melanoma, the growth is unpredictable. Melanoma metastasizes to

DO NOT DISTRIBUTE - 47 -
Study Notes – Surgery James Lamberg 28Jul2010

common locations such as lymph nodes, liver, lung, brain, and bone. In additionally, melanoma can metastasize to
abnormal location like wall of ventricle or wall of duodenum.
* 44yo man has unequivocal signs of multiple liver metastases. No primary tumor has been identified by multiple
diagnostic studies of the abdomen and chest. The only abnormality on physical exam is a missing toe, which was
removed at age 16 for a black tumor under the toenail. Answer is metastasis from melanoma of the toe.
* Another case would be multiple metastases and only physical exam finding is a missing eye. Patient says there
was a tumor of the eye many years ago. Answer is melanoma of the choroid in the eye, now metastases found.
* 32yo gentleman has Clark level 5, 3.4mm deep melanoma, removed from middle of the back four years ago. He
now has a tumor growing in the ischiorectal area and one in the wall of the ventricle. Answer is melanoma
metastasis. There is no predictable behavior, so we do not know how likely the patient is to die. Thus, we will resect
the tumors found in abnormal locations because we do not know.
--------------------------------------------------------------------------------------------------------------------------------------------
Surgery: Ophthalmology
* 1yo child has suspected strabismus, physical exam shows reflection from eyes come from different locations. We
develop the ability to view images within the initial years of life. Answer is surgical correction now.
* If child is 4yo and just got strabismus, answer is refraction problem. Answer is give child glasses.
* 2yo child has huge pedunculated lipoma hanging out of upper eyelid, obstructing vision in right eye. Answer is
surgical removal to prevent amblyopia.
* Young mother is visiting your office for routine medical care. She has her 18-month-old baby with her, and you
notice one of the baby’s pupils is white and one is black. Answer is either congenital cataract (bad) or
retinoblastoma (very bad). Answer is immediate consult to the ophthalmologist for removal of tumor.
* 53yo Eastern Asian woman is in the ED complaining of extremely severe frontal headache. Pain started an hour
ago after she left the movies, watching a double feature. She sees halos around the lights of the parking lot when
leaving the theater. On physical exam the pupil is mid-dilated, doesn’t react to light, and corneas are cloudy with
greenish hue, and affected eyes feel hard as a rock. Answer is acute angle closure glaucoma. Fluid is trapped in the
anterior chamber and the pupillary dilation makes it worse, thus patients with glaucoma get eye drops to constrict
the pupils. Word glaucoma means “green.” Answer is give acetazolamide (Diamox), pilocarpine drops, or mannitol.
Next step is refer to ophthalmologist so a hole can be drilled in the iris to relieve the pressure.
* 32yo woman presents in ED with swollen, red, hot eyelids on the left eye. She has a fever and leukocytosis. When
prying the eyelids open you can see the pupil is dilated and fixed, with limited motion of the left eye. Answer is
orbital cellulitis, an emergency, identified by dilated fixed pupil due to pus behind the eye. Answer for next step is
CT scan and emergency drainage of the pus before the optic nerve is destroyed.
* Frantic mother reaches you on the phone reporting that her 10yo boy accidentally splashed Draino (drain cleaner)
on his face and is screaming in pain. He complains his right eye hurts terribly. Answer for first step is massive
irrigation with water as soon as possible, under tap water for 30minutes before boy is brought to the ED. Once they
are in the ED, fragments of the toxic agent should be removed. Before going home, eye pH should be measured.
* 59yo myopic gentleman reports seeing flashes of light at night when his eyes are closed. He also sees floaters
during the day, numbering 10-20, and a cloud at the top of his visual field. Answer is retinal detachment, an
emergency. The flashes of light at night imply a tugging of the retina, stimulating it. The cloud at the top implies
debris settling at the bottom of the eye. Answer for next step is refer to ophthalmologist and emergency laser
treatment to burn the retina into place and prevent further detachment.
* 77yo man suddenly loses sight from his right eye. He calls you immediately and reports no other neurological
symptoms. Answer is embolic occlusion of the retinal artery. Answer for next step is go to ED. En-route, have
patient breath into a paper bag and have someone push and release on his eye repeatedly. Breathing into a bag
creates vasodilation and pushing/releasing creates movement in the eye, with the hope that the clot will move further
down the arterial tree and occlude a branch instead of the larger trunk, causing less damage. In the ED, the
ophthalmologist may put a hole in the eye, deflating it, with the hope that the clot will move further along.
* 55yo man is diagnosed with type-II diabetes mellitus. Sometimes after a heavy dinner the T.V. looks blurry to him
and he has to squint to see it clearly. Answer is progressive diabetic retinopathy. Answer for management, which the
ophthalmologist would do, is laser treatment of the neovascularity that occludes vision.
--------------------------------------------------------------------------------------------------------------------------------------------
Surgery: Otolaryngology
* Neck masses can be congenital, inflammatory, or neoplastic. The timetable of presentation gives the first clue.
Congenital masses is in young children but present for several years. Inflammatory mass occurs within a few days or
weeks. Neoplastic mass occurs for a few months of relentless growth.

DO NOT DISTRIBUTE - 48 -
Study Notes – Surgery James Lamberg 28Jul2010

* 15yo girl has a round 1cm cystic mass in the midline of her neck at the level of the hyoid bone. When the mass is
palpated at the same time the tongue is pulled, there seems to be a connection between the two. The mass has been
present for several years but recently bothers the patient because it got infected. Answer is thyroglossal duct cyst.
Answer for management with removal, via Sistrunk procedure, where at the middle part of the hyoid bone to the
tongue is cored out. Some feel a thyroid scan is necessary but it is rarely useful.
* 18yo woman has a 4cm fluctuant round mass at the side of her neck, just beneath and in front of the
sternocleidomastoid. She reports that it has been there for ten years and has become larger recently. CT scan shows
the mass as cystic. A dimple of the skin is seen over the area of the mass. Answer is branchial cleft cyst. Answer for
management is surgical removal, taking great care not to injury structures in the area.
* 6yo child has a mushy fluid-filled mass at the base of the neck, noted for several years, 6cm in diameter, occupies
most of supraclavicular area, seems to go deeper into the chest on physical exam. Answer is cystic hygroma. Answer
for next step is CT scan to determine if a thoracic approach is needed on top of the neck approach.
* 22yo lady notices an enlarged lymph node in her neck, in jugular chain, is 1.5cm, is not tender, and was
discovered by the patient yesterday. The rest of the history and physical exam are unremarkable. Answer is much
more likely to be inflammatory than neoplastic. Answer for management is give it time, wait 3-4 weeks, not CT scan
or putting a needle in it. The timetable of an inflammatory mass will make things obvious 3-4 weeks later, and if it is
a tumor then we did not wait far too long. Antibiotics really aren’t necessary here.
* 22yo woman seeks help regarding an enlarged lymph node in her neck, in jugular chain, 2cm, firm, non-tender,
discovered six weeks ago. There is a history of low-grade fever and night sweats for past three years, physical exam
reveals axillary lymph nodes and nodes in groin. Answer is lymphoma. Rule 1, for neoplastic nodes, young person
with multiple nodes is lymphoma. Rule 2, anode in the supraclavicular area (Virchow node) is from a primary tumor
that is below the clavicle, e.g. gastric cancer, lung cancer, colon cancer, etc. Rule 3, node in the neck that is steadily
growing in an old man who smokes, drinks, and has rotten teach is metastatic squamous cell carcinoma and primary
tumor is in the head and neck. AIDS patients also get squamous cell carcinoma, with nodes in the neck. So, in this
original case it is lymphoma. Answer for first step of a large node in the neck is an FNA. If negative, must proceed
further. Answer then is surgical removal of most accessible or largest node for the pathologist to confirm lymphoma
and also establish which type.
* 72yo man has 4cm hard mass in left supraclavicular area. Mass is movable, nontender, present for 3 months, 20lb
weight loss over 3 months. Answer is look for primary tumor below clavicle. We may get a clue, like chronic
cougher with hemoptysis (lung), blood in the urine (kidney), etc. Answer for first step is do an FNA of the mass then
remove and biopsy the node if FNA is negative. This may identify the specific histological type and tell where the
primary tumor is.
* 69yo man who smokes, drinks, has bad teeth, has hard fixed 4cm mass in neck. The mass is medial and in front of
the SCM at thyroid cartilage, has been there for 6 months, and is growing. Answer is metastatic squamous cell
carcinoma, primary is somewhere in the mucosa of the head and neck. Answer is not to biopsy the node. Answer is
search for primary tumors by triple endoscopy (pan endoscopy) with biopsies. We do not biopsy the original node
because we may need a radical neck dissection with resection of the floor of mouth, part of tongue, larynx, or other
places. The incision we just made for the node dissection messes up the radial neck dissection, so it is best to have a
virgin neck to operate on.
* 69yo man who smokes, drinks, has bad teeth, has hoarseness persisting for 6 weeks despite antibiotic therapy.
Answer is squamous cell carcinoma in or near the vocal cords.
* 69yo man who smokes, drinks, has bad teeth, has a painless ulcer in the floor of the mouth present for 6 weeks and
has not healed. Answer is squamous cell carcinoma at that ulcer location. Patient still needs pan endoscopy.
Combination of smoking and drinking are multiplicative. Bad teeth is not additive, it just indicates the individual has
very poor hygiene, a reflection of their lifestyle.
* 23yo man with AIDS has painless ulcer in the floor of the mouth, not healing, does not smoke or drink. Answer is
squamous cell carcinoma.
* 69yo man who smokes, drinks, has bad teeth, has unilateral earache that has not gone away for several weeks.
Physical examination shows serous otitis media on that side but not on the other. Answer is squamous cell
carcinoma occluding the eustachian tube. Answer for next step is triple “pan” endoscopy.
* 52yo man complains of hearing loss. When tested he is found to have unilateral sensory hearing loss on one side
only. He does not engage in any activity such as sport shooting that would subject that ear to excessive noise. If
conductive hearing loss it would be cerumen plug or middle ear problem. If sensory hearing loss, most commonly
bilaterally such as working in a noisy factory or a private pilot flying a loud plane. Answer here is tumor affecting
the 8th cranial nerve. Answer for next step is MRI looking for tumor in the head.

DO NOT DISTRIBUTE - 49 -
Study Notes – Surgery James Lamberg 28Jul2010

* 56yo man develops slow progressive paralysis of the facial nerve on one side. It took several weeks for the
paralysis to become obvious and has been present for three months. It affects both the forehead and lower face. If
normal person went to bed and woke up with half-face paralysis, it would be Bell palsy. This case is progressive
however. Answer for next step is MRI scan, with gadolinium enhancement.
* 45yo man presents with 2cm firm mass in front of left ear, present for 4 months. The mass is deep to skin and
painless, patient has normal function of facial nerve. Any mass around the ear should be diagnosed as parotid tumor
until proven otherwise. Most common tumor form is pleomorphic adenoma, which is benign with malignant
potential, and presents with no pain. There is only one shot at doing an elective removal of the pleomorphic tumor.
Answer for management is complete parotidectomy or superficial parotidectomy, which can spare the facial nerve in
virgin territory. If we do a nucleation of the tumor, it will for sure grow back, then a parotidectomy is much more
difficult as the scar tissue can bind to the facial nerve, preventing preservation. A FNA could be used here, but
nothing more invasive than that before full surgery. The person doing the FNA may accidentally transect a branch of
the facial nerve. One case not to do an FNA is in the liver with suspected hemangioma. An FNA near the parotid
gland could create a salivary fistula. Answer is to refer to an expert who can do the formal parotidectomy.
* 65yo man presents with 4cm hard mass in front of the left ear, which has been present for 6 months. The mass is
deep to the skin and fixed. He has constant pain in the area and for the past two months has had gradual progression
of left facial nerve paralysis, has rock hard lymph nodes of the left neck. Answer is cancer of the parotid gland.
Answer for next step is referral to expert to do a total parotidectomy with sacrifice of facial nerve, lymph node
dissection, and a nerve graft using a nearby sensory nerve.
--------------------------------------------------------------------------------------------------------------------------------------------
Surgery: Pediatric Otolaryngology
* 2yo has unilateral earache. Answer is foreign body. Answer is removal of the foreign body.
* 2yo has foul smelling purulent rhinorrhea. Answer is foreign body. Answer is removal of the foreign body.
* 2yo has unilateral wheezing and lung looks darker on that side. Answer is foreign body. Answer is removal of the
foreign body, including anesthesia if endoscopy is needed. Note that many of the toys children stick in their ears,
nose, or throat are radiolucent.
--------------------------------------------------------------------------------------------------------------------------------------------
Surgery: Otolaryngology Emergencies & Miscellaneous
* 45yo lady with recent tooth infection has huge, hot, red, tender, fluctuant mass on left lower side of face and upper
neck, including underside of the mouth. The mass pushes up the floor of the mouth on that side. She is febrile.
Answer is abscess, Ludwig angina. Answer is incision and drainage, including intubation or tracheostomy.
* 29yo lady calls at 10am saying she woke up with one side of her face paralyzed. Answer is Bell palsy. This is
relatively benign, but antiviral medications can be used and should be given as early as possible. That is why
healthcare workers who get needlesticks must get antiviral (anti-HIV) medication immediately, within first hour.
Answer is have lady be seen to rule out stroke then give antiviral medication.
* A patient with multiple trauma from a car accident is being attended to in the ED. As multiple invasive things are
being done, he repeatedly grimaces with pain. He is noted the next day to have facial nerve paralysis on one side. If
the patient came into the ED unable to move face, then operative intervention is needed quickly. In this case, there is
swelling of the nerve and nothing needs to be done. The paralysis will go way.
* You get a call from a middle-aged lady who you have repeatedly treated in the past for sinusitis. Six days ago you
started her on decongestants and antibiotics, for what you diagnosed as frontal and ethmoidal sinusitis. Now she tells
you that ever since she woke up this morning she is seeing double. Answer is either orbital cellulitis or cavernous
sinus infection, either way it is an emergency. Answer for next step is CT scan and immediate intervention.
* 10yo girl has epistaxis, mother says she often picks her nose. Answer is bleeding from anterior part of septum due
to direct trauma. Answer for management is local pressure, anterior pack, or with epinephrine soaked packs.
* 18yo boy has epistaxis, patient denies picking his nose, no source of anterior bleeding is seen. Answer is bleeding
from septal perforation from cocaine abuse or posterior juvenile nasopharyngeal angiofibroma. The angiofibroma
destroys nearby structures so it must be managed surgically.
* 72yo hypertensive male on aspirin for arthritis has a copious nosebleed. His BP is 220/115 in the ED, he began
swallowing blood before it came out of front of nose. Answer is hypertensive nosebleed, typically in posterior part
of septum, and can be life threatening. Answer is not local pressure or anterior epinephrine packing. Answer for
management is posterior pack and ENT consult for possible ligation of vessels.
* 57yo man seeks help for dizziness. He explains that he gets light headed and unsteady but the room is not spinning
around. Answer is problem in head, cerebellum. Management includes examining blood supply going to head.
* 57yo man seeks help for dizziness. He explains that the room spins around him and that he is very frightened.
Answer is problem in the inner ear. Management includes inner ear workup.

DO NOT DISTRIBUTE - 50 -
Study Notes – Surgery James Lamberg 28Jul2010

--------------------------------------------------------------------------------------------------------------------------------------------
Neurosurgery: Brain
* Vascular problems have sudden onset, without headache means occlusive, with headache means hemorrhagic.
* Typical timetable of a brain tumor is a few months of a progressive headache. A headache that is worse in the
morning, eventually accompanied by other symptoms like projectile vomiting. Even fast headaches like
glioblastoma multiform take two months. Timetable for degenerative disease is years. Timetable for infectious
disease is a few days and may be associated with recent infection in the area. If the problem is metabolic, it will
have a quick onset but be generalized with symptoms.
* 62yo right-handed man has transient episodes of weakness in the right hand, blurred vision, and difficulties
expressing him self. There is no headache, the episodes have sudden onset, and last 5-10 minutes, and resolve
spontaneously leaving no neurological sequela. Answer is occlusive neurological problem, a transient ischemic
attack (TIA). Most TIAs come from disease in the neck vessels. Here it is the left carotid artery. Most of the time the
offending lesion will be stenosis of >70% of the lumen at the carotid bifurcation or an ulcerated plaque at the carotid
bifurcation, throwing off emboli to the brain. TIAs imply an ischemic stroke is on the way. Answer for diagnostic
test is duplex scanning of the neck, a high-quality image of the carotids with Doppler studies of the flow. The test of
choice use to be arteriogram, but one side effect was embolic stroke, so now the duplex scan is the test of choice.
Arteriogram only used if duplex scan doesn’t provide enough information. Answer for management is carotid
endarterectomy, removing the plaque, tack the intima back in place, and close the incision without occluding the
area. This greatly diminishes the risk of future stroke.
* 61yo man presents with one-year history of vertigo, diplopia, blurred vision, dysarthria, and instability of gait. The
episodes have sudden onset, last several minutes, have no associated headache, and no neurological sequela. Answer
is TIA involving the vertebral artery. Answer still is start with duplex scan of carotids. Next step is an arch study,
which is an arteriogram where the dye is put into the arch of the aorta and it travels up both vertebral and carotid
arteries to see all 4 vessels.
* Last week, a 60yo diabetic male had a prompt onset of right third never paralysis and contralateral hemiparesis, no
associated headache, patient alert, neurological deficits have not resolved for a week. Answer is an ischemic stroke,
not a TIA, so it is too late to do revasculization of the brain. If we happened to reperfuse that ischemic area, it would
turn into a hemorrhagic stroke and be worse. Answer for next step is CT scan. Answer for management is
rehabilitation. Eventually we will study the neck vessels to prevent future strokes.
* 64yo black man complains of a very severe headache of sudden onset and then lapses into a coma. Past medical
history includes untreated hypertension, exam reveals a stuporous man with profound weakness in the left
extremities. Answer is vascular hemorrhagic problem, a hemorrhagic stroke. Common causes of hemorrhagic stroke
are hypertension, arteriovenous malformations, and Berry aneurysm. Answer for diagnostic test is CT scan. Answer
for management is rehabilitation efforts and control of hypertension to prevent future strokes.
* 39yo woman presents to the ED with a severe headache of sudden onset, which she says is different and worse
than any headache she has had before. Neurological exam is normal, so she is given pain medication and sent home.
She improves over the next few days but 10 days later she gets a sudden, severe, singular, and diffuse headache and
returns to the ED. Nuchal rigidity is found. Answer is subarachnoid bleed from intracranial Berry aneurysm. Answer
for next step is CT scan, showing blood in subarachnoid space. Then angiogram to find bleeding location. Finally,
neurosurgery consult so they can fix the problem. Headache may also be called “thunderclap headache.”
* 31yo nursing student develops persistent headaches that began about four months ago, have gradually increased in
intensity and are worse in the morning. She has had projectile vomiting for three weeks. Thinking that she may need
new glasses due to blurred vision, she goes to the optometrist who finds bilateral papilledema. Answer is brain
tumor, but we may not be able to determine location from symptoms. Answer is for next step is MRI, which is the
best tool for seeing tumors. Answer also includes lowering intracranial pressure (ICP) due to tumor, which is high
dose steroids, namely dexamethasone (Decadron). Could also include hyperventilation, mannitol, and furosemide.
* 42yo right-handed man has history of progressive speech difficulties and right hemiparesis for five months. He has
had progressively severe headaches for two months, is confused on admission, has vomiting, blurred vision,
papilledema, and diplopia. Shortly after his BP goes to 190/110 and he develops bradycardia. Answer is brain tumor,
on the left side pushing on motor and speech centers lower in the temporal lobe. The hypertension with bradycardia
is the Cushing reflex, indicating very high ICP. Answer for management is MRI of head and emergency measures to
lower ICP. Then, craniotomy to remove tumor soon after.
* 42yo man has been fired from his job because of inappropriate behavior. For the past two months he has gradually
developed very severe, explosive headaches on the right side above the eye. Neurologic exam shows optic nerve
atrophy on the right, papilledema on the left, and anosmia. Answer is brain tumor, base of frontal lobe on the right
side (Foster Kennedy syndrome). Answer for next step is MRI then surgery.

DO NOT DISTRIBUTE - 51 -
Study Notes – Surgery James Lamberg 28Jul2010

* 12yo boy, short for his age, has bitemporal hemianopsia, and a calcified lesion above the sella on x-rays of the
head. Answer is craniopharyngioma. Answer for next step is MRI then craniotomy to remove tumor.
* 23yo nun presents with amenorrhea and galactorrhea of six months duration. She is concerned others will think
she is pregnant and absolutely denies that is the case. Answer is prolactinoma. Test for beta hCG anyway. Answer
for diagnostics is measure prolactin then do an MRI (may be very small tumor in pituitary). Answer for management
is bromocryptine, if unresponsive to therapy or patient wishes to become pregnant then transnasal transsphenoidal
removal of the tumor.
* 44yo man is referred for treatment of hypertension. He has big fat sweaty hands, large jaw, thick lips, large tongue,
huge feet, and a touch of diabetes. Has headaches and his wedding ring no longer fits his finger. Answer is
acromegaly. May be shown a picture of a patient with hands up on either side of the face. Answer for next step is
measure somatomedin C (insulin like growth factor, IGF-1). Some say glucose suppression test. Then do MRI for
pituitary adenoma, finally pituitary surgery or radiation.
* 15yo girl has gained weight and become ugly. She shows a picture from a year ago when she was a lovely young
lady. Now she has a hairy, red, round face full of pimples, has posterior neck hump, fat supraclavicular areas, fat
trunk, thick extremities, mild diabetes, hypertension. Answer is Cushings. Answer for testing is AM and PM cortisol
levels, will be elevated and no longer have a different, then dexamethasone suppression test. If suppresses at a low
dose, no disease. If suppresses at higher dose, microadenoma in pituitary. No suppression at high dose, then
adenoma in adrenal gland itself or paraneoplastic syndrome.
* 55yo woman is involved in a minor traffic accident where her car was hit sideways by another car that she did not
see at an intersection. She appears tanned and is found to have bitemporal hemianopsia. Ten years ago she had
bilateral adrenalectomies for Cushing disease. Patient might not even know they have bitemporal hemianopsia
because most concentrate on their central vision, thus she missed the side traffic. Another example would be a man
driving down a rural road and hitting mailboxes. Before MRI technology, microadenomas were not seen, thus
bilateral adrenalectomies were done thinking that was the primary location of the problem. Well, this cured the
problem temporarily, until the tumor grew huge 10-years later, causing bilateral hemianopsia. Answer is Nelson
syndrome. Answer now is MRI then surgical removal. Patient is tan because she is making ACTH due to adrenal
gland removal and along with ACTH is melanin stimulation.
* 27yo woman presents with six-month history of headaches, visual loss, and amenorrhea. Upon admission, she has
severe headache, marked deterioration of remaining vision, and stupor. BP is 75/45, fundoscopic reveals bilateral
pallor of the optic nerves. Answer is pituitary apoplexy. Lady has pituitary tumor then she bled into that tumor,
giving sudden headache and deterioration of neurological function. This also destroyed the pituitary gland and she is
in shock due to adrenal insufficiency. Answer for management is give steroids, else she will die.
* 32yo man complains of progressive severe generalized headaches, began three months ago, are worse in the
mornings, and lately have been accompanied by projectile vomiting. He has lost his upward gaze and has sunset
eyes (setting sun sign). Answer is a brain tumor, at the location of the pineal gland, Parinaud syndrome. Answer for
next step is MRI, then neurosurgery.
* 6yo boy has been stumbling around the house and complaining of severe morning headaches for several months.
While waiting in the office to be seen, he assumes the knee-chest position as he holds his head. Neurological exam
demonstrates truncal ataxia. In adults, most tumors are above tentorium. In children, most tumors are below the
tentorium, in the posterior fossa. Answer is brain tumor, in the location of the cerebellum. If tumor is on a little
pedicle, it can occlude the aqueduct of Sylvius, increasing ICP, making headache really bad, and kid realizes the
knee-chest position causes the tumor to swing and the CSF can flow. Answer is MRI then neurosurgery.
* 23yo man develops severe headaches, seizures, and projectile vomiting over two weeks. Has low grade fever,
recently treated for acute otitis media and mastoiditis. Answer is brain abscess. Answer for next step is CT scan, not
expensive MRI. Answer for management is neurosurgical resection, not just drainage.
--------------------------------------------------------------------------------------------------------------------------------------------
Neurosurgery: Spinal Cord
* 52yo woman has constant severe back pain for two weeks, while working in her yard she suddenly falls and
cannot get up again. In ED, is paralyzed below the waist. Had mastectomy two years ago for breast cancer. Answer
is cancer metastasis to vertebral bone pedicle. Answer is also fractured vertebra with spinal cord damage. Answer
for first step is x-ray, not bone scan as bone is broke and we’re not looking for early metastasis. Answer for step
after that is MRI to look at the spinal cord directly. Then, consult neurosurgery to decompress.
* 45yo man has aching back pain for several month, told he has muscle spasms, given analgesics and muscle
relaxants, comes back now because of sudden onset of severe back pain when he tried to lift a heavy object. The
pain is like an electrical shock shooting down his leg, worse with sneezing, prevents him from ambulating, keeps the
affected leg flexed, straight leg test gives severe pain. Answer is lumbar disk herniation, peak age 45, at L4-L5

DO NOT DISTRIBUTE - 52 -
Study Notes – Surgery James Lamberg 28Jul2010

(lightning exits near big toe) or L5-S1 (lightning exits by little toe). Answer for first step is MRI. Answer for
management is bed rest, unless progressive weakness or sphincter defects with saddle anesthesia (cauda equina
syndrome). Cauda equina is an emergency.
* 79yo man complains of leg pain with walking, relieved with rest. Has to sit down or bend over for the pain to go
away, standing at rest does not do it. He can exercise for long periods of time while hunched over, such as riding a
bike or pushing a shopping cart. He has normal pulses in legs. Answer is neurogenic claudication, osteophytes
growing into the spinal canal and compressing the cord. Answer for first test is MRI. Answer for management is
surgical decompression of the spinal canal.
* Business executive who has been a T6 paraplegic for many years, is held at a business meeting for several hours
beyond the time where he would have done his in-and-out self-catheterization of the urinary bladder. He develops a
pounding headache, profuse sweating, bradycardia, BP is 220/120. Answer is autonomic dysreflexia, triggered by
bladder distension or any other visceral stimulation. Answer for management is empty bladder and give alpha-
adrenergic blocking agents and possibly long-term calcium blockers to prevent future problems.
* 60yo man complains of extremely severe, sharp, shooting pain like a bolt of lightning in his face that is brought
about by touching a specific area. It lasts for about 60 seconds. Neurological exam is normal but part of face is
unshaven as the patient fears touching that area. Answer is trigeminal neuralgia (tic douloureux). Answer for
management is anticonvulsives like carbamazepine (Tegretol). Also, an MRI to rule out other lesions, basically for
legal liability. Another option is radio-ablation or heat ablation of the nerve.
* Several months after an arm crush injury, a patient complains of constant burning agonizing pain that does not
respond to the usual analgesic medications. The pain is stimulated by even the slightest touch in the area. The arm is
cold, cyanotic, and moist. Answer is reflex sympathetic dystrophy (causalgia). Answer for management is
sympathetic block. If that relieves symptoms, a surgical sympathectomy is expected to be curative.
--------------------------------------------------------------------------------------------------------------------------------------------
Urology: Emergencies
* 14yo boy presents to the ED with very severe pain of sudden onset in his right testicle, no fever, pyuria, or history
of recent mumps. Testes is swollen, painful, high-riding with a horizontal line, cord is not tender. Answer is
testicular torsion, an emergency. Answer for management is go to the operating room for an orchiopexy, no need for
further studies (no sonogram). Orchiopexy on the other side may be done as well to prevent future torsion.
* 24yo man presents to the ED with very severe pain of recent onset in his right scrotum, fever of 103F, pyuria.
Testes in normal position, swollen, very painful, cord also tender. Answer is acute epididymitis, an STD. Answer for
next step is rule out torsion by doing a sonogram. Answer for management is antibiotics.
* 72yo man is being observed with urethral stone that is expected to pass spontaneously. He then develops chills,
temp spike to 104F, and flank pain. Answer is combination of obstruction and infection. Answer for management is
immediate high-dose antibiotics and decompression of the urinary tract above the stone. Patient may get a
nephrostomy or catheter from below, but must get a catheter above.
* Adult female relates that five days ago she had frequent painful urination, but small volumes of cloudy and
malodorous urine. Answer is cystitis (bladder infection). However, for the first three days she had no fever. For the
past two days she has had chills, high fever, nausea, and vomiting. She also had right flank pain, no treatment up to
now. Answer is pyelonephritis, a serious condition. Answer for management is IV antibiotics and a urological
workup. Old gold standard test was intravenous pyelogram (IVP). IVP not useful for early bladder cancer, that
would need a cystoscopy. Some people are allergic to the IVP dye plus marginal renal function (creatinine above 2)
is a contraindication. So, answer for urological workup is CT scan or sonogram (sonogram has no
contraindications). “Only way to kill a patient with a sonogram? Hit the patient over the head with the machine.”
* 62yo man presents with chills, fever, dysuria, urinary frequency, diffuse low back pain, exquisitely tender prostate
on rectal exam. UTIs not common in men. Answer is acute bacterial prostatitis. Answer for management is
antibiotics and do not repeat the rectal exam.
* 33yo man has urgency, frequency, and burning pain with urination. Urine is cloudy and malodorous, he has mild
fever. Physical exam shows prostate is not warm, boggy, or tender. Answer is UTI. Answer for management is
antibiotics. But why is this male having a UTI? So, answer for management includes urinary culture and a urological
workup to determine if there is obstruction or congenital problem. So, do an IVP or sonogram. Never instrument an
infected bladder so do not pick cystoscopy.
--------------------------------------------------------------------------------------------------------------------------------------------
Urology: Congenital
* Called to the nursery for an otherwise healthy looking newborn boy because he has not urinated in the first 24
hours of life. Physical exam shows large distended bladder. Kids have to pee within first day of life, so answer is not
to reassure the parents. Answer is also renal atresia, because that comes with non-developed lungs. Answer is

DO NOT DISTRIBUTE - 53 -
Study Notes – Surgery James Lamberg 28Jul2010

obstruction, either meatal stenosis or posterior urethral valves. If meatus is open, not meatal stenosis. Answer for
next step is place a catheter to empty the bladder then do a voiding cystourethrogram. Answer for treatment is
endoscopic fulguration or resection of the valves.
* A bunch of newborn boys are lined up in the nursery for you to do circumcisions. You notice one has a urethral
opening on the ventral side of the penis, about midway down the shaft. Answer is hypospadias. Answer for
management is do not do the circumcision, as it may be needed for plastic reconstruction to move urethral opening
to the tip of the penis.
* 7yo child falls of a jungle gym and has minor abrasions and contusions. Urinalysis shows microhematuria. Minor
trauma should not product hematuria in a child. Child may have congenital anomaly. Answer is urological workup,
such as a sonogram and then IVP.
* 9yo boy gives a history of three days of burning urination with frequency, low abdominal and perineal pain, left
flank pain, fever, and chills. Little boy should not be getting a UTI, so must be congenital anomaly. Answer is give
antibiotics and urinary culture for UTI. Also, show reflux exists with IVP and voiding cystogram. Then, do long
term antibiotics as the child grows out of the problem, meaning the ureter gets pulled up with growth and is less
likely to reflux.
* Mother brings her 6yo girl to you because she has failed to get proper toilet training. Little girl has normal
sensation to void, voids normally and at appropriate intervals, but is wet with urine all the time. Answer is low
implantation of one ureter, which leaks all the time. This does not occur in boys. Physical exam may show ureter
emptying into the vagina or perineum. Answer for diagnosis for sure is IVP. Answer for management is surgery.
* 16yo boy has first time of beer binge drinking. Shortly after, at midnight, he develops colicky flank pain. Answer
is ureteropelvic junction obstruction. Some boys have narrow “kinked” lumen at urethral pelvis at the junction.
Normal urine flow can get through. If boy is challenged with a diuretic bolus, such as what occurs with alcohol,
obstruction occurs. Answer for diagnosis is ultrasound. Answer for management is surgical repair.
--------------------------------------------------------------------------------------------------------------------------------------------
Urology: Genitourinary Tumors
* Hematuria unrelated to trauma should always be worked up. Although most have benign disease, they could have
renal cell carcinoma or transitional cell carcinoma of the bladder or ureters. Workup is IVP, then cystoscopy.
* 62yo man known to have normal renal function reports an episode of gross, painless hematuria. Further
questioning reveals total (continuous) hematuria rather than initial or terminal hematuria. A problem with the
urethral or prostate, it won’t likely last the entire length of the urination. Answer for next step is IVP, and we know
the patient has normal renal function. Answer for next step is cystoscopy.
* 70yo man is referred for evaluation because of a triad of hematuria, flank pain, and flank mass. Has
hypercalcemia, erythrocytosis, and elevated liver enzymes. Answer is renal cell carcinoma, far advanced. Answer
for next step is IVP and cystoscopy. You already know it is renal cell carcinoma, so toss in CT scan.
* 61yo man presents with a history of hematuria, IVP shows renal mass. Sonogram shows it to be solid, not cystic.
CT scan shows heterogenic solid tumor. Answer is renal cell carcinoma.
* 55yo chronic smoker reports three instances in past two weeks of painless, gross, total hematuria. In past two
months he has been treated twice for irritating voiding symptoms, no febrile, negative urinary cultures. It’s
hematuria so you work this up. Answer is bladder cancer, associated with smoking even more so than lung cancer.
Answer is IVP then cystoscopy, could be a tumor in bladder and ureter thus you do IVP first. If poor renal function,
then you would do a CT scan rather than IVP.
* 59yo black man has rock hard discrete 1.5cm nodule found in prostate during rectal exam. Answer is likely
prostate cancer. Answer for next step is transrectal biopsy of the mass.
* 59yo black man has been told his PSA has gone up significantly since his last visit. He has no palpable
abnormalities in his prostate by rectal exam. May be cancer. Answer is transrectal sonogram of prostate to identify
the area then do needle biopsy. Answer for prostate cancer at early stage is radiation, surgery, radioactive seeds.
* 62yo man had a radical prostatectomy for cancer of the prostate three years ago. He now has widespread bone
pain. Bone scan shows metastasis throughout the entire skeleton, including several large spots. Answer for next step
is eliminating antigens for dramatic palliation, done by orchiectomy. The palliation will last a couple years.
Expensive alternative is luteinizing hormone releasing hormone (LHRH) agonists or anti-androgens like flutamide.
* 82yo man with CHF and COPD is told by his primary care physician that his prostate specific antigen (PSA) is
high. This should not have happened, as asymptomatic men get PSA and rectal exams at an early age. There is an
age limit for this though, as every male who lives long enough (unless castrated) will get prostate cancer. Men who
develop prostate cancer late in life will die of something else before the cancer so longevity is not changed by
testing. Age limit is 75 normally. Maybe a better question would be a patient who is 76yo asking if they need a PSA
and you explaining it is not needed.

DO NOT DISTRIBUTE - 54 -
Study Notes – Surgery James Lamberg 28Jul2010

* 25yo man presents with a painless hard testicular mass. Answer is testicular cancer. Answer for next step is radical
orchiectomy by inguinal route then ask the pathologist if it was a cancer. If there is a hard mass and you know
clinically it is from the testicle in a young man, it is cancer. Benign hard testicular masses in young men don’t exist.
Any trans-scrotal approach is incorrect, no trans-scrotal needle biopsy or trans-scrotal orchiectomy because it causes
the cancer to spread. Answer also includes blood samples to check alpha-fetoprotein and beta hCG.
* 25yo man is found on pre-employment CXR to have pulmonary metastasis from unknown primary tumor. Hard
testicular mass is found, lost weight over past six months for no reason. Answer is not palliative care. Most testicular
cancers are radiosensitive and chemo-sensitive, particularly to platinum-based chemotherapy.
--------------------------------------------------------------------------------------------------------------------------------------------
Urology: Retention & Incontinence
* 60yo man shows up in the ED because he has not been able to void for the past 12 hours, he wants to, but cannot.
On exam his bladder is palpable halfway up from the pubis to umbilicus and he has a big boggy prostate without
nodules. He gives a history of several years of nocturnal polyuria. Due to a cold, he took antihistamines, nasal drops,
and drank plenty of water. Answer is acute urinary retention with underlying benign prostatic hypertrophy (BPH).
Nasal drops usually have alpha-adrenergic agents, which also close the bladder neck. Answer for management is
indwelling Foley bladder catheter, not a simple straight catheter, left in for three days. Long-term management is
transurethral resection of the prostate (TURP), alpha-blockers, and mechanical options.
* 2nd post-op day after bilateral inguinal hernia repair, patient reports he cannot hold his urine. Every few minutes
he urinates a few mLs of urine. Exam reveals a large palpable mass arising from the pelvis and extending almost to
the umbilicus. Answer is acute urinary retention with overflow incontinence. This shouldn’t happen, as there is
usually an order for the patient to void within 6 hours after the operation. Answer for management is indwelling
bladder catheter.
* 42yo woman consults you for urinary incontinence, is the mother of 5 children with vaginal deliveries, every since
last birth she leaks a small amount of urine whenever she sneezes, laughs, gets out of a chair, or lifts heavy objects.
She relates that she can hold her urine all night without leaking. Answer is stress incontinence. With increased intra-
abdominal pressure, pressure is equal on the bladder and sphincter. But now the lady has a cystocele and urethra is
outside of the pressure area, so the bladder body gets pressure but not the urethra. Answer for management is
surgical repair of the pelvic floor.
--------------------------------------------------------------------------------------------------------------------------------------------
Urology: Stones & Miscellaneous
* 72yo who in the past year has passed three urinary stones, is now again having symptoms of urethral colic. Has
relatively mild pain that began six hours ago, does not have much nausea or vomiting. X-ray shows a 3mm urethral
stone just proximal to the ureterovesicular (UV) junction. The patient will likely pass this stone with time, it is
small, negotiated ureter already, and is ready to enter the bladder. Answer for management is plenty of fluid, pain
medication, and watchful waiting. Answer is not extracorporeal shock wave lithotripsy (ESWL), electrohydraulic
lithotripsy, laser holmium lithotripsy, or any other fancy gadget. This has about a 70% chance of passing through.
* 54yo woman has severe urethral colic, IVP shows 7mm stone at the ureteropelvic junction. This doesn’t have a
good chance of getting through urethra. This has about a 5% chance of passing through spontaneously. Answer for
next step is extracorporeal shock wave lithotripsy (ESWL), which involves having the patient in a water bath.
Contraindications for ESWL are pregnancy, bleeding diathesis, or stones are large (staghorn calculi).
* 72yo man for the past several days noticed bubbles of air in the urine when he urinates. He has symptoms of mild
cystitis. Answer is fistula between GI tract and bladder, mostly between sigmoid colon and mostly due to
diverticulitis. Answer is not barium in sigmoid, radiopaque dye in the bladder, endoscopy in the sigmoid, or
cystoscopy in the bladder. Any hole that could be seen would cause sepsis and kill the patient. This is a pinhole
fistula. Answer for diagnosis is CT scan, showing inflammatory diverticulitis. Then, do a proctosigmoidoscopic
exam to rule out cancer, which could be an option for the fistula formation. Finally, surgery to fix the problem.
--------------------------------------------------------------------------------------------------------------------------------------------
Urology: Impotence & Erectile Dysfunction
* 32yo man has sudden onset of erectile dysfunction. One month ago, he was unexpectedly unable to perform with
his wife after an evening of heavy eating and heavy drinking. Ever since then, he has not been able to achieve an
erection with his wife, but he still gets nocturnal erections and can masturbate normally. Answer is psychogenic
impotence (erectile dysfunction). Answer is psychotherapy soon, not delayed as it could become permanent.
* Ever since he had a motorcycle accident where he crushed his perineum, a young man has become impotent.
Answer is organic impotence, due to vascular injury. Answer for management is vascular repair.

DO NOT DISTRIBUTE - 55 -
Study Notes – Surgery James Lamberg 28Jul2010

* Ever since a perineal resection for rectal cancer, a 52yo man has been impotent. Answer is organic impotence, due
to plexus of erectile nerves near the rectum and sometimes cannot be spared during resection. Cannot be repaired.
Answer is prostatic devices such as vacuum device or implants that cause permanent erection.
* 66yo diabetic man with generalized atherosclerotic occlusive disease, notices gradual loss of erectile dysfunction.
At first he could get erections but they did not last long enough, later the quality was poor, eventually he had
complete impotence. He does not get nocturnal erections. Answer is organic impotence. Answer for management is
sildenafil (Viagra), which breaks down enzymes that normally terminate vasodilation on which erection depends.
--------------------------------------------------------------------------------------------------------------------------------------------
Transplant Surgery
* 62yo man who had a motorcycle accident has been in a coma for several weeks, has been on a respirator, had
pneumonia on and off, has been on vasopressors, and shows no sign of neurological improvement. The family
inquires about brain death and possible organ donation. There use to be a bunch of contraindications for organs,
such as elderly patients, not on vasopressors, no infections, etc. Now, the major problem is the disparity in the
number of organs available and the number of people needing transplants. Even now we use organs from patients
with hepatitis, given to patients who have hepatitis. Cancer patients can donate corneas these days. Answer for this
case is yes we can use this patient as an organ donor.
* Three types of solid organ rejection, hyperacute rejection (within minutes and due to preformed antibodies,
destroys graft immediately, not seen clinically anymore), acute rejection (within days to first month, decreased
organ function, even with immunosuppressants, can be reversed with bolus of steroids), and chronic rejection (years
after, poorly understood, insidious gradual loss of graft function, no treatment). For acute rejection, exceptions are
the liver (every technically complicated) and the heart (biopsies done on a schedule).
* Ten days after liver transplantation, levels of GGT, ALP, and bilirubin begin to go up. Answer for next step is
ultrasound for biliary anastomosis and Doppler studies for vascular anastomosis. Say we find no ultrasound
obstruction or Doppler thrombosis. Answer for next step is percutaneous biopsy of the liver. If acute rejection is
diagnosed, give steroid boluses or anti-lymphocytic agents.
* On third week after a closely matched renal transplant, there are early clinical signs of decreased renal function.
Answer is biopsy of the renal graft. Likely shows acute rejection, give steroid boluses.
* Two weeks after a lung transplant, patient has fever, dyspnea, hypoxemia, decreased FEV1, and interstitial
infiltrate on chest x-ray. Answer is endobronchial biopsies and treat rejection episode.
* Several years after a successful transplantation, there is gradual insidious loss of organ function. Answer is chronic
rejection. Answer for next step is organ biopsy, as there is a chance it is a late version of the acute rejection. If the
problem is chronic rejection, answer for management is look for another donor for graft replacement.
--------------------------------------------------------------------------------------------------------------------------------------------

DO NOT DISTRIBUTE - 56 -

S-ar putea să vă placă și